Sei sulla pagina 1di 153

Corso di Fisica

Terza Liceo Scientifico


Questo libro è stato interamente scritto a Pordenone da Francesco Saitta, Docente di Matematica e Fisica,
c.d.c A027 ( già A049).

Un ringraziamento particolare va a Francesca Del Puppo, studentessa di Fisica all’università di Trieste,


per la sua accurata lettura delle note, le puntuali osservazioni ed i preziosi suggerimenti per la stesura
del testo attuale.

Testo elaborato e prodotto con Latex (http://www.latex-project.org/),


Figure prodotte con Geogebra (http://www.geogebra.org/) o Gnuplot (http://www.gnuplot.info/),

Ultimo aggiornamento Settembre 2019.

Quest’opera è distribuita con Licenza Creative Commons


Attribuzione - Condividi allo stesso modo 4.0 Internazionale.

Tu sei libero di:


• Condividere: riprodurre, distribuire, comunicare al pubblico,
esporre in pubblico, rappresentare, eseguire e recitare questo
materiale con qualsiasi mezzo e formato
• Modificare: remixare, trasformare il materiale e basarti su di
esso per le tue opere
per qualsiasi fine, anche commerciale, ai seguenti termini:
• Attribuzione: Devi attribuire adeguatamente la paternità sul
materiale, fornire un link alla licenza e indicare se sono sta-
te effettuate modifiche. Puoi realizzare questi termini in qual-
siasi maniera ragionevolmente possibile, ma non in modo tale
da suggerire che il licenziante avalli te o il modo in cui usi il
materiale.
• Stessa Licenza: Se remixi, trasformi il materiale o ti basi su di
esso, devi distribuire i tuoi contributi con la stessa licenza del
materiale originario.
Questo è un riassunto in linguaggio accessibile a tutti (e non
un sostituto) della licenza, per leggere una copia completa della
quale visita il sito web http://creativecommons.org/licenses/by-
sa/4.0/ o spedisci una lettera a Creative Commons, 171 Second
Street, Suite 300, San Francisco, California, 94105, USA.

Francesco Saitta, Pordenone Settembre 2019 2


Indice

1 Introduzione 7

2 Moti e Dinamica 9
2.1 Cinematica rettilinea 11
2.1.1 Moto rettilineo uniforme 12
2.1.2 Moto rettilineo uniformemente accelerato 13
2.1.3 Caduta dei gravi 14
2.1.4 Moto rettilineo vario 15
2.2 Cinematica bidimensionale 16
2.2.1 Moto parabolico 18
2.2.2 Moto circolare uniforme 18
2.2.3 Moto armonico 20
2.3 I principi della dinamica 21
2.3.1 Primo principio della dinamica 21
2.3.2 Secondo principio della dinamica 22
2.3.3 Terzo principio della dinamica 23
2.3.4 Sistemi di riferimento non inerziali 24
2.4 Work and Energy 25
2.4.1 Mechanical work 25
2.4.2 Work done by a constant force 25
2.4.3 Work done by a force varying on space 26
2.4.4 Work done by the gravitational force 27
2.4.5 Work done by elastic force 29
2.4.6 Conservative forces 29
2.4.7 Energy 29
2.4.8 Kinetic energy 30
2.4.9 Gravitational energy 31

Francesco Saitta, Pordenone Settembre 2019 3


2.4.10 Elastic energy 31
2.4.11 Conservation of mechanical energy 32
2.4.12 Work done by non-conservative forces 34
2.4.13 Problem Solving 34
2.4.14 Power 35
2.4.15 The Feynman point of view 36
2.5 Meccanica dei fluidi 37
2.5.1 Fluidi in moto 38
2.5.2 Equazione di continuità 38
2.5.3 Teorema di Bernoulli 39
2.5.4 Tubo di Venturi 41
2.5.5 Portanza 43
2.5.6 Aneurismi e Trombosi 43
2.6 Esercizi 46

3 Relatività Galileiana 55
3.1 Spazio e tempo nella fisica classica 56
3.1.1 Il carattere assoluto di spazio e tempo: Galilei e Newton 56
3.1.2 Il concetto di spazio secondo Berkeley e Mach 57
3.2 Legge di composizione delle posizioni e degli spostamenti 58
3.3 Legge di composizione delle velocità 59
3.4 Legge di composizione delle accelerazioni 60
3.5 Trasformazioni di Galileo 60
3.5.1 Invarianza delle lunghezze 61
3.6 Principio di relatività galileiano 61
3.7 Esercizi 64

4 Quantità di moto ed urti 69


4.1 Teorema dell’impulso 69
4.2 Teorema di conservazione della quantità di moto 70
4.3 Descartes e Leibniz: dibattito sui principi di conservazione 71
4.4 Urti 74
4.4.1 Urti completamente anelastici in una, due o tre dimensioni 75
4.4.2 Urti elastici in una dimensione 76
4.4.3 Urti elastici in due dimensioni 76

Francesco Saitta, Pordenone Settembre 2019 4


4.5 Centro di massa e moto di sistemi di particelle 77
4.6 Esercizi 81

5 Momento angolare e moto rotatorio 87


5.1 Momento d’inerzia 87
5.1.1 Rotazione di un punto materiale 88
5.1.2 Rotazione di un sistema di punti materiali 89
5.1.3 Rotazione di un corpo rigido 89
5.2 Momento angolare 90
5.3 Teorema di conservazione del momento angolare 91
5.4 Dinamica rotazionale 92
5.4.1 Lavoro dei momenti delle forze 92
5.4.2 Energia cinetica rotazionale 93
5.4.3 Moti di un corpo rigido 94
5.5 Parallelismo tra dinamica traslatoria e rotatoria 95
5.6 Equazioni cardinali della dinamica 96
5.7 Esercizi 98

6 Gravitazione universale 103


6.1 Le leggi di Keplero 103
6.1.1 La prima legge di Keplero 103
6.1.2 La seconda legge di Keplero 104
6.1.3 La terza legge di Keplero 104
6.2 La legge di gravitazione universale 104
6.3 Il concetto di campo 106
6.3.1 Linee di campo 108
6.4 Il campo gravitazionale 108
6.5 L’energia potenziale gravitazionale 109
6.6 Pianeti e satelliti 110
6.6.1 I satelliti artificiali della terra 110
6.7 Traiettorie ed energia 111
6.8 Esercizi 114

7 Termodinamica 119
7.1 Teoria cinetica dei gas perfetti 121
7.1.1 Principio di equipartizione dell’energia 123

Francesco Saitta, Pordenone Settembre 2019 5


7.2 First principle of thermodynamics 124
7.2.1 Introduction to Thermodynamics 124
7.2.2 Thermal Machines 125
7.2.3 Internal Energy 125
7.2.4 Heat capacities of gases 126
7.2.5 Mechanical work of gases 127
7.2.6 The first law of thermodynamics 132
7.3 Rendimento delle macchine termiche 133
7.3.1 Macchina di Stirling 134
7.3.2 Macchina di Carnot 134
7.3.3 Macchina di Otto (motore 4 tempi) 136
7.3.4 Macchina Diesel 137
7.3.5 Macchina Frigorifero 138
7.4 Secondo principio della termodinamica 138
7.4.1 Enunciati di Clausius, Kelvin-Planck e loro equivalenza 139
7.4.2 Enunciato di Carnot ed entropia 139
7.5 Esercizi 145

Bibliografia 153

Francesco Saitta, Pordenone Settembre 2019 6


1 Introduzione

Questo corso nasce dall’idea di consegnare agli studenti uno strumento di lavoro gratuito, snello e flessi-
bile. L’idea fondamentale è che queste note possano essere veicolo di uno studio dinamico, che vengano
sottolineate, integrate dagli appunti presi in classe o dagli approfondimenti personali, anche modificate
nel corso degli anni per renderle sempre più utili all’apprendimento della disciplina.

Le dispense sono pensate come inserite in un percorso di didattica collaborativa e multimediale:


• non vi si trovano schemi riassuntivi o formulari perché questi verranno chiesti agli studenti durante il
corso all’interno di una piattaforma multimediale o semplicemente negli appunti personali;
• possono e devono essere affiancate ad una costante ricerca di fonti, materiali audio e video, simulazioni
digitali ed attività di laboratorio vere e proprie.
Gli esercizi, destinati a crescere in numero di anno in anno, non sono suddivisi nei paragrafi del libro
perché gli studenti siano stimolati a capire l’argomento specifico cui fanno riferimento, affinché l’esercizio
non diventi atto puramente meccanico e relativo ad una certa formula, ma abbia una forte componente
di discernimento e descrizione della realtà. Anche gli esercizi verranno supportati da una serie di attività
didattiche, esempi, compiti e test, forniti nella piattaforma digitale di classe.

Talvolta le costanti da utilizzare non sono esplicitate o presenti nel testo, così da stimolare gli studenti
alla loro ricerca, con i canali preferiti.

Alcune parti del testo, così come gli ultimi esercizi di ogni capitolo, sono in inglese; questo per abi-
tuare gli studenti all’utilizzo della lingua veicolare della scienza al di fuori dell’insegnamento dedicato.
In relazione al programma della classe redatto dal consiglio di classe ad inizio anno, alcune parti di pro-
gramma (tipicamente quelle già scritte in inglese) potranno essere affrontate con alcune lezioni in lingua,
in collaborazione con il/la docente di inglese.

Francesco Saitta, Pordenone Settembre 2019 7


2 Moti e Dinamica

Ammesso - come visto ed ampiamente discusso gli scorsi due anni- che la
fisica è la scienza che descrive e predice i fenomeni naturali che avvengo-
no nell’universo, la prima cosa che possiamo fare per descrivere la realtà
che ci circonda è quella di studiare come si muovono i punti materiali e
quali siano le cause del loro moto. Gli argomenti di questo capitolo sono
già stati affrontati nel corso del primo biennio di liceo: li riprendiamo all’i-
nizio del secondo biennio per evidenziare ed approfondire alcuni aspetti
concettuali e matematici utili per il proseguo del corso, in particolare la
geometria analitica e l’algebra vettoriale.

Fu Galileo Galilei, l’ultimo dei filosofi naturali ed il primo dei fisi- Pisa, 1564 - Arcetri, 1642
ci, a studiare in modo quantitativo la cinematica: la descrizione dei moti
dal punto di vista geometrico, fatta senza prendere in considerazione le
cause dei moti stessi. Questa semplificazione ha permesso e permette
di schematizzare e descrivere anche sperimentalmente diverse situazioni
concrete: lo stesso Galilei riuscì a verificare le leggi della cinematica in
laboratorio, riuscendo a minimizzare gli effetti reali di attrito non presi in
considerazione dalla cinematica, che pur esistono. Nel corso dei suoi studi
lo scienziato pisano - come vedremo nel capitolo seguente con il principio
di relatività galileiano - affrontò anche il problema delle cause del moto,
ed in qualche modo anticipò alcune delle leggi che ora conosciamo come
i principi della dinamica, ma non le formalizzò mai in termini scientifici.

La dinamica per come è conosciuta al giorno d’oggi affonda le sue ra-


dici negli studi di Isaac Newton, fisico e filosofo britannico, presidente Woolsthorpe, 1642 - Londra,
della Royal Society, il cui lavoro spaziò dallo studio delle forze all’ottica, 1727

dalla gravitazione universale allo sviluppo della matematica infinitesima-


le. Newton studiò dal punto di vista formale e matematico il problema
delle cause del moto, riprendendo alcuni ragionamenti fatti da Galilei ed
assumendo tutta la cinematica del fisico pisano. Nella sua opera “Philo-
sophiae Naturalis Principia Mathematica” 1 , lo scienziato britannico esplicita 1
(Newton, 1687)
le basi matematiche delle sue teorie ed enuncia i famosi tre principi della
dinamica che fondano, assieme al lavoro di Galilei, tutta l’impostazione
della meccanica e della fisica dei seguenti due secoli.

Riprendiamo dunque alcuni concetti fondamentali per la descrizione di


cinematica e dinamica: Punto materiale, Sistemi di riferimento, Variabili
cinematiche, Traiettoria e Legge oraria.

Francesco Saitta, Pordenone Settembre 2019 9


Moti e Dinamica

Punto materiale Uno dei concetti fondamentali della meccanica è quello


di punto materiale. Con punto materiale si intende un corpo le cui dimen-
sioni si possono trascurare nella descrizione del suo moto. É ben chiaro che questa
possibilità dipende dalle condizioni concrete di questo o quell’altro problema. Per
esempio, i pianeti possono esser considerati punti materiali quando si studia la
loro rivoluzione intorno al sole, ma, naturalmente, non quando si considera la
loro rotazione.2 . 2
(Landau e Lifshitz, 1999a)

Sistemi di riferimento Per poter definire la posizione di un punto mate-


riale in modo quantitativo dobbiamo definire un riferimento rispetto al
quale farlo. Chiedendoci ad esempio la posizione di Luca all’interno della
classe qualcuno potrà rispondere “è un posto davanti a me” oppure “il
posto davanti alla cattedra” o ancora “nella fila centrale il terzo banco dal
fondo a sinistra”: in ogni caso la risposta prevede un riferimento al luogo
in cui ci si trova. La scelta che gli scienziati fanno per poter definire la
posizione di punti materiali è quella di definire dei sistemi di coordina-
te; il più utilizzato, che useremo anche noi durante tutto il corso di fisica
è il sistema di riferimento cartesiano: un sistema di rette orientate
perpendicolari tra loro all’interno delle quali definire la posizione di un
punto materiale come vettore con coordinate sulle rette, una se il proble-
ma è unidimensionale, due se bidimensionale, tre se tridimensionale. In
figura (2.1) vediamo un esempio di posizione rispettivamente nello spazio
, nel piano e nella retta. Figura 2.1: Posizione di un pun-
to materiale in un riferimento
cartesiano.

(a) Punto in 3D (b) Punto in 2D (c) Punto in 1D

Variabili cinematiche Le variabili cinematiche di interesse per lo studio


dei moti sono i vettori posizione ~s(t), velocità ~v(t) ed accelerazione ~a(t).
A seconda della dimensione di riferimento del problema naturalmente si
tratta di vettori in una, due o tre dimensioni.

Il vettore posizione è il vettore che identifica la posizione del punto


materiale nello spazio: ha come coordinate le coordinate del punto stesso.
La posizione di un punto materiale in SI3 si misura in metri. 3
SI è il Sistema Internazionale
delle unità di misura, nato nel
1960 durante l’undicesima Con-
Il vettore velocità viene definito a partire dalla velocità media ~vm = ferenza Generale dei Pesi e delle
∆s/∆t:
~ la velocità media tra due istanti t2 e t1 è data dallo spostamen- Misure (CGPM) a Parigi
to effettuato tra i due istanti e il tempo trascorso (la differenza tra i due

Francesco Saitta, Pordenone Settembre 2019 10


Moti e Dinamica

istanti).4 La velocità istantanea invece rappresenta la velocità in un certo 4


(!!) in fisica il concetto di spo-
istante: possiamo pensare che la velocità istantanea sia la velocità media stamento è diverso dal pensie-
ro intuitivo di spazio percorso.
tra due istanti molto vicini (l’intervallo di tempo deve essere trascurabile Lo spostamento è la differen-
rispetto al fenomeno che si sta considerando); dal punto di vista mate- za tra due vettori posizione in
due istanti diversi. Risulta così
matico si può dire che la velocità istantanea è il limite per ∆t che tende a che se il punto materiale in con-
zero della velocità media, come si vedrà nel corso di matematica parlando siderazione parte dal punto A
di analisi matematica e limiti. Per quanto ci riguarda quando scriveremo e dopo un certo percorso torna
al punto A stesso avrà percorso
d anziché ∆ intenderemo considerare un intervallo molto piccolo rispet- una certa quantità di spazio, ma
to al fenomeno considerato; per cui scrivendo v = ∆s/∆t parleremo di lo spostamento risulta essere il
vettore nullo
velocità media, scrivendo v = ds/dt parleremo di velocità istantanea, e
così con tutte le altre grandezze ottenibili come rapporto tra variazioni di
altre quantità. La velocità di un punto materiale in SI si misura in metri
al secondo.

Il vettore accelerazione viene definito a partire dall’accelerazione me-


dia ~am = ∆v/∆t:
~ l’accelerazione media tra due istanti t2 e t1 è data dalla
differenza dei vettori velocità tra i due istanti e il tempo trascorso (la diffe-
renza tra i due istanti). Analogamente al caso della velocità l’accelerazione
istantanea è il limite per ∆t che tende a zero dell’accelerazione media: è
l’accelerazione media calcolata tra due istanti di tempo molto vicini tra
loro, tali che la differenza tra i due istanti sia molto più piccola rispetto al
fenomeno considerato.

Traiettoria Si definisce traiettoria l’insieme dei punti dello spazio percor-


si dal punto materiale. Essa è quindi il percorso che il punto materiale
descrive durante il fenomeno considerato; per quanto detto in precedenza
è importante sottolineare come la lunghezza della traiettoria non corri-
sponda necessariamente con lo spostamento del punto materiale nè con
lo spazio percorso; ad esempio un punto materiale che si muova in linea
retta in avanti per 2 m, poi indietro per 3m, poi ancora in avanti per 1m
avrà percorso 6m, fatto spostamento nullo e la traiettoria un segmento di
lunghezza 3m.

Legge oraria Si definisce legge oraria la relazione tra posizione del punto
materiale e tempo trascorso. È una funzione reale di variabile reale nel
caso di un problema unidimensionale, una funzione vettoriale in due o
tre dimensioni negli altri casi, per studiare i quali basta considerare il vet-
tore ~s(t) nelle sue componenti, ognuna delle quali sarà una funzione reale
di variabile reale. Per quanto ci riguarda studieremo nel dettaglio il caso
unidimensionale, in cui la legge oraria è semplicemente data dalla fun-
zione s(t), e bidimensionale in cui si avrà ~s(t) = ( x (t), y(t)) o altrimenti
scritto ~s(t) = x (t) x̂ + y(t)ŷ) con x̂ e ŷ i versori5 degli assi cartesiani. 5
(!!) un versore è un vettore di
modulo unitario adimensionale,
che indica solamente una dire-
2.1 Cinematica rettilinea zione ed un verso. In generale
un versore si ottiene come rap-
porto tra un vettore ed il suo
I moti più semplici da descrivere sono i moti che avvengono lungo una modulo: v̂ = ~v/v. Nel caso bi-
retta, in cui le variabili cinematiche posizione, velocità ed accelerazione dimensionale cartesiano si avrà
per le due direzioni principali
sono vettori in una dimensione, descrivibili quindi come scalari: positi- x̂ = (1, 0), ŷ = (0, 1).
vi se il verso è concorde all’asse scelto, negativi se il verso è discorde.

Francesco Saitta, Pordenone Settembre 2019 11


Moti e Dinamica

Particolarmente efficace per la descrizione di questo moto è il diagram-


ma orario, un grafico cartesiano che rappresenta in ascissa il tempo ed in
ordinata la posizione del punto materiale. A differenza della traiettoria,
che resta sempre una retta o un segmento se consideriamo il moto rettili-
neo tra due particolari istanti di tempo, il diagramma orario rappresenta
la legge oraria e permette di ricavare informazioni importanti su velocità
ed accelerazione, come vedremo nel paragrafo 2.1.4.

2.1.1 Moto rettilineo uniforme


Diremo che un punto materiale si trova in uno stato di moto rettilineo uni-
forme se la sua velocità media è sempre uguale alla velocità istantanea, ed
ha modulo costante, v. In questa situazione si ricavano le seguenti leggi Per ricavare la legge oraria in
per la posizione e la velocità: questo caso basta applicare la
definizione di velocità media in
questa situazione:
s ( t ) = v ( t − t0 ) + s0 (2.1.1)
∆s
v(t) = v (2.1.2) v = vm = =
∆t
s ( t ) − s0
=
Nel rappresentare la legge oraria nel diagramma orario (t,s) riconoscia- t − t0
mo l’equazione di una retta con coefficiente angolare v ed intercetta s0 , =⇒ v(t − t0 ) = s(t) − s0
come rappresentato in figura (2.2); la rappresentazione della legge per =⇒ v(t − t0 ) + s0 = s(t)

(a) v > 0 pendenza positiva (b) v = 0 pendenza nulla (c) v < 0 pendenza negativa
Figura 2.2: Diagramma orario
del moto rettilineo uniforme.
la velocità è invece molto più semplice, essendo la velocità costante nel Le grandezze sono intese esse-
tempo, come rappresentato nella figura (2.3). È interessante notare co- re misurate in SI: spazio in me-
tri, tempo in secondi, velocità in
metri al secondo.

Figura 2.3: Rappresentazione


cartesiana della legge della ve-
locità nel moto rettilineo uni-
forme. Le grandezze sono in-
tese essere misurate in SI: spa-
zio in metri, tempo in secondi,
velocità in metri al secondo.

me la legge oraria si può ottenere graficamente a partire dalla legge della

Francesco Saitta, Pordenone Settembre 2019 12


Moti e Dinamica

velocità nel seguente modo: lo spostamento effettuato dal punto materia-


le s(t) − s0 è l’area sottesa dalla curva v(t) nel grafico di velocità, come
mostrato dalla figura (2.4). Questa regola - che verrà dimostrata dall’ana-
lisi matematica e lo studio delle derivate - si può assumere come regola
generale della cinematica.

2.1.2 Moto rettilineo uniformemente accelerato

Diremo che un punto materiale si trova in uno stato di moto rettilineo


uniformemente accelerato se la sua accelerazione media è sempre uguale
alla sua accelerazione istantanea, ed ha modulo costante, a. In questa si-
tuazione si ricavano le seguenti leggi per la posizione e la velocità: Figura 2.4: Relazione tra spazio
percorso e grafico della velocità.

1
s(t) = a ( t − t0 )2 + v0 ( t − t0 ) + s0 (2.1.3)
2
v ( t ) = a ( t − t0 ) + v0 (2.1.4)

L’equazione della velocità è ricavabile in modo analogo alla legge ora-


ria del moto rettilineo uniforme. Per quanto riguarda la legge oraria in-
vece dobbiamo ricorrere alla regola grafica vista nel paragrafo preceden-
te, come rappresentato in figura (2.5): lo spostamento effettuato s(t) − s0
è dato quindi dall’area del trapezio di altezza (t − t0 ), base maggiore
v(t) = a(t − t0 ) + v0 e base minore v0 . Il grafico della legge oraria rap-

(a) Costruzione della legge oraria (b) a > 0 concavità verso l’alto (c) a < 0 concavità verso il basso
Figura 2.5: Moto rettilineo
uniformemente accelerato.
presenta una parabola, come già parzialmente visto in matematica nella
risoluzione di disequazioni di secondo grado e come si vedrà nel corso di
matematica studiando geometria analitica. Il segno dell’accelerazione de-
finisce la concavità della parabola: ad accelerazione positiva corrisponde
concavità verso l’alto e ad accelerazione negativa concavità verso il basso;
ad accelerazione sempre maggiore corrisponde una parabola sempre più
schiacciata verso l’asse delle ordinate, ad accelerazione minore una para-
bola sempre più schiacciata verso l’asse delle ascisse. Gli altri parametri
contribuiscono alla forma della parabola nei modi che verranno studiati
nel corso di matematica e che possono essere testati con semplici prove
fatte usando carta millimetrata e penna o software di geometria dinamica
(geogebra ad esempio).

Francesco Saitta, Pordenone Settembre 2019 13


Moti e Dinamica

2.1.3 Caduta dei gravi

Il tipico moto rettilineo uniformemente accelerato che possiamo osservare


in natura è la caduta di un oggetto da una certa altezza. Ogni oggetto la-
sciato cadere da una certa altezza, nell’ipotesi in cui si possano trascurare
tutti gli effetti di attrito dell’aria sull’oggetto stesso, si muove di moto ret-
tilineo uniformemente accelerato con un’accelerazione - che chiameremo
accelerazione di gravità - approssimativamente costante in prossimità del-
la superficie terrestre di modulo pari a g = 9, 81m/s2 , direzione lungo la
congiungente tra la posizione iniziale dell’oggetto ed il centro della terra
e verso che punta al centro della terra.

L’analisi di Galileo Galilei sulla caduta dei gravi fu sicuramente uno


dei primissimi esempi di applicazione del metodo scientifico, con cui il
fisico pisano segnò il passaggio tra la filosofia naturale e la fisica. La leg-
genda dice che Galilei per verificare l’ipotesi secondo cui la velocità di un
grave in caduta libera è direttamente proporzionale al tempo di caduta e
non dipende dalla massa o dalla forma del grave stesso, come invece do-
veva essere secondo la teoria Aristotelica del moto, avrebbe lasciato cadere
una serie di oggetti dalla torre di Pisa. Non siamo certi che questo esperi-
mento sia stato realmente effettuato, ma di certo Galilei ha lasciato molti
scritti all’interno dei quali giustifica in modo logico le sue affermazioni 6 6
(Galilei, 1592, 1638, 1632)
; in particolare nell’ultimo di questi scritti alla giornata terza lo scienziato
pisano descrisse l’esperimento con cui verificò la sua ipotesi, utilizzando
un piano inclinato ed un orologio ad acqua: “In un regolo, o vogliàn dir
corrente, di legno, lungo circa 12 braccia, e largo per un verso mezo bracio e per
l’altro 3 dita, si era in questa minor larghezza incavato un canaletto, poco più
largo d’un dito; tiratolo drittissimo, e, per averlo ben pulito e liscio, incollatovi
dentro una carta pecora zannata e lustrata al possibile, si faceva in esso scende-
re una palla di bronzo durissimo, ben rotondata e pulita; costituito che si era il
detto regolo pendente, elevando sopra il piano orizontale una delle sue estremità
un braccio o due ad arbitrio, si lasciava (come dico) scendere per il detto canale la
palla, notando, nel modo che appresso dirò, il tempo che consumava nello scorrerlo
tutto, replicando il medesimo atto molte volte per assicurarsi bene della quanti-
tà del tempo, nel quale non si trovava mai differenza né anco della decima parte
d’una battuta di polso. Fatta e stabilita precisamente tale operazione, facemmo
scender la medesima palla solamente per la quarta parte della lunghezza di esso
canale; e misurato il tempo della sua scesa, si trovava sempre puntualissimamente
esser la metà dell’altro: e facendo poi l’esperienze di altre parti, esaminando ora
il tempo di tutta la lunghezza col tempo della metà, o con quello delli duo ter-
zi o de i 3/4, o in conclusione con qualunque altra divisione, per esperienze ben
cento volte replicate sempre s’incontrava, gli spazii passati esser tra di loro come
i quadrati e i tempi, e questo in tutte le inclinazioni del piano, cioè del canale
nel quale si faceva scender la palla; dove osservammo ancora, i tempi delle scese
per diverse inclinazioni mantener esquisitamente tra di loro quella proporzione
che più a basso troveremo essergli assegnata e dimostrata dall’Autore. Quanto
poi alla misura del tempo, si teneva una gran secchia piena d’acqua, attaccata in
alto, la quale per un sottil cannellino, saldatogli nel fondo, versava un sottil filo
d’acqua, che s’andava ricevendo con un piccol bicchiero per tutto ’l tempo che la

Francesco Saitta, Pordenone Settembre 2019 14


Moti e Dinamica

palla scendeva nel canale e nelle sue parti: le particelle poi dell’acqua, in tal guisa
raccolte, s’andavano di volta in volta con esattissima bilancia pesando, dandoci
le differenze e proporzioni de i pesi loro le differenze e proporzioni de i tempi; e
questo con tal giustezza, che, come ho detto, tali operazioni, molte e molte volte
replicate, già mai non differivano d’un notabil momento.”

2.1.4 Moto rettilineo vario


Diremo che un punto materiale si trova in uno stato di moto rettilineo
vario se caratterizzato da variabili cinematiche s(t), v(t), a(t) che non
necessariamente restano costanti nel tempo. In generale recuperando le
considerazioni fatte nei paragrafi precedenti potremo dire che la variazio-
ne di velocità v(t) − v0 è ottenibile come l’area sottostante la curva a(t)
nel piano cartesiano (t, a), la variazione di posizione s(t) − s0 come l’area
sottostante la curva v(t) nel piano cartesiano (t, v).

Diagramma orario In questo paragrafo affronteremo il problema opposto:


conoscendo il diagramma orario capiremo come ottenere informazioni su
velocità ed accelerazione del punto materiale. Consideriamo ad esempio
un punto materiale il cui moto sia descritto dalle figure (2.6,2.7, 2.8).
La prima figura mostra il diagramma orario che rappresenta il moto
del punto materiale: possiamo certamente dire che per circa un secondo il Figura 2.6: diagramma orario
punto materiale si muove in avanti (quindi con velocità positiva), poi per
circa due secondi, dal secondo uno al secondo tre invece si muove all’in-
dietro (con velocità negativa quindi), infine riparte in avanti passando per
l’origine poco dopo il quarto secondo.

Velocità Applicando la definizione di velocità media tra due punti possia-


mo vedere dalla seconda figura come il modulo della velocità possa essere
ricavato come il rapporto tra la variazione di posizione ∆s = s(t) − s0 ed
il tempo trascorso ∆t = t − t0 : la velocità media tra i punti s(t A ) ed s(t B )
è dunque la pendenza della retta secante il diagramma orario nei punti A
e B. La velocità istantanea diventa dunque la pendenza della retta secante
il diagramma orario nel caso in cui i due punti A e B siano molto vicini Figura 2.7: velocità
tra loro, al punto di non essere distinguibili l’uno dall’altro:
è dunque la pendenza della retta tangente alla curva nel punto in que-
stione! Diventa così evidente che tutti i punti in cui la tangente al dia-
gramma orario è parallela all’asse delle ascisse sono punti in cui la veloci-
tà è nulla: osservando attentamente il grafico potremo dedurre che questi
punti sono tutti i massimi o i minimi della curva: definiamo un punto
P = ( x P , y P ) della curva come massimo se tutti i punti più vicini ad esso,
sia a destra che a sinistra, hanno ordinata minore di y P , minimo invece se
i punti più vicini hanno ordinata maggiore di y P .

Accelerazione L’accelerazione è una variazione di velocità: partendo da


quest’idea intuitiva di accelerazione possiamo capire come l’accelerazione
del punto materiale sia legata alla concavità della curva, come mostrato Figura 2.8: accelerazione

nella terza figura del moto vario. Se la velocità diminuisce la curva ten-
derà a ”piegarsi” verso il basso dando origine ad una concavità verso il

Francesco Saitta, Pordenone Settembre 2019 15


Moti e Dinamica

basso, se invece la velocità aumenta accadrà il contrario. Come per il caso


della velocità, queste affermazioni al momento piuttosto qualitative tro-
veranno una dimostrazione matematica formale verrà fatta nel contesto
dell’analisi matematica e dello studio delle derivate.

2.2 Cinematica bidimensionale

La cinematica bidimensionale si fonda sul principio di indipendenza del-


le azioni simultanee che afferma come se il moto di un punto materiale
avviene in più dimensioni, esso può essere sempre scomposto in com-
ponenti indipendenti l’una dall’altra. Esso può essere quindi studiato
attraverso le sue componenti , che noi assumeremo sempre essere coordi-
nate cartesiane; il moto risultante sarà il moto del punto materiale la cui
posizione è descritta dal vettore

~s(t) = x (t) x̂ + y(t)ŷ

La rappresentazione grafica della legge oraria diventa dunque una rap-


presentazione in tre dimensioni, o una rappresentazione delle leggi orarie
delle singole componenti: per questo motivo utilizzeremo spesso la rap-
presentazione della traiettoria per descrivere graficamente il moto, che nel
caso bidimensionale è più significativa rispetto al caso unidimensionale.
É importante sottolineare come in un moto in più dimensioni il vettore
velocità sia sempre tangente alla traiettoria percorsa dal punto materiale:
non facciamo qui una dimostrazione formale di questo fatto, ma possiamo
rendercene conto in modo piuttosto intuitivo pensando alla definizione di
velocità; il vettore velocità è un vettore la cui direzione è data dalla dif-
ferenza tra due vettori posizione, per cui è un vettore che ha origine in
un punto della traiettoria e fine in un altro punto della traiettoria stessa:
evidentemente quando questi due punti tendono a diventare lo stesso la
direzione del vettore velocità diventa il tangente alla curva che definisce
la traiettoria. La figura (2.9) mostra proprio questo fatto.

Figura 2.9: Velocità in un moto


bidimensionale.

(a) (b)

Per quanto riguarda l’accelerazione in un moto bidimensionale, sia-

Francesco Saitta, Pordenone Settembre 2019 16


Moti e Dinamica

mo soliti pensare che essa possa essere generata solo da una variazione
del modulo del vettore velocità: è bene sottolineare come si possa avere
un accelerazione anche solamente con la variazione della direzione o del
verso del vettore velocità. Sappiamo bene infatti che la differenza di due
vettori con lo stesso modulo, ma direzione diversa, non necessariamen-
te risulta essere il vettore nullo. Ecco allora che l’accelerazione in due
dimensioni può essere causata da una variazione di modulo del vettore
velocità, da una variazione della sua direzione, o da una combinazione
delle due variazioni. In generale chiameremo accelerazione tangenziale
l’accelerazione dovuta ad una variazione del modulo del vettore velocità
del punto materiale, accelerazione centripeta invece l’accelerazione dovu-
ta alla variazione della direzione del vettore velocità. Diremo quindi che
sempre l’accelerazione di un punto materiale può essere scomposta nel
seguente modo:
~a = ~at +~ac ,

dove ~at è l’accelerazione tangenziale e ~ac è l’accelerazione centripeta. La


componente tangenziale dell’accelerazione è sempre diretta parallelamen-
te al vettore velocità, e dunque tangente alla traiettoria, mentre la compo-
nente centripeta è sempre perpendicolare alla velocità stessa. Per darci
ragione della direzione delle due componenti dell’accelerazione facciamo
riferimento alla figura (2.10). Come si può vedere in figura se l’accele-
razione è dovuta ad una variazione unicamente di modulo della velocità
ha la stessa direzione dei due vettori velocità ~a = ~v1 − ~v2 , se invece l’ac-
celerazione è dovuta solo ad una variazione di direzione della velocità
si ha che l’angolo tra velocità ~v2 ed accelerazione ~a è dato da γ = π + α
2 :
è evidente che se consideriamo la velocità istantanea l’angolo α diventa
molto piccolo e man mano che α diventa piccolo γ si avvicina sempre più
a π/2. L’accelerazione centripeta è dunque perpendicolare alla velocità
istantanea.

Figura 2.10: Direzione del-


le componenti dell’accelerazio-
ne in un moto bidimensionale.

(a) Accelerazione tangenziale (b) Accelerazione centripeta

Francesco Saitta, Pordenone Settembre 2019 17


Moti e Dinamica

2.2.1 Moto parabolico


Il moto parabolico è un moto in cui una componente del vettore posizione
si muove di moto rettilineo uniforme, l’altra di moto rettilineo uniforme-
mente accelerato. Ne conseguono le seguenti equazioni per la descrizione
del moto:
x (t) = v0x t + x0 (2.2.1)
1
y(t) = at2 + v0y t + y0 (2.2.2)
2
v x (t) = v0x (2.2.3)
vy (t) = at + v0y (2.2.4)

Il moto si chiama moto parabolico perchè la traiettoria descritta dal punto


materiale dal punto di vista della geometria analitica è un parabola, come
si vedrà nel dettaglio nel corso di matematica. Tutte le considerazioni fatte
studiando i due moti che compongono il moto parabolico possono essere
riutilizzate nella descrizione di un moto parabolico. Nella nostra espe-
rienza quotidiana di fenomeni naturali il moto parabolico è uno dei moti
che possiamo osservare con più facilità: risponde infatti alla situazione di
caduta di un oggetto con una componente orizzontale di velocità iniziale.

2.2.2 Moto circolare uniforme


Definiamo moto circolare un moto la cui traiettoria è una circonferenza. Il
vettore velocità dunque, essendo sempre tangente alla traiettoria cambia
continuamente; per questo il concetto di “uniforme” in questo caso non
può significare, come nel caso unidimensionale, che la velocità è costan-
te. Moto circolare uniforme si definisce un moto la cui traiettoria è una
circonferenza ed il modulo della velocità è costante. Il moto circolare uni-
forme è dunque caratterizzato dalle tre grandezze: velocità tangenziale
(~v), periodo (T, il tempo che il punto materiale impiega a percorrere un
giro di circonferenza) e il raggio della circonferenza (R). Il periodo può
essere sostituito dalla frequenza ( f o ν), che rappresenta il numero di giri
che il punto materiale compie in un secondo, e si misura in Hertz (Hz).
Queste grandezze si relazionano, per definizione di velocità, nel seguente
modo:
2πR
v= = 2πR f
T
Abbiamo anche dimostrato, utilizzando le similitudini tra triangolo, come
illustrato in figura (2.11) che l’accelerazione, che per quanto detto prece-
dentemente deve essere centripeta e dunque diretta verso il centro della
circonferenza per le proprietà della geometria euclidea, è ottenibile dalla
seguente formula:
v2
ac =
R

Le grandezze angolari La descrizione del paragrafo precedente del moto


circolare uniforme è una descrizione che si basa sulle variabili cinematiche

Francesco Saitta, Pordenone Settembre 2019 18


Moti e Dinamica

Figura 2.11: Dimostrazione


della formula ac = v2 /R

classiche descritte anche all’inizio di questo capitolo; per quanto riguar-


da i moti circolari hanno grande rilevanza anche le variabili cinematiche
angolari: l’angolo spazzato dal vettore posizione, il vettore velocità an-
golare ed il vettore accelerazione angolare. L’angolo spazzato dal vettore
posizione ~r (t) tra due istanti t2 e t1 si definisce come l’angolo θ compreso
tra i due vettori r2 ed r1 . La definizione degli angoli in radianti, come mo-
strato in figura (2.12), ci permette di passare dalla descrizione tra variabili
cinematiche classiche e variabili cinematiche angolari: lo spazio percorso
s è dato da Rθ, se l’angolo è misurato in radianti. Definiamo quindi la

Figura 2.12: Gli angoli in


radianti

velocità angolare media ω~m il vettore che ha come modulo la variazione


di angolo spazzato dal vettore posizione del punto materiale e l’intervallo

Francesco Saitta, Pordenone Settembre 2019 19


Moti e Dinamica

di tempo stesso:
θ ( t2 ) − θ ( t1 )
ωm =
t2 − t1
la velocità angolare istantanea sarà in analogia con le quantità cinematiche
istantanee tradizionali la velocità angolare media calcolata in un interval-
lo di tempo molto piccolo rispetto alla durata del fenomeno che stiamo
considerando. dal punto di vista della velocità istantanea quindi si ha

∆s R∆θ
v= = = Rω
∆t ∆t

Il passaggio è giustificato solo nel caso di velocità istantanea, in cui l’inter-


vallo di tempo è molto piccolo rispetto alla durata del fenomeno e quindi
l’arco di circonferenza ∆s può essere approssimato con la corda ∆r. La
direzione del vettore velocità angolare ci permette di collegare il vettore al
verso di rotazione del punto materiale: la direzione di ω ~ è perpendicolare
al piano formato da ~r1 ed ~r2 ed il suo verso definito dalla regola destrorsa,
come mostrato in figura (2.13).
Unendo le caratteristiche del vettore ω appena descritte possiamo con-
cludere che
~ ×~r
~v = ω
L’accelerazione centripeta può essere espressa anch’essa come funzione
Figura 2.13: La velocità angola-
della velocità angolare secondo l’espressione
re

~ × ~v
~ac = ω
ac = ω 2 r

Per moti in cui la velocità angolare non è costante si può definire l’accele-
razione angolare α:
∆ω
~
~α =
∆t
Nel caso del moto circolare uniforme avremo:

~α = ~0 (2.2.5)
~ = ~k
ω (2.2.6)
v
|~
ω| = (2.2.7)
r
θ = ωt + θ0 , (2.2.8)

di cui notiamo le somiglianze formali con le equazioni del moto rettilineo


uniforme per le variabili cinematiche ~s, ~v ed ~a.

2.2.3 Moto armonico


Definiamo moto armonico un moto per cui vale la relazione

~a(t) = −ω 2~s(t)

Una relazione di questo genere tra accelerazione e vettore posizione de-


scrive un’oscillazione periodica di periodo T = 2π/ω del punto materiale

Francesco Saitta, Pordenone Settembre 2019 20


Moti e Dinamica

attorno ad un centro (~s = ~0). Abbiamo già discusso nel corso del primo
biennio come la proiezione del moto circolare uniforme lungo una dire-
zione particolare ed il moto del pendolo semplice siano esempi di moto
armonico. I moti armonici ricoprono un ruolo di prima importanza in fi-
sica, perchè sono alla base della descrizione di moltissimi fenomeni fisici,
dal pendolo semplice alle onde meccaniche, dai circuiti elettrici alla strut-
tura microscopica della materia, come avremo modo di accennare durante
il corso di fisica del secondo biennio e del quinto anno di liceo.

2.3 I principi della dinamica

I principi della dinamica sono tre leggi che Newton introdusse con l’in-
tento di descrivere come le forze siano collegate ai moti dei corpi (senza
considerare i moti di rotazione dei corpi attorno a loro stessi). Sono leg-
gi derivate dall’esperienza e l’osservazione, non sono teoremi derivabili
da definizioni date precedentemente, per questo le chiamiamo principi.
Essi sintetizzano il lavoro fatto dai pensatori del 1600, Galilei e Newton
sopratutto, e pongono le basi per la costruzione della fisica moderna, la
meccanica in particolare.

2.3.1 Primo principio della dinamica

“Lex I: Corpus omne perseverare in statu suo quiescendi vel movendi


uniformiter in directum, nisi quatenus a viribus impressis cogitur sta-
tum illum mutare” (Newton, 1687)
Il primo principio della dinamica dice che ogni corpo non soggetto a
forze persevera nel suo stato di quiete o di moto rettilineo uniforme. In
altre parole, se la somma di tutte le forze agenti su un corpo è nulla esso
rimarrà nel suo stato di quiete o di moto rettilineo uniforme, e viceversa

n
~R(= ∑ ~Fi ) = ~0 ⇐⇒ ~v = ~k (2.3.1)
i =1

Il primo principio della dinamica, che fu proposto in termini non forma-


li già da Galilei, è di fondamentale importanza per la fisica moderna in
quanto introduce il concetto di equilibrio di un punto materiale; è chia-
mato anche principio d’inerzia: si definisce inerzia infatti la proprietà dei
corpi che si oppone ad un cambiamento dello stato di moto del corpo stes-
so. Nella discussione fatta da Newton è implicito il concetto di sistema
di riferimento inerziale che nell’idea dello scienziato britannico si defini-
sce come il sistema di riferimento all’interno del quale valgono i principi
della dinamica. Questi sistemi sono tutti i sistemi in stato di quiete o di
moto rettilineo uniforme l’uno rispetto all’altro. In questo senso ancora
una volta la fisica che propone Newton si pone in continuità con la fisica
di Galileo: circa cinquant’anni prima di Newton Galileo propose il suo
principio di relatività (che noi studieremo nel prossimo capitolo), il quale
diceva che tutte le leggi della fisica devono avere la stessa forma in sistemi
di riferimento inerziali tra loro.

Francesco Saitta, Pordenone Settembre 2019 21


Moti e Dinamica

2.3.1.1 Esempi ed applicazioni


• Se siamo seduti in un’automobile che inizia a frenare, ci sentiamo spin-
gere in avanti; in realtà è la nostra inerzia che si manifesta: tendiamo a
continuare a muoverci di moto rettilineo uniforme!

• Se pensiamo a dover mantenere la velocità costante andando in bici-


cletta capiamo bene di dover imprimere una forza sui pedali. Questa
forza, secondo il primo principio della dinamica, deve essere esatta-
mente uguale ed opposta alla forza d’attrito che si sviluppa tra le ruote
ed il terreno e tra la bicicletta, il ciclista e l’aria.

• Un esempio particolarmente importante di sistema di riferimento che


normalmente approssimiamo come sistema inerziale è dato dalla terra
stessa: perché valgano i principi della dinamica tutti gli esperimenti
che facciamo ogni giorno nei nostri laboratori devono essere fatti in
sistemi di riferimento inerziali! Ma come possiamo giustificare questa
approssimazione dato il fatto che conosciamo il moto di rivoluzione
della terra attorno al sole e dunque sappiamo che la terra ha una certa
accelerazione centripeta? Calcolando l’accelerazione centripeta della
terra nella sua rivoluzione intorno al sole e nella rotazione intorno a
se stessa possiamo notare come queste accelerazioni abbiamo intensità
molto piccole, per lo meno rispetto ai tempi tipici della maggior parte
degli esperimenti che possiamo immaginare di fare in un laboratorio.

2.3.2 Secondo principio della dinamica


“Lex II: Mutationem motus proportionalem esse vi motrici impressae, et
fieri secundum lineam rectam qua vis illa imprimitur.” (Newton, 1687)
Il secondo principio della dinamica dice che l’accelerazione di un punto
materiale è direttamente proporzionale alla somma delle forze su di esso
impresse. La relazione matematica tra le due grandezze è dunque

n
~R(= ∑ ~Fi ) = m~a (2.3.2)
i =1

Il secondo principio della dinamica mette in evidenza la relazione tra acce-


lerazione e forza: la forza applicata ad un punto materiale è la causa del-
l’accelerazione del punto materiale stesso. Dal punto di vista concettuale
questa legge è importante per due motivi:

• è la prima legge fisica che parla di come una causa si relaziona con
il suo effetto, caratteristica che diventa distintiva della fisica moderna
dandole la capacità di predire fenomeni e non solo descriverli,

• definisce la massa di un punto materiale come la sua inerzia, la carat-


teristica che si oppone al cambiamento dello stato di moto.

2.3.2.1 Esempi ed applicazioni


• Il peso è una delle prime forze cui applichiamo la seconda legge del-
la dinamica per concludere che ~g, il vettore che siamo stati abituati a

Francesco Saitta, Pordenone Settembre 2019 22


Moti e Dinamica

moltiplicare alla massa per ottenere la forza peso è certamente un’acce-


lerazione, giustificando formalmente il suo utilizzo nei moti di caduta
libera e parabolici.

• I produttori di automobili spesso fanno pubblicità mostrando i tempi in


cui le autovetture raggiungono velocità elevate: ad esempio la Ferrari
2011 FF “può andare da 0 a 100 km/h in 3,7 secondi”. Questa frase si
riferisce all’accelerazione, la compagnia ci sta semplicemente dicendo
−0
che l’accelerazione che può raggiungere la macchina è a = 27,8 3,7 = 7, 5
m/s2 . Da questa informazione, sapendo che un’automobile del genere
ha una massa m = 1800 kg possiamo cercare di capire quanta forza
deve imprimere il motore V12 della ferrari sulla macchina stessa per
fornire quest’accelerazione (senza considerare gli attriti...) F = ma =
13524 N, poco meno della forza necessaria ad alzarla completamente
(P = mg = 17640 N)!!

• Ricordiamo come le forze siano grandezze vettoriali e come quindi


spesso siamo obbligati ad operare in due o tre dimensioni, scompo-
nendo le forze in componenti perpendicolari tra loro. Basta risolvere
il seguente problema, che fa riferimento alla figura (2.14) per render-
sene conto: Una valigia ha una massa m = 10kg e viene trainata dal suo
proprietario con una forza F = 50 N con una direzione che forma un angolo
α = 60 rispetto all’orizzontale; supponendo un coefficiente d’attrito k = 0, 2
tra la valigia e il pavimento, si calcoli l’accelerazione della valigia.

2.3.3 Terzo principio della dinamica


Figura 2.14: Esempio 3
“Lex III: Actioni contrariam semper et æqualem esse reactionem: sive
corporum duorum actiones in se mutuo semper esse æquales et in par-
tes contrarias dirigi.” (Newton, 1687)
Il terzo principio della dinamica dice che se un punto materiale A im-
prime una forza ~F1 su un punto materiale B, allora il punto materiale B
imprimerà una forza ~F2 uguale e contraria ad ~F1 sul punto materiale A,
vale cioè la relazione
~F1 = −~F2 (2.3.3)

Un elemento particolarmente importante da evidenziare parlando del ter-


zo principio della dinamica è dato dal fatto che le due forze ~F1 ed ~F2
devono agire lungo la stessa direzione ma non sono applicate allo stesso
corpo, per cui non si annullano ma generano accelerazione sui due punti
materiali coinvolti (~a1 ed ~a2 ). Il terzo principio della dinamica introduce
per la prima volta nella storia della scienza il concetto di forze a distanza:
esso si applica anche se i due punti materiali non sono in contatto l’u-
no con l’altro. L’intuizione degli esseri umani suggerisce l’esistenza delle
forze come forze di contatto, in natura invece troviamo molte forze che
agiscono a distanza e che studieremo nel dettaglio nel corso degli anni di
liceo come ad esempio la forza gravitazionale e quella elettrostatica.

Francesco Saitta, Pordenone Settembre 2019 23


Moti e Dinamica

2.3.3.1 Esempi ed applicazioni


• Per poter correre in avanti dobbiamo spingere con il piede all’indietro!!!
Come è possibile? Per il terzo principio della dinamica se il piede
spinge all’indietro il pavimento (o il blocco di partenza per le corse
di atletica leggera), il pavimento risponderà con una forza uguale e
contraria sul piede, effetto della quale sarà l’accelerazione in avanti del
corridore.

• Immaginiamo di pesarci sulla bilancia premendo con una scopa sul


soffitto. La bilancia peserà una forza maggiore della nostra forza peso,
perché la stessa forza con cui premiamo sul soffitto verso l’alto viene
impressa dal soffitto su di noi verso il basso, andando a sommarsi al
nostro peso nella misurazione della bilancia.

• Se immaginiamo di essere sui pattini di fronte ad un muro e spinger-


ci in avanti con le mani sul muro siamo sicuri di iniziare a muoverci
all’indietro. Anche questa situazione può sembrare assurda, ma non
è nient’altro che nuovamente l’applicazione del terzo principio della
dinamica.

2.3.4 Sistemi di riferimento non inerziali


Sistemi non inerziali si definiscono come sistemi accelerati gli uni rispetto
agli altri. Nel prossimo capitolo entreremo nel merito dei ragionamenti
che vennero fatti dagli scienziati dei secoli scorsi per definire un sistema
inerziale o non inerziale in senso assoluto e non in relazione ad un altro
sistema, mentre in questo paragrafo ci limitiamo a far notare come i tre
principi della dinamica valgano solo all’interno di sistemi inerziali. Pren-
dendo in considerazione sistemi non inerziali infatti sembrano accadere
strani fenomeni dal punto di vista della meccanica newtoniana, in par-
ticolare sembrano esistere delle accelerazioni senza nessuna forza che le
causi. Pensiamo ad esempio alle seguenti situazioni

• l’accelerazione che percepiamo se l’automobile in cui ci troviamo sterza


di colpo, frena o accelera

• l’accelerazione che percepiamo stando su una giostra rotante (come il


tagadà del luna park)

• l’accelerazione che percepiamo quando l’ascensore parte verso l’alto o


verso il basso

In ognuno di questi fenomeni percepiamo delle accelerazioni, e le misu-


riamo fisicamente, senza che nessuna forza agisca su di noi direttamente.
Sembra quindi non valere il secondo principio della dinamica! Riflettendo
su questi fenomeni, come descriveremo nel dettagli nel capitolo ??, possia-
mo capire che queste accelerazioni sono in realtà l’effetto della descrizione
del fenomeno da un sistema di riferimento in accelerazione, causate so-
stanzialmente dalla nostra inerzia. Per descrivere questi effetti restando
nel formalismo newtoniano la comunità scientifica ha scelto di introdurre
delle forze cosiddette apparenti, che sono la causa dell’accelerazione che

Francesco Saitta, Pordenone Settembre 2019 24


Moti e Dinamica

percepiamo in sistemi di riferimento accelerati. Esempi di queste forze,


già descritte nel biennio, sono la forza centrifuga o la forza di Coriolis.

2.4 Work and Energy

2.4.1 Mechanical work


The term ”work” that we use in our everyday language has a meaning
that we all know and understand; in science instead this is a specific word
related to the action of a force on a body. We know that the action of
a force on a body is the cause for the accelleration of the body itself: it
means that the body changes its state of motion under the effect of forces,
it moves in a particular way, with an acceleration ~a = ~F/m (with ~a the
acceleration of the body, ~F the sum of all the forces acting on the body,
m the mass of the body). The work done by the force ~F on the body is
related to the connection between the force and the space travelled by the
body as a consequence of the action of ~F.

2.4.2 Work done by a constant force


In the simplest case in which the force is constant work is defined by the
relation:
L = ~F ·~s, (2.4.1)

where ~F is the constant force, ~s is the displacement of the body and the
operation · is the scalar product between the two vectors, it means that
the equation (2.4.1) can be written as

L = Fs cos α

where α is the angle between the two vectors ~F and ~s. At this point we can
start asking ourselves ”what is the physical meaning of this quantity?”. To
answer this question let’s have a look to the picture (2.15). In the picture
there are four different situations:

1. in (A) we can see a body lying on a surface. There are two forces acting
on this body, its weight ~ ~
P, and the normal reaction of the surface, N:
since the weight is balanced by the normal reaction, and the body is
initially at rest, it is not cause of any motion. As a consequence, ~s = 0
and the work done by the weight equals zero.

2. in (B) we can see a body that is moving up (by means of an external


force). Since the weight is acting to the bottom and the displacement is
towards the top, α = π, cos α = −1, so the work done by the weight
it will be negative: the weight is acting against the displacement of the
body.

3. in (C) we can see a body that is moving down (by means of its weight).
Since the weight is acting to the bottom, as the displacement, α = 0,
cos α = 1, so the work done by the weight it will be positive: the weight
is acting for the displacement of the body.

Francesco Saitta, Pordenone Settembre 2019 25


Moti e Dinamica

Figura 2.15: work done by a


constant force

4. in (D) we can see a body that is moving horizzontally (by means of


an external force). Since the weight is acting to the bottom, and the
displacement is horizontal α = π/2, cos α = 0, so the work done
by the weight it will be zero: the weight has nothing to do with the
displacement of the body.

In this example we can understand how mechanical work is a physical


quantity that measures the relation between a force acting on a body and
the displacement of the body itself. We will use L (the italian way) or
W (the english way) to indentify mechanical work. Mechanical work is
measured in Joule (J): one Joule is the work done by a force with strenght
of 1 N causing a displacement of 1 m along the direction of the force:
1J = 1N · 1m.

2.4.3 Work done by a force varying on space


Let’s now investigate how to calculate the work done by a force varying
on space: we assume a force ~F (s) acting on a body, with a constant angle
α between the force and the displacement. In such a case we cannot apply
the previous formula, we do not know which value of F has to be put
into Fscos(α), since it is varying. The trick to solve the problem is to
consider many tiny works and then sum all of them. Indeed, if we divide
the displacement in many tiny pieces, d~s (where d, instead of ∆ mean
that our segments are negligible with respect to the total lenght of the
displacement), in each of them we can consider the force constant, and use
~F · d~s to find a work dL. Then we have to ”sum” al these tiny works. This
is a well known operation in math, called integral, that we are not going
to study in mathematical detail at this point. This mathematical concept
is important even if we will not see tecnical details of the calculation,
to understand a geometrical way to calculate the work done by a force
varying on space: the area under the function Fcos(α) (as a function of

Francesco Saitta, Pordenone Settembre 2019 26


Moti e Dinamica

Figura 2.16: work done by a


force varying on space

the displacement) in a s-F cartesian graph. Let see figure (2.16): this is
the geometrical representation of the work done by the force ~F, causing
the displacement ~s (from 1 to 7 units) (we assume the angle α between
~F and ~s as a constant). In the picture is clear that if we consider a tiny
ds the force can be considered constant (with strenght F ∗ cos(α) in our
example) and the area between the function and the s-axis is the area
of the rectangle with basis ds and height F ∗ cos(α): the work done by the
”constant” force F ∗ cos(α) along the displacement ds! Then, the work done
along the displacement s, it will be the area between the function Fcos(α)
and the s-axis. At this point we know what is mechanical work and we
know how to calculate it: in next paragraphs we will calculate the work
done by two well known forces: the gravitational force and the elastic
force.

2.4.4 Work done by the gravitational force


The first force we analize is the gravitational force acting on a point par-
ticle with mass m: ~F = m~g. Gravitational force is a constant force, always
acting on the vertical line connecting the point particle to the ground.
Figure (2.17) describes four different situations:

• in (A) we can see that if the point particle moves horizzontally by means
of an external force the work done by the gravitational forces equals
zero: the gravitational force has nothing to do with that displacement.

• in (B) we can see that if the point particle is moving up vertically by


means of an external force, the work done by the gravitational force is
negative (the work is done ”against” the gravitational force) and equals
the quantity −mgh, where h is the difference in height between the
ground and the final position of the point particle.

Francesco Saitta, Pordenone Settembre 2019 27


Moti e Dinamica

• in (C) we can see that if the point particle is moving down vertically by
means of the gravitational force, the work done by the gravitational for-
ce is positive(the work is done by the gravitational force) and equals the
quantity mgh, where h is the difference in height between the ground
and the initial position of the point particle.

• in (D) we can see that even if the displacement is not along the vertical
line connecting the initial point and the ground the work done by the
gravitational force is again mgh! This is because of the scalar product
that defines mechanical work.

We can conclude that it doesn’t matter what is the trajectory done by the
point particle, the work done by the gravitational force does not depend
on it, but only on the initial and final heights and the difference between
them: L~P = mg(hi − h f ).

Figura 2.17: work done by the


gravitational force

Francesco Saitta, Pordenone Settembre 2019 28


Moti e Dinamica

2.4.5 Work done by elastic force

The elastic force is the tipical force varying on space: ~Fe = −k~s. So,
to calculate the work done by this force we have to look at the graph
Fe − s, since the angle α is always constant α = π and on the graph we
will plot Fe cos(α) = − F = ks, or F = −ks. We can see from picture
(2.18) that the work done by the elastic force is obtained by the area of
( B+b)∗h
the trapezoid defined by the function Fe : L = 2 = 21 ks2i − 12 ks2f .

Figura 2.18: work done by the


elastic force

Thinking on a spring we can say that the elastic force does work trying to
bring the spring back to the equilibrium position, and each compression
or extension of it is done by an external force that works ”against” the
elastic force.

2.4.6 Conservative forces


In this section we define conservative forces: it is a very important concept
in physics, and we will find it in almost all the topics we will study in
these years. A force is defined to be conservative if the work done by it
does not depend on the path travelled from the point particle. In other
words, a force is defined to be conservative, if the work done to bring a
point particle from a point A to a point B depends only on the two points
A and B. The two forces we studied in previous paragraphs, they are
conservative! Indeed
L~P = mghi − mgh f ,
1 2 1 2
L~Fe = ks − ks f ,
2 i 2
the work depends just on si and s f , not on the path done by the point
particle. In next paragraphs we will introduce energy, and it will be clear
the importance of a conservative force in physics.

2.4.7 Energy
In science energy is defined as the ability of a physical system to do work.
What does it mean? It means that a system has to ”contain” energy in or-

Francesco Saitta, Pordenone Settembre 2019 29


Moti e Dinamica

der to do work, and any time a system (the forces inside the system) does
work, it spends energy. Energy is a sort of scientific-money to do work.
with this definition it is clear that energy has the same unit than work, Jou-
le. In next sections we will analize three different energies arising from
the forces we studied before: generic constant force, gravitational force,
elastic force.

2.4.8 Kinetic energy


Let’s analize the following situation (fig.(2.19)): a point particle moving
with constant velocity starts to be under the effect of a constant force
~F: the effect will be that the point particle will move with a uniformly
accelerated motion, with initial velocity vi . Since the particle is moving
v −v
the force will do work: L = ~F ·~s = Fs = ma(s f − si ) = m f∆t i (s f − si ) =
1 a ( ∆t )2 + v
i ∆t
m ( v f − vi ) 2∆t = m(v f − vi )( 12 a∆t + vi ) = m(v f − vi )( 21 (v f − vi ) +
(v +v )
vi ) = m(v f − vi ) f 2 i = 21 m(v2f − v2i ) (here we used the equation of the
uniformly accelerated motion s f = 21 a(t f − ti )2 + vi (t f − ti ) + si , v f =
a(t f − ti ) + vi ). This calculation suggests us that the energy used by the
force to accelerate the particle equals the work done; since this energy is
the difference between the quantity 12 mv2 at the end and at the beginning
of our situation, we can argue that every point particle moving with a
certain speed v it has a ”Kinetic” energy K = 21 mv2 (in italian we will use
Ec ). Every time a point particle is accellerated by an external force the
particle acquires kinetic energy, following the Kinetic energy theorem:

L = ∆K = K f − Ki .

Figura 2.19: Kinetic energy


theorem

Francesco Saitta, Pordenone Settembre 2019 30


Moti e Dinamica

2.4.9 Gravitational energy

We have already seen that the work done by the gravitational force equals
mghi − mgh f : we can define gravitational energy as Ug = mgh, where h
is the spatial coordinate of the point particle referred to its height. We
notice that the possibility to define an energy for the gravitational force
here depends on the fact that the gravitational work depends only on the
initial and the final position of the particle: since the gravitational force
is conservative then we can define a gravitational energy! If the work is
done by the force, the gravitational energy is decreasing, if the work is
done by an external force, the gravitational energy is increasing:

L = −∆Ug .

Figura 2.20: Gravitational ener-


gy

2.4.10 Elastic energy

We have already seen that the work done by the elastic force equals
1 2 1 2 1 2
2 ksi − 2 ks f : we can define elastic energy as Ue = 2 ks , where s is the
spatial coordinate of the point particle referred to its distance from the
equilibrium point. We notice that the possibility to define an energy for
the elastic force depends again on the fact that the elastic work depends
only on the initial and the final position of the particle: since the elastic
force is conservative then we can define an elastic energy! If the work is
done by the force, the elastic energy is decreasing, if the work is done by
an external force, the elastic energy is increasing:

L = −∆Ue .

Francesco Saitta, Pordenone Settembre 2019 31


Moti e Dinamica

Figura 2.21: Elastic energy

2.4.11 Conservation of mechanical energy


All the energies related to conservative forces are called potential energies:
them are the energies that allow a force to do some work, kinetic energy
instead is the energy gained (or lost) by the point particle under the effect
of a force acting on it. An important characteristics of potential energies
is that they all depends only on the position of the point particle. As
written in the previous pictures, in presence of a conservative force it
holds: L = −∆U, so we can say that if a force is conservative the work
done on a closed path (the starting point coincides with the final one),
the work done equals zero: L A→ A = 0. We are ready to enounce the
theorem of conservation of mechanical energy: any conservative physical
system (a system in which all the forces are conservatives) has a constant
mechanical energy, Etot given by the sum of all the potential energies
and the kinetical energy in a given istant of time. It means that if a force
is doing work on a point particle, the force is losing potential energy, while
the point particle is gaining kinetic energy. It can be demostrated in few
simple calculations:
L~F = ∆K,
n n
L~F = ∑ ~Fi = − ∑ ∆Ui ,
i =0 i =0
n
∆K = − ∑ ∆Ui ,
i =0
n
∆K + ∑ ∆Ui = 0,
i =0
n
∆(K + ∑ ∆Ui ) = 0,
i =0

∆Etot = 0,
n
Etot = K + ∑ ∆Ui = const.
i =0

Francesco Saitta, Pordenone Settembre 2019 32


Moti e Dinamica

Figura 2.22: Conservation of


mechanical energy

The simplest example of conservation of energy is described in figure


2.22. Consider a point particle standing on the top of a double slide, as in
figure:

• since it stands at a certain height h, it has a potential energy Ug = mgh;


it means that the gravitational force can do work: the work done when
the point particle start falling along the slide.

• Sliding down, the gravitational energy decreases, while kinetic energy


increases (the point particle accelerates under the effect of the gravita-
tional force).

• When the point particle is on the bottom of the slide, it has no more
gravitational energy, which all trasformed into kinetic energy 21 mv2 .
According to the conservation theorem the energy at this point as to
equal the initial energy, so Ugi = K f , mgh = 21 mv2 .

• The velocity gained by the point particle allows it to move up on the


other side of the slice: gravitational energy starts increasing, kinetic
energy decreases (the point particle slows down while going up)

• When all the initial kinetic energy does its work the point particle stops;
at that point all the kinetic energy transformed into potential energy:
Ki = Ug f , 12 mv2 = mgh.

The conservation of energy is very important theorem in physics basically


for two reasons:

1. it gives us an invariant, a quantity that, under some hypothesis (con-


servative forces), is always constant. It allows us to label a system: the
system with energy 5J, 8J, ...

2. as we will see in paragraph (2.4.13), it is a useful tool to solve problems


in a simpler way than the study of kinematics of point particles

Francesco Saitta, Pordenone Settembre 2019 33


Moti e Dinamica

2.4.12 Work done by non-conservative forces


The tipical example of non-conservative force is friction. Friction is a non
conservative force because the work it does depends on the path travel-
led by the point particle: Fa = µF⊥ , L Fa = − Fa s = −µF⊥ s. Along a
certain path friction does more work than a shorter one. Since friction is
not a conservative force, we cannot define a friction energy: friction always
consumes energy of the system, never gains it. In presence of a non con-
servative force Fnc , the conservation of mechanical energy does not apply,
but we can write:
∆Etot = L Fnc .

2.4.13 Problem Solving


In this paragraph we will see how to set up a procedure to solve most of
conservation of energy based exercises.

1. First step: draw a schetch of the system to study;

2. Second step: draw in the schetch all the energies involved in all the
important points of the problem;

3. Third step: apply the conservation of energy at the different points.

Let’s apply the procedure on the follwing exercise:


A point particle of mass m = 10kg is free-falling from a height h = 10m. Find
its speed the istant before it hurts the floor. Following our steps:

Figura 2.23: First step: The


draw of our system

Francesco Saitta, Pordenone Settembre 2019 34


Moti e Dinamica

Figura 2.24: Second step: The


draw of our system with all the
energies involved

Figura 2.25: Third step: The ap-


plication of the conservation of
energy

p
The solution of the problem (v f = 2gh) is the same if we apply ki-
nematics (as we saw last year and at the beginning of this one), but using
conservation of energy is much faster an simpler!

2.4.14 Power

We define Power as the physical quantity given by the ratio between the
work done by a force and the time taken to make the work:

L
Pm = (2.4.2)
∆t

This physical quantity is measured in Watt (1 W=1 J/s), and tells us the
capacity of a force to do a certain amount of work in a given period of
time. If we think that this work is done on a point particle we can find
a relationship between the average power, the average force acting on the

Francesco Saitta, Pordenone Settembre 2019 35


Moti e Dinamica

point particle and its average velocity:

L
Pm =
∆t
~Fm · ∆~s
=
∆t
∆~s
= ~Fm ·
∆t
= ~Fm · ~vm

The istantaneous power is then the average power when the time interval
∆t is tiny with respect to the phenomenon we are considering, in such
a way that the average force becames the istantaneous force and so the
velocity:
P = ~F · ~v (2.4.3)

We can interpret power as the energy consumed or provided per unit of


time.

A simple example Two runners with the same mass m = 70 kg reach


the same velocity v f = 10 m/s from a standing start. They both make
L = ∆Ec = 3500 J of work; we say that the runner A, who reaches the
velocity v in 10 seconds is much powerful than the runner B who reaches
the velocity v in 20 seconds: the power consumed by A is PA = 350 W,
while the power consumed by B is PB = 175 W. The work done by the two
runners is the same but the power is different.

2.4.15 The Feynman point of view


The approach we used in these paragraphs to the topic of work and ener-
gy is the typical approach used in italian high schools: starting from the
definition of work, then talking about energy, the theorem of conservation
of mechanical energy to get finally to the conservation of total energy (at
the fourth year of high school) studying heat and thermodynamics. But it
is not the only possible way to introduce this topic. For example, the great
physicist Richard Feynman in his famous lectures uses a completely New York, 1918 - Los Angeles,
different approach. In the chapter “What is energy?” Feyman writes: 1988

There is a fact, or if you wish, a law, governing natural phenomena that are kno-
wn to date. There is no known exception to this law; it is exact, so far we know
The law is called conservation of energy; it states that there is a certain quantity,
which we call energy, that does not change in manifold changes which nature
undergoes. That is a most abstract idea, because it is a mathematical principle; it
says that there is a numerical quantity, which does not change when something
happens. It is not a description of a mechanism, or anything concrete; it is just
a strange fact that we can calculate some number, and when we finish watching
nature go through her tricks and calculate the number again, it is the same (Feyn-
man, 2001).
Physicists are very fond of this type of setting because energy conser-
vation is really a very important principle and this aproach does not bind
energy to the work of a force like we did. In this case, the work of a
force becomes a possible form of energy, as well as heat, internal energy

Francesco Saitta, Pordenone Settembre 2019 36


Moti e Dinamica

(which we will study in thermodynamics) or electromagnetic energy. The


approach is interesting and useful, and all the formulas and concepts that
we have just described can also be derived from this idea. Among many
positive aspects there is also a lack in Feynman’s approach: it is hard to
see what difference there is between energy and whatever else is preser-
ved, such as mass or electric charge, for example. Although our approach
is different from this, it is interesting to note, however, as in science there
may be different ways to think and to present its fundamental concepts.

2.5 Meccanica dei fluidi

Nel biennio abbiamo studiato la fluidostatica, ovvero la parte della fisi-


ca che studia i fluidi nei loro stati di equilibrio. Quello che faremo in
questo paragrafo è di considerare fluidi in movimento, applicando le co-
noscenze che abbiamo della fisica fuori equilibrio che abbiamo ripreso ed
approfondito in questo primo capitolo al caso dei fluidi. Prima di entrare
nei dettagli della meccanica dei fluidi riprendiamo brevemente i concetti
principali della fisica dei fluidi in equilibrio.

Fluido Definiamo fluido una sostanza che si trovi in uno stato fisico non
dotato di forma propria. Possiamo per il momento immaginare che fluido
è una qualsiasi sostanza che non si trovi allo stato solido. Parleremo di
fluidi incomprimibili in presenza di un fluido a densità costante (ρ =
m/V = cost).

Pressione Definiamo come pressione P la grandezza fisica che si ottiene


come rapporto tra il modulo della forza perpendicolare ~F⊥ agente su una
superficie e l’ampiezza A della superficie stessa:

F⊥
P=
A

L’unità di misura della pressione nel sistema internazionale si chiama Pa-


scal (Pa): un Pascal è la pressione esercitata da una forza di un Newton
che agisce perpendicolarmente su una superficie di un metro quadrato:
1Pa=1N/1m2 .

Fluido in equilibrio Diremo che un fluido è in equilibrio se, presa comun-


que una superficie aperta7 all’interno del fluido, la pressione esercitata 7
si definisce aperta una superfi-
su un lato della superficie è uguale alla pressione esercitata sull’altro lato cie per cui non si riesca a defini-
re un interno od un esterno al-
della superficie. la superficie. Immaginiamo per
esempio un fazzoletto rettango-
lare o circolare; la superficie di
Principio di Pascal Il principio di Pascal asserisce che in un fluido in equi- una sfera invece non è aperta.
librio una variazione di pressione in un certo punto del fluido viene tra-
smessa inalterata a tutto il fluido, in tutte le direzioni fino alle pareti del
contenitore in cui il fluido è contenuto.

Pressione idrostatica e legge di Stevino Definiamo pressione idrostatica la


pressione in un fluido incomprimibile dovuta alla gravità: un certo fluido
che si distribuisce su diverse altezze (o profondità) risentirà della forza di

Francesco Saitta, Pordenone Settembre 2019 37


Moti e Dinamica

gravità che la parte di fluido più in alto imprime sulla parte più bassa.
Esisterà quindi una differenza di pressione a diverse altezze dovuta alla
gravità, che possiamo trovare grazie alla legge di Stevino:

∆p = ρgh (2.5.1)

dove ρ è la densità del fluido, g l’accelerazione di gravità ed h il dislivello


tra i due punti tra cui stiamo calcolando la differenza di pressione.

Principio di Archimede Il principio di Archimede asserisce che un corpo


immerso in un fluido riceve una spinta dal basso verso l’alto (la spinta di
Archimede appunto), pari al peso del liquido che occuperebbe il volume
del corpo immerso nel fluido. Questo principio, su cui abbiamo ragionato
molto nel primo anno di liceo, non porta molte conseguenze rispetto alla
meccanica dei fluidi per come la studieremo qui, ma è stato riportato per
avere un riferimento completo dei concetti visti nel biennio per quanto
riguarda l’equilibrio dei fluidi.

2.5.1 Fluidi in moto


Descriveremo fluidi in moto tramite il moto delle particelle di cui immagi-
niamo essi siano composti, più nello specifico noi ci riferiremo ad elementi
di fluido: immagineremo cioè di suddividere il fluido in moltissimi volumi
molto piccoli dV che si muovono formando nell’insieme il moto completo
del fluido. La descrizione dei moti di un fluido può essere molto comples-
sa sia dal punto di vista concettuale che matematico, e proprio per questo
motivo faremo delle approssimazioni che ci aiuteranno nella comprensio-
ne e descrizione di questi fenomeni. La descrizione di moti reali dei fluidi
poi potrà essere più o meno fedele a queste approssimazioni, richieden-
do in alcuni casi delle descrizioni più complesse che però non tratteremo
durante il corso del liceo. Le approssimazioni che facciamo per studiare
fluidi in movimento sono:

1. Considereremo solo fluidi incomprimibili

2. Considereremo solo fluidi non viscosi (trascureremo quindi gli attriti


interni del fluido stesso)

3. Considereremo solo moti stazionari, ovvero moti in cui la velocità di


un elemento di fluido resta costante nello spazio v = v(~r )

4. Considereremo solo moti irrotazionali, ovvero moti in cui non esistono


elementi di volume del fluido in cui la traiettoria di un elemento di
fluido sia circolare

In particolare chiameremo fluidi ideali i fluidi che soddisfano alle condi-


zioni (1) e (2).

2.5.2 Equazione di continuità


La prima conseguenza delle richieste di approssimazione fatte nel para-
grafo precedente è l’equazione di continuità. Immaginiamo di avere un

Francesco Saitta, Pordenone Settembre 2019 38


Moti e Dinamica

fluido che scorre all’interno di un tubo orizzontale che varia la sua di-
mensione trasversale. Diciamo che, come rappresentato in figura (2.26) la
sezione passi da un certo valore S1 ad un altro valore S2 , S1 < S2 .
Immaginiamo ora un certo volume V di fluido incomprimibile che at-
traversi la superficie S1 in un intervallo di tempo ∆t: per l’incomprimi-
bilità del fluido uno stesso volume V nello stesso intervallo di tempo ∆t
dovrà attraversare anche la superficie S2 . Immaginando che il volume
sia distribuito su tutta la superficie con un’estensione longitudinale molto
piccola ∆x avremo che il volume che attraversa S1 è dato da V1 = S1 ∆x1
ed il volume che attraversa la superficie S2 è dato da V2 = S2 ∆x2 . Si avrà
dunque

V1 V2
= Figura 2.26: Flusso di un
∆t ∆t fluido attraverso due superfici
S1 ∆x1 S2 ∆x2 differenti S1 ed S2
=
∆t ∆t
∆x ∆x2
S1 1 = S2
∆t ∆t
E dunque, immaginando le estensioni longitudinali molto piccole, così
come l’intervallo di tempo di attraversamento molto piccolo possiamo
scrivere l’equazione di continuità:

Q = Sv = cost (2.5.2)

Questa semplice equazione dà importanti informazioni sul moto dei flui-


di incomprimibili: velocità del fluido e sezione attraversata sono inversa-
mente proporzionali, un fluido ideale, per evitare “ingorghi” deve avere
velocità maggiori man mano che attraversa sezioni più piccole! La quan-
tità Q = Sv si chiama portata e si misura in m3 /s ed è una costante nel
moto di fluidi ideali.

2.5.3 Teorema di Bernoulli


Il teorema di Bernoulli è un’estensione della legge di equilibrio per i fluidi
e rende conto in qualche modo, come vedremo a breve, della conserva-
zione dell’energia anche nel caso di fluidi in movimento. Prendiamo in
considerazione un fluido ideale che faccia un percorso in cui varia non
solo la sezione trasversale come nel caso precedente, ma che vari anche la
sua altezza rispetto al livello del mare, come in figura (2.27).
Evidentemente per portare il fluido verso l’alto ci deve essere del la-
voro esterno fatto contro la forza di gravità che tenderebbe a spingere la
massa di fluido verso il basso. Tale lavoro dovrà essere uguale alla somma
della variazione di energia cinetica del fluido e della variazione di ener-
gia potenziale gravitazionale del fluido nei punti di attraversamento della
superficie S1 e della superficie S2 :

L = ∆Ec + ∆Ug
Figura 2.27: Fluido in movimen-
to in un percorso che varia sia la
Analizzeremo quindi ogni elemento di questa equazione di conservazione sezione che l’altezza
dell’energia per ottenere quello che chiameremo il teorema di Bernoulli.
Sottolineiamo qui come per dimostrare questo teorema stiamo usando un

Francesco Saitta, Pordenone Settembre 2019 39


Moti e Dinamica

approccio “alla Feynman”, stiamo cioè assumendo che l’energia si con-


servi, ovvero che il lavoro fatto dalle forze esterne al sistema bilanci esat-
tamente la somma delle diverse energie in gioco: la cosa è ragionevole
visto che stiamo trascurando gli attriti interni al sistema, ma è pur sempre
un’assunzione che estende il concetto di campo gravitazionale conserva-
tivo e conservazione dell’energia da un sistema di meccanica dei solidi ad
un sistema di meccanica dei fluidi. Immaginiamo quindi che un certo vo-
lume V di fluido attraversi la superficie S1 in un certo intervallo di tempo
∆t: per l’incomprimibilità del fluido dovrà essere quindi che un volume
V di fluido attraversi la superficie S2 nello stesso intervallo di tempo.

Il lavoro L Il lavoro esterno L fatto sul fluido avrà due componenti: la


prima L1 data dalla massa di fluido che spinge il volume V che attraversa
S1 dato da L1 = F1 ∆x1 , con ∆x1 lo spostamento longitudinale del volume
ed il lavoro L2 = F2 ∆x2 sulla superficie S2 subito dalla massa di fluido
che si trova dopo la superficie S2 ; questi due lavori hanno segno opposto:
prenderemo come positivo L1 e negativo L2 immaginando che la massa
di fluido si stia spostando da S1 ad S2 . Il lavoro totale sarà dunque L =
L1 − L2 = F1 ∆x1 − F2 ∆x2 . Per riscrivere in termini di pressione questo
lavoro possiamo moltiplicare e dividere L1 per S1 e moltiplicare e dividere
L2 per S2 ottenendo l’espressione

L = p1 V1 − p2 V2 = ( p1 − p2 )V

L’uguaglianza tra i due volumi può essere assunta sempre in virtù dell’in-
comprimibilità supposta del fluido.

L’energia cinetica ∆Ec La variazione di energia cinetica del fluido è data


da:

1 1 1 1 1
∆Ec = m2 v22 − m1 v21 = ρ2 V2 v22 − ρ1 V1 v21 = ρV (v22 − v21 )
2 2 2 2 2

sempre utilizzando l’incomprimibilità del fluido.

L’energia potenziale gravitazionale ∆Ug La variazione di energia potenziale


del fluido è data quindi da:

∆Ug = m2 gh2 − m1 gh1 = ρVg(h2 − h1 )

con h2 ed h1 le altezze del volume di fluido attraversando rispettivamente


S1 ed S2

L’equazione di Bernoulli Mettendo ora assieme tutti i termini trovati pos-


siamo scrivere la nostra equazione nel seguente modo:

1
( p1 − p2 )V = ρV (v22 − v21 ) + ρVg(h2 − h1 )
2

Questa equazione esprime la conservazione dell’energia nel caso del flui-


do in movimento, ed è una prima versione del teorema di Bernoulli.
Notiamo come questa relazione possa essere riscritta portando al primo

Francesco Saitta, Pordenone Settembre 2019 40


Moti e Dinamica

membro tutti i termini relativi alla superficie S1 ed al secondo membro


tutti i termini relativi alla superficie S2 ottenendo

( p1 + ρv21 + ρgh1 )V = ( p2 + ρv22 + ρgh2 )V

questa espressione mette in evidenza una quantità, che chiameremo anche


pressione generalizzata, ottenuta come somma della pressione del fluido,
la densità di energia cinetica e la densità di energia potenziale gravitazio-
ne, che rimane costante durante tutto il tragitto del fluido! Si ha quindi la
legge di conservazione:

1
p + ρv2 + ρgh = cost (2.5.3)
2

Questo risultato, ottenuto per la prima volta da Daniel Bernoulli , dice Groninga, 1700- Basilea, 1782
come la pressione generalizzata, interpretabile come la densità di energia
di un fluido in movimento, rimanga costante lungo il percorso di un fluido
ideale in moto stazionario ed irrotazionale. Il teorema di Bernoulli, come
vedremo in qualche semplice applicazione qui di seguito è alla base di
moltissimi studi ed applicazioni della meccanica dei fluidi.

2.5.3.1 Applicazioni
L’utilizzo dell’equazione di continuità e del teorema di Bernoulli permette
la descrizione di moltissimi fenomeni di moto dei fluidi, ed ha permesso
nella storia della fisica lo studio e la produzione di moltissimi strumenti ed
apparati tecnologici che hanno a che fare con la fluidodinamica. Vedremo
qui di seguito tre applicazioni, la prima - l’effetto Venturi - tecnologica
per la misura della portata di un fluido, la seconda - la portanza - che
mostra il principio base per il funzionamento degli aeroplani ed infine
alcune applicazioni in campo medico - gli aneurismi e le trombosi.

2.5.4 Tubo di Venturi


Giovanni Battista Venturi è stato il sacerdote e fisico italiano che Bibbiano, 1746 - Reggio Emilia,
inventò uno strumento di misura della portata di un fluido in movimento. 1822

L’apparato sperimentale inventato da Venturi è descritto in figura (2.28):


esso è costituito da un tubo con due diverse sezioni S1 ed S2 collegato ad
un altro tubo ad U posto sotto al primo tubo che si collega ad esso nei
punti di sezione S1 ed S2 . All’interno del tubo con le due sezioni viene
fatto passare orizzontalmente il fluido di cui si vuole misurare la portata; il
tubo ad U invece è un manometro in cui viene posto un liquido che possa
variare le sue altezze a seconda della differenza di pressione presente sui
due rami del manometro stesso. Venturi cercò una relazione tra la portata
del fluido e la differenza di altezza misurata dal manometro, conoscendo
le due sezioni dello strumento. Infatti, per la legge di continuità si ha che
S1 v1 = S2 v2 , dunque la velocità del fluido che attraversa S2 è maggiore
della velocità del fluido che attraversa S1 ; questa differenza di velocità si
manifesta in una differenza di pressione grazie al teorema di Bernoulli,
che viene misurata dal manometro. Otteniamo dunque la relazione che ci

Francesco Saitta, Pordenone Settembre 2019 41


Moti e Dinamica

Figura 2.28: Tubo di Venturi

permette di ottenere la portata in funzione di S1 , S2 e ∆h: considerando il


tratto rettilineo del tubo di Venturi possiamo scrivere:

Q = S1 v 1 = S2 v 2
1 1
p1 + ρv21 = p2 + ρv22
2 2
1 1
p1 + ρ( Q/S1 ) = p2 + ρ( Q/S2 )2
2
2 2
1 2 2
ρ[( Q/S1 ) − ( Q/S2 ) ] = p2 − p1
2
Q2 S22 − Q2 S12 2( p2 − p1 )
2 2
=
S1 S2 ρ
2( p2 − p1 ) S12 S22
Q2 =
ρ S22 − S12
s
2( p2 − p1 ) S12 S22
Q=
ρ S22 − S12

A questo punto possiamo usare la legge di Bernoulli lungo il tubo ad U


per scrivere che

p1 + ρgh1 = p2 + ρgh2
p2 − p1 = ρg(h1 − h2 )

E sostituire il risultato nell’ultima delle equazioni precedenti trovando la


relazione desiderata:
s
S2 S2
Q = 2g(h1 − h2 ) 2 1 2 2 (2.5.4)
S2 − S1

Francesco Saitta, Pordenone Settembre 2019 42


Moti e Dinamica

2.5.5 Portanza
La portanza in fluidodinamica è la forza perpendicolare rispetto alla velo-
cità relativa di un’ala di un aereo o di un veicolo che si muove all’interno
o sopra ad un fluido. Essa è di fondamentale importanza per la stabilità
del veicolo e per la sua capacità di viaggiare. Studiamo in modo sem-
plice la portanza nel caso dell’ala di un aeroplano, facendo riferimento
alla figura (2.29). Vogliamo dare giustificazione del fatto che esiste una

Figura 2.29: Profilo di un’ala


di aeroplano per lo studio della
portanza

differenza p1 − p2 > 0 che genera una forza dal basso verso l’alto sul-
la superficie dell’ala e che aiuta sostanzialmente l’aereo a volare. Per far
questo ci richiamiamo al teorema di Bernoulli per cui:
1
p1 − p2 = ρ(v22 − v21 ) + ρg(h2 − h1 )
2
Notiamo come, dato il profilo dell’ala (ecco l’importanza della forma dei
profili in aerodinamica), l’aria che passa sopra all’ala deve avere una velo-
cità maggiore dell’aria che passa sotto all’ala, in quanto deve percorre più
spazio in meno tempo, sempre assumendo che non si formino moti vor-
ticosi attorno all’ala, cosa che si rivelerebbe pericolosa più che fastidiosa
dal punto di vista del modello fisico; inoltre h2 > h1 : ecco allora che - in
dipendenza della densità dell’aria naturalmente - esiste una differenza di
pressione nella giusta direzione affinché l’aereo possa volare!

2.5.6 Aneurismi e Trombosi


In medicina si definiscono aneurismi le dilatazioni di vene o arterie do-
vute al cedimento delle pareti di una di esse; il nostro intento qui non è
quello di capirne le motivazioni o i rimedi, ma la fluidodinamica del feno-
meno per capire in che direzione potrebbe evolvere l’aneurisma secondo
le regole della fisica. In figura (2.30) vediamo lo schema di un possibile
aneurisma: chiameremo S1 la superficie in cui abbiamo evidenziato la ve-
locità del sangue ~v1 ed S2 quella dove abbiamo evidenziato la velocità ~v2 .
Per l’equazione di continuità avremo evidentemente v2 < v1 . A questo
punto possiamo applicare il teorema di Bernoulli alla nostra vena (suppo-
nendo che sia in orizzontale o che la differenza di altezza sia trascurabile
viste le dimensioni del fenomeno in questione) ed abbiamo:
1
p1 − p2 = ρ(v22 − v21 ) ⇒ p1 < p2
2

Francesco Saitta, Pordenone Settembre 2019 43


Moti e Dinamica

Figura 2.30: Aneurisma

Ovvero la pressione laddove la vena ha ceduto aumenta: il che significa


che dal punto di vista medico la fisica non tende a risolvere il problema,
anzi, serve capire come poter intervenire per non peggiorare il problema.

In medicina di definiscono trombi le ostruzioni di vene o arterie dovu-


te al deposito di sostanze di diversa origine. Evidentemente il fenomeno
dal punto di vista fluidodinamico è l’esatto opposto del precedente, ed è
descritto dalla figura (2.31). Evidentemente, ricalcando la dimostrazione

Figura 2.31: Trombosi

precedente, in questo caso la pressione del fluido diminuisce in corrispon-


denza del trombo, facilitando il deposito di ulteriore sostanza ed anche in
questo caso favorendo il peggiorarsi del fenomeno patologico.

Francesco Saitta, Pordenone Settembre 2019 44


Moti e Dinamica

Francesco Saitta, Pordenone Settembre 2019 45


Moti e Dinamica

2.6 Esercizi

Le equazioni del moto di un punto materiale, quando non specificato in modo diverso, si considerino espresse con le
unità di misura del sistema internazionale. Ad esempio s = 5t + 2 esprime lo spazio in metri in funzione del tempo
in un moto rettilineo uniforme, 5 rappresenta la velocità in metri al secondo, 2 la coordinata iniziale in metri, t il
tempo in secondi.
1. Si descriva il moto rettilineo di un punto materiale A rappresentato in figura (2.32), trovando i moduli
delle velocità dei diversi tratti di percorso del punto materiale. Qual è la velocità media su tutto il
percorso vm ? Quante volte si incontrerebbe il punto materiale A con il punto materiale B che parte
nello stesso istante di A ma muovendosi costantemente alla velocità v B = vm ?

Figura 2.32: .

[v1 =2 m/s; v2 =0 m/s; v3 =2 m/s; v4 =3,3 m/s; v5 =0 m/s; v6 =2,2 m/s; vm =0,75 m/s; due volte,
escludendo il punto di partenza]
2. Due punti materiali A e B si stanno muovendo, secondo un sistema di riferimento fissato s con le
seguenti equazioni del moto: s A = 7t, s B = 7t + 5. I due punti materiali si incontreranno? Perché?

[No, perché...]
3. Due punti materiali A e B si stanno muovendo, secondo un sistema di riferimento fissato s con le
seguenti equazioni del moto: s A = 7t, s B = −7t + 7. Dove (lungo s) e quando si incontreranno?
Si esprima la soluzione anche dal punto di vista geometrico disegnando il diagramma orario della
situazione.

[t=0,5 s; s=3,5 m]
4. Un calciatore A sta correndo in linea retta dalla sua area di rigore verso il centrocampo alla velocità
v A = 5m/s. Il portiere lancia la palla, sempre in linea retta, quando il calciatore A si trova a 30 metri
dal punto in cui il portiere calcia il pallone. Se la palla raggiunge il giocatore si trova a 60 metri dal
punto in cui il pallone è stato calciato dal portiere, a che velocità il portiere ha calciato il pallone? (Si
approssimino i moti del calciatore e della palla con moti rettilinei uniformi). Si descriva la situazione
in un diagramma orario.

[v P =10 m/s]

Francesco Saitta, Pordenone Settembre 2019 46


Moti e Dinamica

5. A causa di un colpo di sonno, Gigetto chiude gli occhi alla guida, mentre l’auto si muove ad una
velocità v A = 80km/h verso un albero che si trova ad una distanza d = 120m. Quanto tempo ha per
svegliarsi, sapendo che una frenata a quella velocità necessita 30 metri per fermare l’auto?

[t=4,05 s]
6. Un’automobile da corsa raggiunge la velocità di 100 km/h in 3,0 secondi, partendo da ferma. Quanto
spazio ha percorso in questo tempo supponendo che la variazione della sua velocità sia costante nel
tempo?

[s=42 m]
7. Un punto materiale A si muove secondo l’equazione del moto s A = 10t2 + 2t − 3, mentre un punto B
s B = 5t + 3. Si incontreranno? Dove e Quando? Si esprima la soluzione anche dal punto di vista geo-
metrico disegnando il diagramma orario della situazione. Si siano le soluzioni con 2 cifre significative.

[s=7,7 m; t=0,94 s]
8. Antonio (A) sta correndo alla velocità v a = 7m/s. Barbara (B) inizia a ricorrere Antonio quando
questo si trova ad una distanza d = 30m. Quale accelerazione deve avere per raggiungere Antonio
in 10s? Si esprima la soluzione anche dal punto di vista geometrico disegnando il diagramma orario
della situazione.

[a A =2 m/s2 ]
9. Un automobile sta viaggiando alla velocità v A = 72km/h quando a 50m un semaforo diventa rosso.
Se il tempo di reazione del guidatore è di mezzo secondo, qual è la minima accelerazione che deve
avere l’automobile per fermarsi al semaforo? Si esprima la soluzione anche dal punto di vista geome-
trico disegnando il diagramma orario della situazione.

[a=-5 m/s2 ]
10. Due corridori A e B stanno correndo verso il traguardo appaiati ad una velocità v = 8m/s. A 30m
dal traguardo il corridore A inizia ad accelerare e vince la gara per 0,50 secondi. Quanto è stata la sua
accelerazione media negli ultimi 30 metri?

[a=0,76 m/s2 ]
11. Un punto materiale A si muove secondo l’equazione del moto s A = 8t2 − t + 5, mentre un punto
B s B = −4t2 + 5t + 3. Si incontreranno? Dove e Quando? Si esprima la soluzione anche dal punto
di vista geometrico disegnando il diagramma orario della situazione. Si siano le soluzioni con 2 cifre
significative.

[Non si incontreranno mai, perchè...]


12. Un punto materiale A si muove secondo l’equazione del moto s A = 8t2 − t + 3, mentre un punto B
s B = −4t2 + 5t + 5. Si incontreranno? Dove e Quando? Si esprima la soluzione anche dal punto di
vista geometrico disegnando il diagramma orario della situazione. Si diano le soluzioni con 2 cifre
significative.

[s=6,5 m; t=0,73 s]
13. I due fratelli Antonio (A) e Barbara (B) stanno facendo il seguente gioco: Antonio sta alla finestra
che si trova ad un’altezza h = 3m da terra, mentre Barbara si trova ad una distanza d = 10m dal muro
della finestra, Antonio lascia cadere una palla in verticale sotto l’effetto della gravità, mentre Barbara

Francesco Saitta, Pordenone Settembre 2019 47


Moti e Dinamica

lancia una palla orizzontalmente rasoterra verso il muro. Che velocità deve avere la palla lanciata da
Barbara affinché le due palle si scontrino?

[v P =13 m/s]
14. La giostra Columbia di Mirabilandia è definita come free fall tower: è una struttura alta 60m che simula
la caduta libera per chi si siede sulle poltroncine che salgono fino alla cima dell’attrazione. Suppo-
nendo di poter trascurare gli attriti, con quale velocità arrivano a terra le persone che provano questa
giostra?

[v=123 km/h]
15. Un altro gioco di Antonio (A) e Barbara (B) è il seguente: Antonio lancia una pallina rasoterra con
una certa velocità verso Barbara, la quale lascia cadere in verticale un’altra pallina cercando di colpire
la pallina lanciata da Antonio. Supponendo che Antonio e Barbara siano ad una distanza d = 3m e
che la velocità con cui Antonio lancia la pallina sia v A =20cm/s, dopo quanto tempo dal lancio della
pallina del fratello Barbara deve lasciar andare la pallina dall’altezza h = 1, 5m per riuscire nel gioco?

[t=14,7 s]
16. Considerando le dimensioni massime regolamentari di un campo da calcio e supponendo che un
portiere calci dando dalla palla una velocità iniziale formante un angolo α = 30 con l’orizzontale qual
è il modulo minimo di questa velocità affinché il portiere faccia una rimessa dalla sua area piccola che
raggiunga la linea di fondo dell’altra metà campo? Qual è l’altezza massima raggiunta dal pallone
supponendo che la velocità iniziale sia questa velocità minima?

[v0 =129,6 km/h; hmax =7,7 m]


17. Alberto (A) e suo figlio Bruno (B) giocano a passarsi la palla colpendola di testa. Alberto è alto
h A =180 cm, mentre Bruno h B =150 cm. Alberto colpisce la palla con una velocità iniziale v0 =2 m/s con
un angolo rispetto l’orizzontale α = 45 e la palla cade esattamente sulla testa di Bruno. A che distanza
orizzontale si trovano i due?

[d=60 cm]
18. Qual è la velocità iniziale che un cestista deve dare alla palla per segnare un tiro libero, supponendo
che l’angolo rispetto l’orizzontale del lancio sia α = 60 e che la palla si stacchi dalla mano del gio-
catore ad un’altezza h = 1, 95m. Si assumano le dimensioni regolamentari del campo da pallacanestro.

[v0 =7,8 m/s]


19. Al giorno d’oggi (Marzo 2014) il record del mondo per il lancio del giavellotto misura d = 98, 48m,
ottenuto nel 1996 dall’atleta ceco Jan Zelezny 8 . Supponendo che l’altezza iniziale
8
del giavellotto sia
http://www.iaaf.org/
records/by-discipline/
di un metro, e che l’angolo di tiro sia α = 30, si trovi la velocità iniziale impressa dal campione ceco
throws/javelin-throw/
nel tiro del record. outdoor/men

[v0 =33,4 m/s]


20. Alessandro vuole saltare un ostacolo alto h = 80 cm. Supponendo che si avvicini all’ostacolo con una
velocità v = 5 m/s e stacchi, mantenendo il modulo della sua velocità pari a v, con un angolo rispetto
l’orizzontale α = 60 ad una distanza d = 1 m dall’ostacolo. Alessandro riuscirà a saltare l’ostacolo?
Se si, quanto di quanto supera in altezza l’ostacolo quando ci si trova sopra? Qual è l’altezza massima
che raggiunge?

[Sì, supera l’ostacolo di 0,15 m; hmax =96 cm]

Francesco Saitta, Pordenone Settembre 2019 48


Moti e Dinamica

21. Antonio vuole lanciare un sasso oltre una siepe che sta di fronte a lui. Antonio si trova a 5 metri
dalla siepe, che è alta 2 metri, e lancia il sasso lasciandolo andare quando si trova ad un’altezza di 1,2
metri con una velocità di modulo v0 = 10 m/s che forma un angolo di 30 gradi con l’orizzontale. Il
sasso andrà oltre la siepe? Se sì, quanto sopra alla siepe passerà? A che distanza dalla siepe toccherà
poi terra?

[Sì, passa 0,5 metri sopra; d=5,6 m]


22. Un giocatore di calcio vuol mostrare la sua bravura colpendo la traversa con un calcio da 30 metri
dalla linea di porta. Supponendo che l’angolo di tiro del giocatore sia α = 20, qual è la velocità che
deve imprimere al pallone per riuscire nel suo intento? Si assuma l’altezza della porta regolamentare,
2,44 metri.

[v0 = 24,2 m/s]


23. Due giocatori di golf sono uno di fronte all’altro ad una distanza d = 10 m l’uno dall’altro. Il primo
giocatore effettua un tiro dando alla palla una velocità iniziale v01 =50 m/s con un angolo rispetto
l’orizzontale α = 30, mentre il secondo colpisce la sua palla dandole una velocità di modulo v02 che
forma un angolo β = 45 con l’orizzontale. Supponendo che i due colpi avvengano nello stesso istante,
quale deve essere la velocità v02 affinché le due palline si colpiscano? In tal caso, in quale punto
avviene l’urto tra le palline (distanza dal primo giocatore d1 ed altezza da terra h)?

[v02 =35,4 m/s; d1 =6,5 m; h=3,6 m]


24. Un pompiere deve spegnere il fuoco che divampa da una finestra che si trova a 10 metri in orizzon-
tale e 5 in verticale rispetto la sua posizione. Supponendo che l’angolo con cui il pompiere punta la
bocca del tubo dell’acqua sia α = 60 rispetto l’orizzontale si trovi la velocità iniziale che deve avere
l’acqua affinché il pompiere riesca a centrare la finestra e spegnere il fuoco.

[v=12,5 m/s]
25. Lasciando suonare un vecchio disco 45 giri per un’ora, quanto angolo è stato spazzato da uno dei
raggi del disco? Lo si esprima sia in radianti che in gradi.

[α = 17 rad ∼ 974°]
26. Un ragazzo lancia un sasso ad una distanza d = 100 metri con velocità iniziale orizzontale da un’al-
tezza h = 3 metri dal suolo ottenuta facendo ruotare uno spago lungo l = 20 cm con un moto circolare
uniforme. Si calcoli la frequenza con cui deve ruotare lo spago per ottenere un lancio del genere.

[f=102 Hz]
27. Si calcoli l’accelerazione centripeta e la velocità tangenziale di un oggetto sulla superficie terrestre
approssimando la terra ad una sfera che ruota attorno ad un suo diametro.

[ac = 3, 38 × 10−2 m/s2 ; v = 470 m/s]


28. Si calcolino le frequenze f m ed f s della lancetta dei minuti e dei secondi delle lancette di un orologio
a parete. Si calcolino inoltre le velocità angolari ωm , ωs delle due lancette e le accelerazioni centripete
di due punti delle lancette entrambi ad una distanza d = 5cm dal centro.

[ f m =2,8×10−4 Hz; f s =0,17 Hz; ωm =1,7×10−3 rad/s; ωs =0,1 rad/s; am =1,4×10−7 m/s2 ; as =5×10−4
m/s2 ]
29. Si trovi l’equazione del moto di un moto armonico con ampiezza 350 cm, pulsazione 5 rad/s e fase π
usando le unità standard del sistema internazionale. Si calcoli la posizione iniziale del punto materiale

Francesco Saitta, Pordenone Settembre 2019 49


Moti e Dinamica

di cui è descritto il moto. Si calcoli la velocità massima del moto del punto materiale.

[s=3,5cos(5t+π); s0 =-3,5 m; vmax =17,5 m/s]


30. Si trovi la pulsazione di un moto armonico con fase nulla la cui velocità massima è vmax = 10 m/s e
la cui posizione al tempo t = 0 s è s = 2 m

[ω = 5rad/s]
31. Un punto materiale si sta muovendo di moto armonico di pulsazione ω = 5 rad/s. Quale sarà la sua
posizione nell’istante in cui la sua accelerazione è a = 0, 4 m/s2 ?

[s=1,6 cm]
32. Un elastico di lunghezza a riposo l0 = 10 cm poggia su un tavolo ed è fissato ad una sua estremità
da un chiodo. All’altra estremità dell’elastico è fissata una massa m = 500 g. Se la massa sta ruotando
con una frequenza f = 50 Hz provocando un allungamento ∆l = 1, 5 cm si calcoli la costante elastica
della molla.

[k = 378, 3 kN/m]
33. Che velocità deve avere il supereroe Flash per riuscire a correre in orizzontale sulla parete verticale
di un palazzo cilindrico con la base di raggio R = 30 m, supponendo che il suo peso sia P = 780 N, ed
il coefficiente d’attrito tra le sue scarpe e la parete k A = 0, 8? Di quanto cambia la velocità se la massa
di Flash triplica?

[v = 69 km/h; la velocità non dipende dalla massa di Flash, perché...]


34. Una pallina si sta muovendo di moto rettilineo uniforme su un piano senza attrito ad una velocità
v0 = 20 km/h. Ad un certo punto entra in una zona in cui è presente sul pavimento un materiale il
cui coefficiente di attrito con la pallina è k A = 0, 5. Dopo quanto tempo e in quanto spazio la pallina
si fermerà?

[t=1,1 s; s=3,2 m]
35. Quale massa m B indicherà una bilancia nel “pesare” un uomo di massa m = 80 kg all’interno di un
aereo che sta accelerando verso l’alto con un’accelerazione a = 5 m/s2 ? A che peso in realtà corri-
sponde?

[m B =76 kg; P=744 N]


36. Si trovi l’accelerazione del sistema in figura (2.33), considerando il piano senza attrito, il filo come
inestensibile e la carrucola di dimensioni e massa trascurabili rispetto al fenomeno in studio.

[a=0,88 m/s2 , facendo scendere M2 ]


37. Si trovi l’accelerazione del sistema in figura (2.34), considerando il piano senza attrito, il filo come
inestensibile e la carrucola di dimensioni e massa trascurabili rispetto al fenomeno in studio.

[a=0,69 m/s2 facendo scendere M1 ]


38. Si risolva il problema 4 nel caso in cui tra il piano e la massa M2 ci sia un coefficiente di attrito
k A = 0, 3.

[a=0,025 m/s2 ]

Francesco Saitta, Pordenone Settembre 2019 50


Moti e Dinamica

Figura 2.33: .

Figura 2.34: .

39. Si risolva il problema 7 nel caso in cui tra il piano e la massa M2 ci sia un coefficiente di attrito
k A2 = 0, 2 e tra il piano e la massa M1 un coefficiente k A1 = 0, 8.

[a=0 m/s2 ]
40. Un ragazzo sta giocando a tirare un sasso di massa m = 50 g con una fionda. La fionda ha un elastico
di costante k = 10000 N/m allungabile fino a 10 cm rispetto alla sua lunghezza a riposo. Supponendo
che il ragazzo lanci il sasso da un’altezza h = 1, 2 m con un angolo α = π/6 rispetto l’orizzontale, si
calcoli la gittata massima del sasso.

[x=180 m]
41. Immaginiamo di approssimare il funzionamento di un tappeto elastico come il funzionamento di
una molla. In questa ipotesi, data la costante elastica del tappeto k = 20000 N/m, si calcoli a che altez-
za minima da terra deve essere messo il tappeto affinché un persona di massa m = 80 kg raggiunga
l’altezza h = 3 m sopra al tappeto stesso.

Francesco Saitta, Pordenone Settembre 2019 51


Moti e Dinamica

[h0 = 0, 5 m]
42. Un punto materiale di massa m si trova in cima ad un piano inclinato liscio alto h, alla base del
quale si trova un piano orizzontale scabro con coefficiente d’attrito tra piano e massa k. Si calcoli il
lavoro fatto dalla forza d’attrito nel piano orizzontale per fermare il punto materiale; si calcoli inoltre
lo spazio (percorso sul piano orizzontale) necessario per fermare il punto materiale. cc
[L= mgh; s=h/g]
43. Nella costruzione di una pista per biglie si vuol fare un giro della morte dopo la partenza da ferma
della biglia da una certa altezza h. Qual è il minimo valore di h per riuscire a far compiere il giro della
morte alla biglia approssimando il giro della morte con una circonferenza di raggio R?

[hmin = 5R/2]
44. Una massa m = 10 kg che comprime una molla orizzontale (k = 100 N/m) di una quantità ∆x = 20
cm viene lasciata muoversi sotto l’effetto della forza elastica. Supponendo che il piano su cui giacciono
molla e massa sia caratterizzato da una costante d’attrito tra massa e piano k = 0, 1 si calcoli il lavoro
fatto dalla forza d’attrito per fermare la massa.

[L=2 J]
45. Una pallina di massa m = 500 g viene spinta in salita lungo un piano inclinato di un angolo α = π/4
da una forza costante ~F di modulo F = 5 N. Se la velocità della pallina all’inizio del piano è vi = 1
m/s e alla fine del piano v f = 3 m/s, quanto lavoro ha fatto la forza ~F? Quanto è lungo il piano
inclinato? Quanto lontano dal piano inclinato cadrà la pallina se continua nel suo moto?

[L=2 J; l=40 cm; d=1,14 m]


46. Data la figura (2.35) si calcolino le velocità o le altezze nei punti indicati del punto materiale di massa
m = 1 Kg.

Figura 2.35: .

[v B = 8, 9 m/s2 ; hC = 3, 7 m; v D = 7, 7 m/s; v E = 9, 9 m/s]


47. Una gru deve portare degli oggetti da terra ad un altezza h = 5 m. Che potenza deve sviluppare il
motore della gru per portare un carico massimo di 100 oggetti da 50 kg in due ore?

Francesco Saitta, Pordenone Settembre 2019 52


Moti e Dinamica

[P=34 W]
48. La fabbrica costruttrice di una moto da corsa dichiara che il modello di punta può portare la moto
da ferma ai 100 km/h in 3 secondi. Supponendo una massa di 150 kg, quale potenza viene consumata
dalla moto in tale accelerazione?

[P= 19,3 kW]


49. Un ragazzino gioca su un altalena le cui catene sono lunghe l = 2, 5 m partendo da fermo formando
un angolo α = 30 rispetto l’orizzontale. Se la massa del ragazzino è m = 60 kg, si calcoli la tensione
delle catene dell’altalena nel punto più basso della traiettoria.

[T=743 N]
50. A force F = 10 N is acting on a point particle of mass m = 10 kg, with initial speed vi = 0 m/s. Find
the work done by the force after ∆t = 10 s.

[500 J]
51. A point particle is standing on the floor. It is pushed by a force F = 10 N making an angle α = π3
with respect to the ground. Find the work done by the force after a displacement of the point particle
along the floor s = 5 m.

[25 J]
52. A point particle (m = 5 kg) is moving with speed v = 50 m/s on a floor with friction coefficient
µ = 2. Find the work done by friction when the point particle stops.

[6250 J]
53. A point particle (m = 2 kg) is moving with speed v = 10 m/s. An external force does on the point
particle a work L = 100 J. Find the final velocity of the point particle.

[14 m/s]
54. Find the work that a force has to do to expand a spring, with k = 10 N/m, from the equilibrium
(l = 5 m) to l = 15 m.

[500 J]
55. Find the energy of the system made up by two identical point particles (m = 1 kg), one moving on
the ground (without friction) with speed v = 10 m/s, the other free falling down, knowing that at
some point the second point particle is at h = 2 m with v = 10 m/s.

[120 J]
56. A spring (k = 1000 N/m) in vertical position is compressed by an external force of ∆l = 0, 1 m; a
point particle (m = 1 kg) on the top of it is launched up by means of the elastic force. Find the higher
point reached by the point particle.

[0,5 m]
57. A point particle (m = 5 kg) free-falls from a height h = 10 m on the top of a spring (k = 1000 N/m).
What speed does the point particle have before hitting the spring?

[14 m/s]

Francesco Saitta, Pordenone Settembre 2019 53


Moti e Dinamica

58. A point particle of mass m = 1, 00 kg is launched by means of a spring (k = 500, 00 N/m) com-
pressed of ∆l = 20, 00 cm and inclined by α = π3 with respect to the floor. Find the maximum height
reached by the point particle, and its kinetic energy at that point.

[0,81 m;2 J]
59. A block of mass m = 10 kg is pushed up on an inclined plane (α = π/4) with an initial velocity
v0 = 2 m/s. If the constant of friction between the block and the plane is k = 0, 5, what is the speed
of the block when it returns to its original position? What is the intensity of the work done by the
friction during this phenomenon?

[v f = 1, 2 m/s; L FA = 13, 2 J]

Francesco Saitta, Pordenone Settembre 2019 54


3 Relatività Galileiana

Relatività Galileiana]cap2 In questo capitolo discuteremo come un feno-


meno possa essere descritto da un osservatore in quiete rispetto al feno-
meno e da un osservatore in moto uniforme rispetto al primo, eviden-
ziando i legami tra le due descrizioni. Il primo scienziato a parlare di
come queste due descrizioni possano essere messe in relazione tra loro
fu Galilei, da cui il nome di relatività galileiana a questa parte di fisica,
dove relatività indica proprio che la descrizione dei fenomeni è relativa
a sistemi di riferimento diversi tra loro. Introdurremo le leggi di com-
posizione e le trasformazioni di Galileo, assieme al principio di relatività
galileiano, relazioni che furono alla base di tutta la fisica fino alla fine del
1800, quando gli scienziati si accorsero che questa teoria non era valida
per la radiazione elettromagnetica (come vedremo nel quinto anno di cor-
so). Albert Einstein propose delle soluzioni ai problemi creati dalla Ulm, 1879 - Princeton, 1955
relatività classica con la teoria della relatività, prima ristretta nel 1905 e
poi generale nel 1916, che risultano essere un ampliamento di ciò che an-
dremo a studiare in questo capitolo. Uno dei problemi filosoficamente più
importanti per la teoria della relatività galileiana è quello della definizione
del sistema di riferimento in quiete assoluta: per affermare infatti se due
sistemi sono inerziali tra loro dobbiamo ammettere che esista un sistema
di riferimento in quiete assoluta, ovvero secondo un qualsiasi osservatore,
rispetto al quale definire lo stato di moto di tutti i punti materiali dell’u-
niverso. Già Galilei tentò di risolvere questo problema individuando il
sistema assoluto con quello solidale alle stelle fisse, argomento che diven-
tò presto inconsistente grazie alle osservazioni celesti sempre più precise
(non ci sono stelle fisse); Newton invece, mosso dal suo sentimento reli-
gioso panteistico, disse che il sistema assoluto era quello solidale con Dio.
Entrambe queste proposte non ebbero grande seguito nella storia della
fisica; gli scienziati del diciottesimo e diciannovesimo secolo furono con-
vinti dell’esistenza dell’etere: una sostanza impalpabile ed invisibile sulla
quale doveva propagarsi la luce e che doveva essere in quiete assoluta.
Ogni tentativo sperimentale di rilevare l’etere risultò però fallimentare,
portò anzi alla formulazione delle teorie di relatività di Einstein, con cui
come già detto si risolsero alcuni dei problemi posti già da Galilei e New-
ton.

Nella nostra descrizione immagineremo sempre un sistema di riferi-


mento in quiete, ed uno in moto rettilineo uniforme rispetto al primo,
come descritto in figura (3.1). Per quanto riguarda i prossimi paragrafi
utilizzeremo sempre la convenzione per cui le quantità senza pedice sono

Francesco Saitta, Pordenone Settembre 2019 55


Relatività Galileiana

le quantità misurate dal sistema di riferimento considerato in quiete, le


quantità con pedice r sono quelle misurate dal sistema di riferimento in
movimento e le quantità con pedice t sono riferite al sistema di riferimento
in movimento (la sua posizione, la sua velocità,...). Per rendere concrete le
leggi dei seguenti paragrafi, che valgono comunque in generale, immagi-
niamo una semplice situazione esemplificativa: consideriamo il passaggio
di un treno in una stazione in cui il treno stesso non sosta, supponendo
rettilineo uniforme (~v = ~vt ) il moto del treno. Il sistema di riferimento
in quiete è solidale alla stazione, possiamo immaginare un personaggio A
seduto su di una panchina che osserva il passaggio del treno, mentre il
sistema in movimento è solidale con il treno stesso, possiamo immaginare
un altro personaggio B seduto all’interno del treno.

Figura 3.1: Sistemi di riferimen-


to inerziali

3.1 Spazio e tempo nella fisica classica

3.1.1 Il carattere assoluto di spazio e tempo: Galilei e Newton


Tutta la fisica classica si basa sui concetti primitivi di spazio e tempo as-
soluti. Nello sviluppo della meccanica Galilei e Newton fecero esplicito
riferimento a questi concetti e tutta la comunità scientifica, fatta eccezio-
ne per alcuni casi che discuteremo nel prossimo paragrafo, guardava allo
spazio ed al tempo come a delle cornici all’interno delle quali descrivere

Francesco Saitta, Pordenone Settembre 2019 56


Relatività Galileiana

i fenomeni naturali. Descrivere un fenomeno utilizzando un sistema di


riferimento faceva capo solamente alla convinzione che l’universo fosse
scritto in termini matematici (Galilei, 1623) e la descrizione con le coordi-
nate fosse solamente il più semplice stratagemma tecnico per farlo. Questa
concezione fu uno dei pilastri filosofici su cui si basò tutta la scienza clas-
sica: il suo superamento con le teorie di Einstein all’inizio del ventesimo
secolo fu sicuramente uno dei momenti più rivoluzionari della storia della
fisica.

Il tempo è indipendente da ogni elemento esterno. Esso è pura durata, da


non confondere con il tempo umano, sensibile ed imperfetto, e neppure
con il tempo meteorologico. “ Il tempo assoluto, vero, matematico, in sé e per
sua natura senza relazione con alcunché di esterno, scorre uniformemente, e con
altro nome è chiamato durata; quello relativo, apparente e volgare, è una misura
(accurata o approssimata) sensibile ed esterna della durata per mezzo del moto che
comunemente viene impiegata al posto del vero tempo; tali sono l’ora il giorno, il
mese, l’anno.” (Newton, 1687).

Lo spazio non ha alcun rapporto con l’esterno. Lo spazio assoluto, uguale


ed immobile è condizione dello spazio, che nella sensazione di dà unica-
mente in relazione ai corpi. “Lo spazio assoluto, per sua natura senza relazione
ad alcunché di esterno, rimane sempre uguale ed immobile; lo spazio relativo è una
dimensione mobile o misura dello spazio assoluto, che i nostri sensi definiscono
in relazione alla sua posizione rispetto ai corpi, ed è comunemente preso come lo
spazio immobile; così la dimensione di uno spazio sotterraneo o aereo o celeste
viene determinata dalla sua posizione rispetto alla terra. Lo spazio assoluto e lo
spazio relativo sono identici per grandezza e specie, ma non sempre permangono
identici quanto al numero. Infatti se la Terra, per esempio, si muove, lo spazio
della nostra aria, che relativamente alla Terra rimane sempre identico, sarà ora
una parte dello spazio assoluto attraverso cui l’aria passa, ora un’altra parte di
esso; e così muterà assolutamente in perpetuo.” (Newton, 1687).

3.1.2 Il concetto di spazio secondo Berkeley e Mach


Nonostante il sostanziale accordo con la natura assoluta di spazio e tem-
po dei contemporanei di Newton, ci furono delle voci che si dissociaro-
no da questa idea i cui principali esponenti furono il vescovo britannico
George Berkeley ed il fisico e filosofo austriaco Ernst Mach . Kilkenny, 1685 - Oxford, 1753
Brno, 1838 - Haar, 1916

Berkeley si oppose all’idea dello spazio assoluto in quanto esso non è


osservabile. Egli sottolineò il fatto che ogni moto esiste in quanto con-
testualizzato in un sistema di corpi e masse, senza i quali non potrebbe
nemmeno essere descritto. Alcuni anni dopo i principia di Newton egli
scrisse:

“Se ogni luogo è relativo, anche ogni moto è relativo; e non si può compren-
dere il moto se non se ne è determinata la direzione, che a sua volta non si può
comprendere se non in relazione al nostro o a qualche altro corpo. In su, in giù,
a destra, a sinistra, tutte le direzioni e i luoghi si basano su qualche riferimento

Francesco Saitta, Pordenone Settembre 2019 57


Relatività Galileiana

ed è necessario presupporre un altro corpo distinto da quello in movimento... per


cui il moto è per sua natura relativo, né può essere compreso finché non si consi-
derino i corpi in relazione a cui esiste; più generalmente, non si può stabilire un
riferimento, se mancano i termini che in riferimento debbono esser posti. Perciò,
se supponiamo che tutto si annulli eccetto un globo, è impossibile immaginare un
qualsiasi movimento di tale globo. Consideriamo ora che i globi siano due, e che
oltre ad essi non esista alcun’altra cosa materiale: il moto circolare di questi due
globi intorno al loro centro comune non può essere immaginato. Ma ammettiamo
che venga improvvisamente creato il cielo delle stelle fisse: saremo allora in condi-
zione di immaginare il moto dei globi per mezzo della loro posizione relativa alle
varie parti del cielo.”(Berkeley, 1721)Berkeley evidentemente si oppose alla
concezione del moto assoluto e quindi della stessa esistenza di un sistema
di riferimento in quiete assoluta, e da diversi studiosi è considerato un
precursore delle teorie della relatività di inizio novecento sviluppate da
Einstein.

Mach in tempi in cui l’autorità delle idee Newton era indiscussa, pari
se non maggiore rispetto all’autorità delle idee di Aristotele nel dicias-
settesimo secolo, riprese le idee del vescovo britannico riaccendendo la
discussione sulla natura dello spazio. Il punto di vista di Mach è molto
simile a quello di Berkeley:

“...Secondo me esistono solo moti relativi... Quando un corpo ruota rispet-


to alle stelle fisse, si producono forze centrifughe; quando ruota rispetto a qualche
altro corpo e non in relazione alle stelle fisse, non si producono forze centrifughe.
Non ho nulla in contrario a chiamare rotazione la prima, purché ci si ricordi che
non vuol dire altro che rotazione rispetto alle stelle fisse.”

“Ovviamente non ha importanza se noi pensiamo che sia la Terra a ruotare


intorno al suo asse, o che essa sia ferma mentre le stelle fisse le girano intorno.
Da un punto di vista geometrico, in tutti e due i casi, si tratta di un moto relativo
della Terra e delle stelle fisse l’una rispetto alle altre. Ma se ammettiamo che la
Terra sia ferma e che le stelle fisse le girino intorno, non c’è schiacciamento del-
la Terra..., almeno secondo la concezione che noi abbiamo della legge di inerzia.
Ora possiamo superare questa difficoltà in due modi: o ogni moto è assoluto, o
la nostra legge di inerzia è formulata in modo sbagliato. Io preferisco la seconda
soluzione. La legge di inerzia deve essere concepita in modo da portare allo stesso
risultato sia nella prima che nella seconda ipotesi. È perciò evidente che nella sua
formulazione bisogna tener presenti le masse dell’Universo”(Mach, 1893)Mach
quindi riprende gli argomenti di Berkeley e come vedremo ispirerà Ein-
stein nei suoi ragionamenti che lo portarono alle formulazioni della teoria
della relatività speciale prima e generale poi.

3.2 Legge di composizione delle posizioni e degli spostamenti

Ritorniamo ora alla situazione del treno e dei due osservatori A e B de-
finita nell’introduzione a questo capitolo: supponiamo di chiedere ad un
certo istante di tempo ai due osservatori A e B quale sia la posizione della
punta del treno. Evidentemente per l’osservatore B, quello seduto all’in-

Francesco Saitta, Pordenone Settembre 2019 58


Relatività Galileiana

terno del treno, avrà una certa posizione ~st che non dipende dal tempo (la
punta del treno sarà sempre un certo numero di metri davanti a lui); per
l’osservatore A invece la punta del treno si trova in un punto diverso, ~s,
ottenibile come la somma del vettore posizione del tizio B sommato alla
posizione misurata dal tizio B, come illustrato in figura (3.2). La formula

Figura 3.2: Legge di composi-


zione delle posizioni

generale che possiamo ottenere è dunque:

~s = ~sr +~st , (3.2.1)

con ~s il vettore posizione misurato dall’osservatore fermo, ~st il vettore


posizione dell’osservatore in movimento rispetto all’osservatore fermo e
~sr il vettore posizione misurato dall’osservatore in movimento. Allo stesso
modo possiamo pensare alla misura di un certo spostamento, ottenendo
la seguente formula:
∆s
~ = ∆s
~ r + ∆s
~ t (3.2.2)

3.3 Legge di composizione delle velocità

Immaginiamo ora che un tizio C si muova con velocità costante all’interno


del treno partendo da B ed andando verso la punta del treno. Il signor B
misurerà una certa velocità ~vr = ∆s ~ r /∆t data dal rapporto tra lo sposta-
mento effettuato dal signor C nel suo sistema di riferimento e l’intervallo
di tempo ∆t che il signor C ha impiegato a percorrere ∆s ~ r . Il signor A
invece misurerà una velocità maggiore, data dalla somma tra la velocità
del treno e la velocità di C all’interno del treno. La situazione è espressa
dall’equazione:
~v = ~vr + ~vt (3.3.1)

Francesco Saitta, Pordenone Settembre 2019 59


Relatività Galileiana

In questo ragionamento è fondamentale la convinzione classica che l’in-


tervallo di tempo ∆t sia lo stesso per l’osservatore A e l’osservatore B:
nella fisica classica il tempo ha un carattere assoluto, scorre nello stesso
modo in qualsiasi sistema di riferimento. Per noi questo concetto è scon-
tato, naturale, non ci sembra possibile che il tempo scorra in modo diverso
a seconda del sistema di riferimento in cui ci troviamo; eppure vedremo
che all’inizio del ventesimo secolo Albert Einstein dimostrò, ed in modo
convincente, che il tempo dipende dal sistema di riferimento in cui lo si
misura! Ricordando che stiamo trattando solo di moti rettilinei uniformi e
che dunque la velocità media coincide con la velocità istantanea, dal pun-
to di vista matematico è semplice dimostrare l’equazione (3.3.1) a partire
dall’equazione (3.2.2):

∆s
~ = ∆s
~ r + ∆s
~ t
∆s
~ ∆s
~ r ∆s ~ t
= +
∆t ∆t ∆t
~v = ~vr + ~vt

3.4 Legge di composizione delle accelerazioni

A questo punto capiamo in modo molto semplice che se il signor C, invece


che muoversi di moto rettilineo uniforme si muovesse di moto rettilineo
uniformemente accelerato, l’accelerazione misurata dal signor A e dal si-
gnor B sarebbe la stessa, visto che il treno non sta accelerando. Da un
punto di vista matematico si può semplicemente dimostrare dividendo
membro a membro l’equazione (3.3.1) per l’intervallo di tempo ∆t e ve-
dendo che il termine ~at sparisce in quanto l’accelerazione del treno è nulla.
Si ha quindi per le accelerazioni la seguente relazione:

~a = ~ar (3.4.1)

Questa relazione è di estrema importanza da un punto di vista concettua-


le perchè dimostra come le accelerazioni siano delle quantità invarianti
nella relatività galileiana, quantità cioè che rimangono sempre le stesse a
prescindere dal sistema di riferimento in cui vengono misurate. Essendo
invarianti le accelerazioni e concependo in modo naturale, così come per
il tempo, la massa come grandezza invariante, possiamo dire che anche le
forze sono grandezze invarianti per la relatività galileiana: le forze sono
dunque grandezze che non dipendono dal sistema di riferimento in cui
sono misurate purché i sistemi in questione siano inerziali. Ecco il motivo
concettuale per cui abbiamo sempre detto che i principi della dinamica
valgono solamente in sistemi di riferimento inerziali tra loro, e che se ci
troviamo in sistemi accelerati compaiono le cosiddette forze apparenti.

3.5 Trasformazioni di Galileo

Storicamente la relazione descritta dall’equazione (3.2.2) viene scritta in


termini di trasformazione matematica, dove le coordinate con apice sono
secondo il nostro schema di lavoro le coordinate misurate dal sistema di

Francesco Saitta, Pordenone Settembre 2019 60


Relatività Galileiana

riferimento in movimento, mentre le coordinate senza apice quelle mi-


surate dal sistema di riferimento in quiete. Il vettore ~v = (v x ; vy ; vz ) è
il vettore che precedentemente avevamo chiamato ~vt , la velocità con cui
si muove il sistema di riferimento in moto. Le seguenti equazioni sono
chiamate trasformazioni di Galileo:



 x0 = x − vx t
 y0

= y − vy t
(3.5.1)

 z0 = z − vz t
 t0

= t

È interessante notare come dal punto di vista matematico le equazioni


(3.5.1) siano semplicemente le equazioni di una traslazione di vettore A ~ =
~vt. L’ultima equazione spesso non è riportata, esprime la convinzione
comune che il tempo sia invariante per la relatività galileiana (si usa anche
dire invariante per trasformazioni di Galileo); la riportiamo qui perché
come detto precedentemente è uno dei paletti della fisica classica che verrà
abbattuto all’inizio del novecento dalle teorie di Einstein.

3.5.1 Invarianza delle lunghezze


Abbiamo già discusso dell’invarianza di massa, tempo, accelerazione e
forze per trasformazioni di Galileo, vogliamo qui dimostrare che anche le
lunghezze lo sono: sembrerebbe infatti decisamente strano che un certo ri-
ghello lungo ad esempio 10 centimetri in un sistema di riferimento in quie-
te possa diventare lungo 8 o 12 centrimetri in un sistema di riferimento in
moto!!! Anche questa è una certezza che lasceremo solamente in quinta
studiando la relatività di Einstein, per il momento la fisica classica ci assi-
cura anche questa invarianza, come vedremo in queste righe con qualche
semplice passaggio matematico. Supponiamo, nell’esempio del treno di
prima, di voler misurare la lunghezza di un righello all’interno del treno.
Il tizio B all’interno del treno semplicemente opererà la misura nel suo
sistema di riferimento, ottenendo L0 = x20 − x10 , dove x20 è la posizione mi-
surata di un estremo del righello nel sistema di riferimento in moto, x10 è la
posizione dell’altro estremo sempre nel sistema di riferimento in moto. Il
tizio fermo in stazione misurerà invece una lunghezza L = x2 − x1 , con x2
ed x1 le posizioni degli estremi del righello misurate nel sistema di riferi-
mento in quiete. Applicando le trasformazioni di Galileo possiamo vedere
con facilità che L0 = x20 − x10 = ( x2 − v x t) − ( x1 − v x t) = x2 − x1 = L, cioè
la lunghezza del righello è una grandezza invariante per trasformazioni di
Galileo, come volevamo dimostrare e come il nostro intuito ci suggerisce.

3.6 Principio di relatività galileiano

Galilei arriva ad intuire il principio di relatività meccanica, ovvero il fatto


che ogni fenomeno meccanico viene descritto nel medesimo modo in
sistemi di riferimento inerziali; riportiamo qui uno dei passi più famosi
delle opere del fisico pisano in cui mette in evidenza il principio appe-
na esposto:Rinserratevi con qualche amico nella maggiore stanza che sia sotto

Francesco Saitta, Pordenone Settembre 2019 61


Relatività Galileiana

coverta di alcun gran navilio, e quivi fate d’aver mosche, farfalle e simili anima-
letti volanti; siavi anco un gran vaso d’acqua, e dentrovi de’ pescetti; sospendasi
anco in alto qualche secchiello, che a goccia a goccia vada versando dell’acqua in
un altro vaso di angusta bocca, che sia posto a basso: e stando ferma la nave,
osservate diligentemente come quelli animaletti volanti con pari velocità vanno
verso tutte le parti della stanza; i pesci si vedranno andar notando indifferente-
mente per tutti i versi; le stille cadenti entreranno tutte nel vaso sottoposto; e voi,
gettando all’amico alcuna cosa non più gagliardamente la dovrete gettare verso
quella parte che verso questa, quando le lontananze siano eguali; e saltando voi,
come si dice, a pié giunti, eguali spazii passerete verso tutte le parti. Osservate
che avrete diligentemente tutte queste cose, benché niun dubbio ci sia che mentre
il vassello sta fermo non debbano succeder così, fate muover la nave con quanta
si voglia velocità; ché (pur che il moto sia uniforme e non fluttuante in qua e in
là) voi non riconoscerete una minima mutazione in tutti li nominati effetti, né
da alcuno di quelli potrete comprender se la nave cammina o pure sta ferma: voi
saltando passerete nel tavolato i medesimi spazii che prima, né, perché la nave si
muova velocissimamente, farete maggior salti verso la poppa che verso la prua,
benché, nel tempo che voi state in aria, il tavolato sottopostovi scorra verso la
parte contraria al vostro salto; e gettando alcuna cosa al compagno, non con più
forza bisognerà tirarla, per arrivarlo, se egli sarà verso la prua e voi verso poppa,
che se voi fuste situati per l’opposito; le gocciole cadranno come prima nel vaso
inferiore, senza caderne pur una verso poppa, benché, mentre la gocciola è per
aria, la nave scorra molti palmi (Galilei, 1632)È evidente come quindi secon-
do questo principio di relatività ogni legge della meccanica abbia sempre
la stessa forma in sistemi di riferimento inerziali, e come quindi non sia
possibile determinare, con esperimenti meccanici, se il sistema inerziale
in cui mi trovo sia in stato di quiete o di moto rettilineo uniforme: questi
sistemi sono tutti equivalenti tra loro. Questo punto è molto importante
in quanto spiega come mai, come studieremo nel corso dell’ultimo anno,
per cercare di trovare l’etere (il sistema di riferimento in quiete assoluta)
si fecero esperimenti di elettromagnetismo e non esperimenti meccanici.

Francesco Saitta, Pordenone Settembre 2019 62


Relatività Galileiana

Francesco Saitta, Pordenone Settembre 2019 63


Relatività Galileiana

3.7 Esercizi

1. Un sistema di riferimento S1 è spostato rispetto al sistema di riferimento S di un vettore d~ = (2; 3).


Quale sarà la posizione per un osservatore solidale con S di un punto materiale che per un osservatore
solidale con S1 ha coordinate P1 = (3; 1)?

[P1 = (5; 4)]


2. Un sistema di riferimento S1 è spostato rispetto al sistema di riferimento S di un vettore d~). Quale
sarà d~ se per un osservatore solidale con S la posizione di un punto materiale è P = (1; 1), mentre per
un osservatore solidale con S1 è P1 = (−5; 4)?

[d~ = (6; −3)]


3. Un sistema di riferimento S1 è spostato rispetto al sistema di riferimento S di un vettore ~v) di modulo
d = 5 ed angolo formato con l’asse delle ascisse di S α = π/6 rad. Quale sarà la posizione di un
punto materiale P, secondo un osservatore solidale con il sistema S, se per un osservatore solidale con
il sistema S1 il punto materiale ha coordinate P1 = (10, 5; 7, 3)?

[P = (14, 85; 9, 8)]


4. Un uomo A sta camminando alla velocità v A = 1 m/s sopra ad un treno, che passa alla velocità
v T = 120 km/h davanti ad una stazione, nella stessa direzione della velocità del treno. Un tizio sedu-
to sulla panchina della stazione che velocità misurerà per l’uomo A?

[v =123,6 km/h]
5. Rispetto al problema (4) che velocità calcolerebbe l’uomo seduto sulla panchina se il tizio A si muo-
vesse in direzione opposta alla velocità del treno?

[v =116,4 km/h]
6. Rispetto al problema (4) che velocità calcolerebbe l’uomo seduto sulla panchina se il tizio A si muo-
vesse in direzione perpendicolare alla velocità del treno?

[v =120,05 km/h]
7. Un ragazzo sta correndo in bicicletta alla velocità v = 20 km/h quando lascia cadere per sbaglio il
telefono cellulare da un’altezza h = 1, 2 m. Trascurando l’attrito dell’aria che moto descriverà per il
telefono un tizio seduto su una panchina che osserva la situazione? Che velocità del telefono misure-
rebbe un osservatore solidale con il tizio seduto sulla panchina un istante prima che il telefono tocchi
terra?

[v = 7, 4 m/s]
8. Se nel problema (7) il ragazzo in bicicletta stesse facendo un moto uniformemente accelerato con una
certa accelerazione a ed il telefono cadesse nell’istante in cui la bicicletta ha una velocità v0 = 20 km/h,
come cambierebbe il risultato?

[v = 7, 4 m/s]
9. Che moto deve fare un carrello a cui è appeso un pendolo affinché il pendolo sia in equilibrio forman-
do un angolo α = π/6 rispetto l’orizzontale?

[moto rettilineo uniformemente accelerato, a = 17 m/s2 ]

Francesco Saitta, Pordenone Settembre 2019 64


Relatività Galileiana

10. Un calciatore sta correndo incontro alla palla ad una velocità vG = 7 m/s; la direzione della corsa
del giocatore forma un angolo α = 135 con la direzione della palla, che si muove rasoterra. A che
velocità è stata lanciata la palla se il giocatore la vede avvicinarsi ad una velocità v P = 90 km/h?

[v = 19, 5 m/s]
11. Un ragazzino all’interno di un treno gioca a lanciare in verticale una pallina. Supponendo che un
osservatore solidale con il treno veda un moto di caduta di grave con velocità iniziale v0 = 6 m/s e
che un osservatore solidale con la banchina della stazione in cui sta passando il treno senza fermarsi
misuri un moto parabolico con velocità iniziale v = 50 m/s qual è la velocità del treno?

[v T = 178 km/h]
12. In una giornata di pioggia un osservatore A fermo sul ciglio della strada osserva le gocce cadere in
verticale; se un osservatore all’interno di un’automobile che viaggia ad una velocità v A = 90 km/h
vede la pioggia lasciare una traccia sul finestrino laterale con un angolo α = π/6 rispetto alla verti-
cale qual è la velocità della pioggia v A che osserva A? Quale la velocità v B misurata dall’osservatore
sull’auto? Si trascurino gli attriti delle gocce sul finestrino.

[v A = 43 m/s; v B = 50 m/s]
13. All’interno di un aereo per la simulazione di assenza di gravità, nella sua fase di caduta libera un
astronauta lancia una palla in orizzontale con velocità v0 = 5 m/s. Dopo quanto tempo toccherà
il pavimento dell’aereo? Che velocità della palla misura un osservatore solidale con la terra dopo 3
secondi dal lancio?

[mai (per lo meno finché l’aereo resta in caduta libera); v = 29, 8 m/s]
14. Si calcoli la velocità tangenziale della luna secondo un osservatore T solidale con la terra ed un os-
servatore S solidale con il sole nel momento in cui si ha l’allineamento Sole-Terra-Luna.

[v T = 948 m/s; vS = 30796 m/s]


15. Si risolva il problema (14) nel caso in cui l’allineamento sia del tipo Sole-Luna-Terra. rivoluzione
terrestre.

[v T = 948 m/s; vS = 28900 m/s]


16. Un operatore televisivo deve filmare la scena di un film in cui il protagonista corre ad una velocità
v = 5 m/s (!). L’attore protagonista non è evidentemente in grado di correre a quella velocità, ma
solamente ad una velocità massima vmax = 1, 5 m/s. Come può l’operatore realizzare comunque la
sua ripresa? Si spieghi il ragionamento fatto.

[facendo muovere la telecamera incontro all’attore ad una velocità di 3,5 m/s]


17. Due ragazzi, A e B stanno giocando a passarsi un pallone dentro ad una barca che si sta muovendo
ad una velocità v = 3 m/s. Essi distano tra loro d = 2 m, lanciano la palla e la ricevono entrambi ad
una altezza h = 1, 2 m, con un angolo di lancio e di ricezione α = π/3 A quale velocità deve lanciare
ognuno di loro (v A per A e v B per B) affinché il gioco riesca? Secondo l’osservatore a terra i due
ragazzi lanciano la palla con la stessa velocità? Che velocità misura un osservatore fermo a riva per il
lancio di A (v0A ) e per il lancio di B (v0B )? Si supponga che l’osservatore veda A lanciare nella direzione
del moto della barca e B in direzione opposta.

[v A = 4, 8 m/s; v B = 4, 8 m/s; v0A = 6, 8 m/s; v0B = 4, 2 m/s]

Francesco Saitta, Pordenone Settembre 2019 65


Relatività Galileiana

18. Un’automobile A si muove con una velocità v A = 25m/s in direzione N30°E, mentre un’automobile
B si muove con velocità v B = 30 m/s in direzione E60°S. Si calcoli il modulo della velocità relativa tra
le due automobili.

[vr = 48 m/s]
19. Durante una partita di calcio il portiere effettua una rimessa dal fondo imprimendo alla palla una
velocità iniziale v0 = 20 m/s formante un angolo α = 30 rispetto al suolo. Se un giocatore che sta
correndo parallelamente alla palla verso centrocampo misura una velocità della palla v = 12 m/s a
che velocità vG sta correndo il giocatore?

[vG = 10, 6 m/s]


20. Un oggetto viene fatto cadere dalla finestra di un palazzo con velocità iniziale v0 = 6, 5 m/s diretta
verso il basso in verticale da un’altezza h = 15 m. Un tizio A sta salendo con un ascensore a velocità
costante v = 1 m/s nel palazzo; ad un’altezza h = 10 m dal suolo vede passare l’oggetto di fronte a
sé: quale velocità misurerà A per l’oggetto in questione?

[v = 12, 8 m/s]
21. Da un carrello che si sta muovendo con velocità costante vC = 50 km/h viene lanciata una palla con
una velocità iniziale v0 = 3 m/s in direzione orizzontale opposta al senso di marcia del carrello ed
altezza iniziale h = 1, 5 m. Quale sarà la gittata Gt del lancio secondo un osservatore solidale con il
terreno e quale la gittata Gc secondo il lanciatore?

[Gt = 6, 0 m; Gc = 1, 6 m]
22. A quale velocità deve correre una persona per mantenere un pallone di massa m = 200 g in equilibrio
in verticale sul petto se il coefficiente di attrito tra la palla e la maglia è k = 3?

[Non importa la velocità, il moto deve essere accelerato con a = 3, 3 m/s2 ]


23. Quale deve essere la velocità v B di una barca che vuole muoversi in direzione Est-Ovest ad una velo-
cità v = 100 km/h se la corrente su cui si sta muovendo è caratterizzata da una velocità vC = 40 m/s
in direzione Nord-Sud?

[v B = 48, 8 m/s in direzione N(0,6rad)O]


24. Un aereo si sta muovendo ad una velocità v = 500 km/h parallela al suolo quando entra in una zona
con una corrente ascensionale di velocità vC = 100 km/h. Quale sarà la velocità dell’aereo rispetto al
suolo all’interno della corrente?

[v = 510 km/h con un angolo α = 11 rispetto l’orizzontale]


25. Si consideri un moto parabolico caratterizzato da una velocità iniziale v0 = 20 m/s che forma un
angolo α = 45 rispetto l’orizzontale. A quale velocità si deve muovere un osservatore A, rispetto al
sistema in cui avviene il moto parabolico, per osservare un moto di caduta di grave?

[v A = 14 m/s nella stessa direzione della velocità orizzontale iniziale del moto parabolico]
26. Find the relative speed of two point particles A and B with velocities ~v A = 2x̂ + 3ŷ (m/s) and
~v B = −2x̂ + 5ŷ (m/s) in a given reference frame Oxy .

[vr = 4, 5 m/s]

Francesco Saitta, Pordenone Settembre 2019 66


Relatività Galileiana

27. In a given reference frame Oxy a point particle A is moving with a velocity ~v A = 4x̂ − 3ŷ (m/s). The
relative velocity of a point particle B with respect to A is ~vr = 2x̂ + 2ŷ (m/s). Find v B .

[~v B = 6x̂ − 1ŷ (m/s)]


28. Find the speed of an airplane if the velocity measured from the earth is v = 300 km/h with an angle
α = 30 with respect to the horizontal and the airplane is flying in an area characterized by a vertical
wind with vW = 80 km/h.

[v A = 269 km/h]
29. A runner A is training and moves with a speed v A = 3 m/s when another runner, B, passes him: in
10 seconds B moves from 5 m before A up to 3 m after A. What is the velocity of B with respect to
another guy C sitting on a bench and looking to the scene?

[v B = 3, 8 m/s]
30. A guy A is moving with a speed v A = 2 m/s inside a train A (v TA = 70 km/h) in the same direction
of the train A. A guy B is moving with a speed v B = 3 m/s inside another train B (v TB = 50 km/h)
in the opposite direction of the train B. What is the speed of the guy B measured by the guy A if the
two trains are moving one towards the other?

[v = 32, 2 m/s]

Francesco Saitta, Pordenone Settembre 2019 67


4 Quantità di moto ed urti

In questo capitolo introdurremo una nuova grandezza fisica: la quantità


di moto di un punto materiale; attraverso questa grandezza arriveremo
ad una formulazione più generale, rispetto a quella vista nel capitolo 2,
delle leggi della dinamica. Tale formulazione ci permetterà una più sem-
plice descrizione di fenomeni complessi quali sistemi a massa variabile,
forze impulsive (che agiscono in un intervallo di tempo molto breve) o
urti tra punti materiali. La necessità dell’introduzione di una grandez-
za simile Risulta evidente dalla difficoltà di descrivere dal punto di vista
Newtoniano molte situazioni concrete, come ad esempio:

• La salita di una mongolfiera inizialmente in equilibrio ad una certa


altezza per effetto del rilascio di una certa quantità di zavorra.

• Il rimbalzo di una palla contro un muro o la battuta di una pallina


durante una partita di tennis.

• L’efficacia di un colpo secco di martello rispetto all’applicazione costan-


te di una certa forza per infilare un chiodo in un pezzo di legno.

A tal fine definiamo la seguente grandezza vettoriale:

~q = m~v, (4.0.1)

che chiamiamo quantità di moto e che misureremo nel sistema internazio-


nale in kg·m/s o N·s.

4.1 Teorema dell’impulso

Il teorema dell’impulso dice che l’impulso di una forza, ovvero il prodotto


tra la forza media ~Fm applicata su un punto materiale in un intervallo di tempo
∆t, è uguale alla variazione della quantità di moto del punto materiale stesso in
quell’intervallo di tempo. Ovvero:

~Fm ∆t = ∆q
~ (4.1.1)

Per darci ragione del teorema immaginiamo una forza variabile che agisca
su un punto materiale per un certo intervallo di tempo: ad esempio pos-
siamo visualizzare un colpo di un tennista sulla pallina da tennis. La forza
agente sul punto materiale avrà in generale un andamento temporale del
tipo illustrato in figura (4.1), concentrata nell’istante t0 ma diversa da ze-
ro lungo tutto l’intervallo ∆t. Possiamo quindi operare con il secondo

Francesco Saitta, Pordenone Settembre 2019 69


Quantità di moto ed urti

Figura 4.1: Forza agente sul


punto materiale in ∆t

principio della dinamica nel seguente modo:

~Fm = m~am
~Fm ∆v
~
= m
∆t
v~f − ~vi
~Fm = m
∆t
mv~f − m~vi
~Fm =
∆t
~Fm ∆q
~
=
∆t

da cui si ottiene facilmente il teorema (4.1.1). Se volessimo considerare un


intervallo di tempo infinitamente piccolo (o quantomeno molto piccolo
rispetto al fenomeno che stiamo considerando) otterremmo, con la solita
convenzione introdotta nel primo capitolo, la relazione:

~F = d~q , (4.1.2)
dt

in cui questa volta arriviamo a definire la forza istantanea come il rapporto


tra la variazione della quantità di moto e l’intervallo di tempo infinitesimo
in cui la variazione stessa è avvenuta. Come per il secondo principio
della dinamica questo teorema parla di cause ed effetti: la relazione tra la
causa forza ed il suo effetto variazione della quantità di moto. Riapplicando
il ragionamento fatto per il lavoro e descritto nella figura (2.16) possiamo
dire che la variazione di quantità di moto totale in figura (4.1) è data
dall’area tra la curva F (t) e l’asse dei tempi1 . 1
in questo testo sottolineiamo
continuamente le relazioni geo-
metriche tra quantità fisiche ed
aree o curve tangenti. Questo
4.2 Teorema di conservazione della quantità di moto affinché, una volta che nel cor-
so di matematica si affronteran-
Dal teorema dell’impulso è facile dedurre che se la somma tutte le forze no i concetti di derivata ed in-
tegrale definito, si possano co-
agenti su un punto materiale è nulla allora sarà nulla la variazione di gliere le diverse applicazioni ad
quantità di moto del punto materiale stesso, ovvero la quantità di moto è argomenti già svolti di fisica
costante. In sistemi isolati la quantità di moto si conserva è l’usuale enunciato

Francesco Saitta, Pordenone Settembre 2019 70


Quantità di moto ed urti

di questo teorema, che matematicamente si esprime come:

~F = ~0 ⇐⇒ ~q = ~k (4.2.1)

Naturalmente considerando un sistema formato da un insieme di punti


materiali possiamo considerare come quantità di moto totale del sistema
la somma delle singole quantità di moto dei diversi punti materiali ed
applicare il teorema ad un qualsiasi sistema di punti materiali. Come
ogni teorema di conservazione in fisica la conservazione della quantità di
moto è importante dal punto di vista concettuale per due motivi: in primo
luogo è una caratterizzazione dei sistemi fisici ed in secondo luogo ci aiuta
a risolvere problemi in sistemi isolati. Al momento quindi possiamo dire
con certezza che:

• In un sistema chiuso si conserva la massa;

• In un sistema conservativo si conserva l’energia meccanica;

• In un sistema isolato si conserva la quantità di moto.

Più quantità che si conservano vengono trovate più si può conoscere nel
dettaglio fisico il sistema stesso, potendolo descrivere e potendo predi-
re la sua evoluzione sulla base delle informazioni date dalle quantità
conservate.

4.3 Descartes e Leibniz: dibattito sui principi di conservazione

Nei secoli dello sviluppo della fisica classica ebbero grande risalto due
scienziati, Renè Descartes e Gottfried Wilhelm Leibniz . In La Haye en Touraine, 1596 -
questo paragrafo approfondiremo le idee di questi due scienziati rispetto Stoccarda, 1650
Lipsia, 1646 - Hannover, 1716
alla quantità di moto; è interessante vedere come queste due grandi menti
della scienza ebbero idee molto diverse tra loro anche per renderci conto
di come, nella storia della fisica, le conclusioni che oggi studiamo in pochi
giorni sono in realtà frutto di processi molto lunghi e complessi. Ci sof-
fermeremo soprattutto sul concetto di conservazione di quantità di moto
visto dal punto di vista dei due scienziati, ed in particolare studieremo il
ragionamento che portò Leibniz a scrivere un saggio comparso sugli Acta
Eruditorum del 1686 (Leibniz, 1863) intitolato Brevis denmostratio erroris me-
morabilis Cartesii et aliorum circa legem naturalem (Breve dimostrazione di un
errore memorabile di Cartesio e di altri riguardo una legge naturale). Sia
Descartes che Leibniz colsero l’importanza di trovare qualche grandezza
fisica che risultasse invariante, cioè che rimanesse immutata qualunque fe-
nomeno fisico possa avvenire nell’universo: da un punto di vista filosofico
questa ricerca è giustificata dalla convinzione che l’universo debba avere
delle regole scientifiche all’interno delle quali tutto possa avvenire e della
ricerca di ciò che genera il moto dei corpi, dal punto di vista pratico le
quantità invarianti (come abbiamo visto nel caso dell’energia) ci permet-
tono di catalogare sistemi a seconda del valore che assume l’invariante e
di risolvere problemi in modo semplice.

• Il punto di vista di Descartes:


Descartes e gli studiosi a lui vicini (i cartesiani) pensavano che la quan-

Francesco Saitta, Pordenone Settembre 2019 71


Quantità di moto ed urti

tità di moto fosse un ”invariante universale”: essi identificano il pro-


dotto tra massa e velocità di una particella con la forza motrice, ciò che
permette il moto nell’universo. Ecco qui di seguito il pensiero di De-
scartes in proposito : (Descartes, 1644)

“Dopo aver esaminato la natura del movimento bisogna che ne consideria-


mo la causa, e poiché essa può essere presa in due maniere, cominceremo dalla
prima e più universale, che produce generalmente tutti i movimenti che sono
al mondo; considereremo in appresso l’altra la quale fa sì che ogni parte del-
la materia acquisti movimenti che non aveva prima. Per quanto riguarda la
prima mi sembra evidente che non ce n’è altra che Dio, che per sua onnipo-
tenza ha creato la materia con il movimento e il riposo, e che conserva adesso
nell’universo, col suo concorso ordinario, tanto movimento o riposo quanto ce
n’ha messo creandolo.Poiché sebbene il movimento non sia che un mo-
do nella materia che è mossa, essa ne ha pertanto una certa quantità
che non aumenta e non diminuisce mai, benché ce ne sia ora più e
ora meno in alcune delle sue parti. Ecco perché quando una parte
della materia si muove due volte più presto di un’altra, e questa è
due volte maggiore della prima noi dobbiamo pensare che c’è tanto
movimento nella più piccola che nella maggiore, e che tutte quante le
volte il movimento di una parte diminuisce, quello di qualche altra
parte aumenta in proporzione. Noi conosciamo anche che è una perfezione
in Dio non solamente di essere immutabile nella sua natura, ma anche di agire
in un modo che non cambia mai, tanto che, oltre i cambiamenti che vediamo nel
mondo e quelli cui crediamo perché Dio li ha rivelati, e che sappiamo accadere
o essere accaduti nella natura senza alcun cambiamento da parte del Creatore,
non ne dobbiamo supporre altri nelle sue opere per paura di attribuirgli inco-
stanza. Da cui segue che poiché egli ha mosso in molte maniere differenti le
parti della materia, quando le ha create, e le mantiene tutte nella stessa maniera
e con le stesse leggi ch’egli ha potuto osservare loro nella creazione, conserva
incessantemente in questa materia un’uguale quantità di movimen-
to.”

Alla luce delle nostre conoscenze notiamo come la conservazione della


quantità di moto secondo Descartes non fosse un concetto completa-
mente errato, l’errore fatto dal grande scienziato fu quello di assolu-
tizzare questa conservazione: sappiamo infatti al giorno d’oggi - e ne
abbiamo visto la dimostrazione in classe - come la quantità di moto si
conservi, ma solamente in sistemi isolati.

• Il punto di vista di Leibniz:


Leibniz comprese come nel ragionamento di Descartes ci fossero degli
errori dal punto di vista fisico-matematico. Egli dimostrò come nell’u-
niverso la quantità che si conserva sempre sia un’altra quantità: mv2 (
che assimilò ad mh). Seppur in modo non analitico, considerando sola-
mente un esempio meccanico, possiamo trovare nelle righe di Leibniz
l’idea moderna di conservazione dell’energia. Qui di seguito il brano
tratto dagli scritti di Leibnitz : (Leibniz, 1863)

Francesco Saitta, Pordenone Settembre 2019 72


Quantità di moto ed urti

“Poiché molti matematici vedono nelle cinque macchine semplici che la ve-
locità e la mole (massa) sono tra loro compensate, generalmente essi valutano
la forza motrice tramite la quantità di moto, ossia tramite il prodotto della
moltiplicazione del corpo per la sua velocità. Oppure, per parlare più geome-
tricamente, essi affermano che le forze di due corpi (della stessa specie) che si
urtano nel movimento e che agiscono parimente mediante la loro massa e il
movimento sono in ragione composta dei corpi, o delle masse, e delle velocità
che hanno. E così poiché è conforme alla ragione che la stessa somma della
potenza motrice è conservata in natura, e non è diminuita, in quanto vediamo
che non è persa alcuna forza da un corpo che non sia trasferita in un altro;
né aumentata perché ancora il moto perpetuo meccanico non avviene mai, per
il fatto che nessuna macchina e di conseguenza neanche il mondo intero può
aumentare la sua forza senza un nuovo impulso esterno; da qui Cartesio, che
considerava equivalente la forza motrice e la quantità di moto, ha affermato che
la stessa quantità di moto è conservata da Dio nell’universo. Io, certamente,
per mostrare quanto si trovi tra queste due, suppongo anzitutto che un corpo,
cadendo da una certa altezza, acquista una forza fino a rialzarsi di nuovo, se
la sua direzione così lo conduce né qualcosa d’esterno l’impedisce: per esempio
un pendolo ritornerà precisamente all’altezza da cui è caduto, a meno che la
resistenza dell’aria e altri impedimenti simili molto piccoli non assorbano un
po’ della sua forza, dai quali noi ora facciamo astrazione. Suppongo anche, in
secondo luogo, che è necessaria una forza tanto grande per sollevare il corpo A
di una libbra fino all’altezza CD di quattro braccia di quella che è necessaria
per sollevare il corpo B di quattro libbre fino all’altezza EF di un braccio. Tutte
queste cose sono ammesse ugualmente dai cartesiani e dagli altri filosofi e mate-
matici dei nostri tempi. Segue da qui che il corpo A lasciato cadere dall’altezza
CD ha acquisito precisamente altrettanta forza del corpo B lasciato cadere dal-
l’altezza EF. Infatti, il corpo A dopo esser stato lanciato da C arriva in D,e là
esso possiede la forza di risollevarsi fino a C, per la prima ipotesi, ossia la forza
di sollevare un corpo di una libbra (vale a dire il proprio corpo) fino all’altezza
di quattro braccia. E parimenti dopo che il corpo B è pervenuto per la caduta
da E in F, dove esso ha la forza di risalire fino ad E, per la prima ipotesi, ossia
la forza di sollevare un corpo di quattro libbre (vale a dire il proprio corpo) fino
all’altezza di un braccio. Pertanto, per la seconda ipotesi, la forza del corpo A
che si trova in D e la forza del corpo B che si trova in E sono uguali. Vediamo
ora se la quantità di moto è la stessa in entrambi i casi. In verità, il più gran-
de disaccordo sarà trovato lì contro ogni speranza. Il che io mostro nel modo
seguente. È stato dimostrato da Galileo che la velocità acquisita mediante la
caduta CD è il doppio della velocità acquisita mediante la caduta EF. Molti-
plichiamo quindi il corpo A che è come 1 per la sua velocità che è come 2, il
prodotto o quantità di moto sarà come 2; moltiplichiamo di nuovo il corpo B
che è come 4 per la sua velocità che è come 1, il prodotto o quantità di moto sarà
come 4.Pertanto la quantità di moto che è del corpo A che si trova in D
è la metà della quantità di moto che è del corpo B che si trova in F, e
tuttavia poco prima le forze sono state trovate uguali. E pertanto esi-
ste una grande differenza tra la forza motrice e la quantità di moto, di
tal sorta che l’una non potrebbe essere calcolata tramite l’altra, come
abbiamo inteso dimostrare. Risulta da ciò come la forza dovrebbe essere
misurata dalla quantità dell’effetto che può produrre, per esempio dall’altezza

Francesco Saitta, Pordenone Settembre 2019 73


Quantità di moto ed urti

a cui precisamente può sollevare un corpo di grandezza e specie date, non di


certo dalla velocità che può imprimere a un corpo. Infatti, non è necessaria una
forza doppia, ma una più grande per dare al corpo una velocità doppia. Nessu-
no sicuramente si meraviglierà che nelle macchine semplici, la leva, l’asse della
ruota, la puleggia, il cuneo, la vite d’Archimede, e simili, c’è equilibrio quan-
do la grandezza di un corpo è compensata dalla velocità dell’altro che nascerà
secondo la disposizione della macchina; o quando le grandezze (essendo data
la stessa specie di corpi) stanno reciprocamente come le velocità; o quando la
stessa quantità di moto si produrrà dall’uno o dall’altro. Infatti deriva ancora
da qui che la quantità dell’effetto dovrebbe essere la stessa in entrambi i casi,
ovvero l’altezza di discesa o di salita in un qualunque lato dell’equilibrio che
tu desideri che il moto sia fatto. È quindi accidentale che la forza possa essere
misurata dalla quantità di moto. Di sicuro si danno altri casi, tale è quello
che abbiamo riportato qui dove esse non coincidono. D’altronde poiché nulla è
più semplice della nostra dimostrazione, è sorprendente che non sia venuta in
mente a Cartesio o ai cartesiani, uomini molto dotti. Ma di sicuro la troppa
confidenza nel suo spirito l’ha deviato dal cammino. Infatti Descartes, per il
difetto comune ai grandi uomini fu reso alla fine un po’ troppo importante.
D’altra parte, temo che non pochi cartesiani comincino a imitare i peripatetici
di cui si burlano, ed è perché hanno l’abitudine di consultare i libri del mae-
stro anziché la retta ragione e la natura delle cose. Si deve dunque dire
che le forze sono in ragione composta dei corpi (della gravità stessa
o della solidità) e delle altezze che producono la velocità, vale a dire
di quelle mediante le quali tali velocità potrebbero essere acquisite
cadendo, o più generalmente (poiché talvolta nessuna velocità è stata
ancora prodotta) delle altezze sul punto di produrre: non in verità
generalmente delle velocità stesse, di qualche maniera che ciò sia plausi-
bile in prima approssimazione e sia constatato dalla maggior parte, e da qui
nacquero molti errori che sono tangibili negli scritti matematico-meccanici dei
reverendi padri Honoré Fabri e Claude Dechales e anche in Giovanni Alfonso
Borelli e in altri, per il resto eminenti in questi studi. E penso anche che di-
penda da ciò se recentemente la regola di Huygens sul centro di oscillazione
dei pendoli, che è vera, è stata revocata in dubbio da alcuni dotti uomini.”

4.4 Urti

Uno dei fenomeni descrivibili in modo semplice con il teorema dell’im-


pulso anziché la seconda legge della dinamica è sicuramente l’urto tra due
punti materiali, ovvero ciò che succede quando le traiettorie del moto di
due punti materiali si incontrano in un dato istante di tempo. Sappiamo
che due corpi non possono occupare un certo volume nello stesso istan-
te e dunque qualcosa deve accadere: i punti materiali restano attaccati
l’uno con l’altro, rimbalzano, cambiano le loro traiettorie,... sono certa-
mente fenomeni con cui abbiamo avuto molte volte a che fare: il lancio di
due palline l’una contro l’altra, il gioco del biliardo, uno scontro tra due
automobili, il lancio di una pallina contro ad un muro,... È importante
notare come le forze interne al sistema dei punti materiali che si scontra-
no sono sempre molto maggiori di tutte le forze esterne presenti (gravità
compresa), per lo meno nell’intervallo di tempo molto piccolo in cui l’urto

Francesco Saitta, Pordenone Settembre 2019 74


Quantità di moto ed urti

avviene; il sistema durante un urto è quindi sempre isolato, pertanto la


quantità di moto totale di un sistema durante un urto si conserva sem-
pre, se si considera un intervallo di tempo molto piccolo. Classificheremo
gli urti quindi non sulla base della conservazione o meno della quantità
di moto, ma sulla base della conservazione o meno dell’energia cinetica:

• Urti elastici: in cui l’energia cinetica si conserva;

• Urti anelastici: in cui l’energia cinetica non si conserva;

• Urti completamente anelastici: urti anelastici in cui i due punti mate-


riali restano uniti dopo l’urto stesso.

Consideriamo solo l’energia cinetica perché considerando il sistema isola-


to (per lo meno in un piccolo intervallo di tempo attorno all’urto stesso)
non sono presenti forze che giustifichino la presenza di energie poten-
ziali. In generale quindi possiamo dire che un urto viene descritto dalle
seguenti equazioni:
(
~qi = ~q f
(4.4.1)
Ei = E f + ∆E

dove ~qi è il vettore quantità di moto iniziale del sistema, ~q f è il vettore


quantità di moto finale del sistema, Ei è l’energia cinetica iniziale del si-
stema, E f è l’energia cinetica finale del sistema e ∆E l’eventuale energia
persa durante l’urto. Il caso più comune di urto anelastico è anche il più
complicato, da risolvere volta per volta affrontando il sistema (4.4.1). Con-
sidereremo qui di seguito alcuni esempi semplici di urti in una o due di-
mensioni, le cui soluzioni possono essere trovate in generale senza troppe
complicazioni matematiche.

4.4.1 Urti completamente anelastici in una, due o tre dimensioni


Gli urti completamente anelastici sono sicuramente gli urti più semplici
da descrivere in termini matematici. Consideriamo due punti materiali
di masse m1 ed m2 , velocità ~v1 e ~v2 che si urtino tra loro in modo da
restare poi attaccati l’uno all’altro: un particolare colpo di biliardo può
esemplificare bene questa situazione in due dimensioni, come mostrato in
Figura 4.2: Urto completamente
figura (4.2). Il sistema (4.4.1) diventa quindi anelastico
(
m1~v1 + m2~v2 = (m1 + m2 )~v f
1 2 1 2
2 m1 v1 + 2 m2 v2 = 12 (m1 + m2 )v2f + ∆E

m1~v1 +m2~v2
(
~v f = m1 + m2
∆E = 1 2 1 2
2 1 v1 + 2 m2 v2
m − 12 (m1 + m2 )v2f

In generale possiamo quindi dire che, per un urto completamente anela-


stico si ha:

 ~v
f = ∑∑i mmi~vi i
i (4.4.2)
 ∆E = 21 ∑i mi v2i − 21 ∑i mi v2f

Francesco Saitta, Pordenone Settembre 2019 75


Quantità di moto ed urti

In un urto completamente anelastico quindi, date le masse ed i vettori


velocità iniziali, possiamo sempre trovare in modo piuttosto semplice (ap-
plicando le relazioni descritte in (4.4.2) ) la velocità finale dell’insieme dei
punti materiali e la perdita di energia cinetica.

4.4.2 Urti elastici in una dimensione


Gli urti elastici, o in generale urti in cui i punti materiali non rimangono
attaccati sono in generale più difficili da risolvere, se risolvere significa
predire le velocità finali dei diversi punti materiali conoscendone le ve-
locità iniziali, in quanto ci sono molte variabili incognite. Nel caso uni-
dimensionale, in mancanza di ulteriori dati oltre alle masse e le velocità
iniziali, l’unico caso risolvibile analiticamente è il caso dell’urto tra due
punti materiali: possiamo pensare nuovamente ad un colpo di biliardo in
cui le due palline però si muovono nella stessa direzione, come in figura
(4.3).
Il sistema di equazioni da risolvere in questo caso sarà:
(
m1 v1 + m2 v2 = m1 v10 + m2 v20
1 2 1 2 1 02 1 02
2 m1 v1 + 2 m2 v2 = 2 m1 v1 + 2 m2 v2

dove v0 sono le velocità finali. Matematicamente il problema si riduce alla


Figura 4.3: Urto elastico in una
risoluzione di un sistema di due equazioni e due incognite di secondo dimensione
grado, che possiamo risolvere come esercizio. Il risultato , scartate le
soluzioni non accettabili fisicamente, è dato da:

 v0 (m1 −m2 )v1 +2m2 v2
1 = m1 + m2
(m2 −m1 )v2 +2m1 v1 (4.4.3)
 v0 =
2 m1 + m2

Nel caso degli urti elastici in una dimensione è interessante notare alcuni
casi particolari, che si invita a studiare come esercizio:

• il caso di masse uguali m2 = m1 = m,

• il caso in cui la velocità iniziale di uno dei due punti materiali sia nulla,

• il caso in cui la velocità iniziale di uno dei due punti materiali sia nulla
e la sua massa sia enormemente più grande dell’altra (l’urto di una
pallina contro ad un muro).

4.4.3 Urti elastici in due dimensioni


Il caso di urti elastici in due dimensioni, anche se coinvolgenti solo due
punti materiali non è sempre risolvibile conoscendo solo le due masse e
le due velocità iniziali:
(
m1~v1 + m2~v2 = m1~v10 + m2~v20
1 2 1 2 1 02 1 02 (4.4.4)
2 m1 v1 + 2 m2 v2 = 2 m1 v1 + 2 m2 v2

considerando infatti le componenti dei vettori questo sistema diventa un


sistema di tre equazioni e quattro incognite. Esiste però un caso partico-
lare su cui si può dire qualcosa : il caso in cui m1 = m2 = m e v2 = 0. In

Francesco Saitta, Pordenone Settembre 2019 76


Quantità di moto ed urti

questo caso particolare infatti il sistema (4.4.4) diventa:

(
~v1 = ~v10 + ~v20
0 0 (4.4.5)
v21 = v12 + v22

Le due equazioni ci assicurano che i due vettori ~v10 e ~v20 debbano essere
perpendicolari tra loro: infatti la prima equazione ci garantisce che i tre
vettori in questione formino un triangolo mentre la seconda è il teorema
di Pitagora sui lati del triangolo, applicabile come sappiamo solo nel caso
in cui il triangolo sia rettangolo ed i cateti siano ~v10 e ~v20 ! Per risolvere il
problema analiticamente trovando le due velocità serve senz’altro un’altra
informazione, ma per lo meno si può dire qualcosa sulla situazione finale.

4.5 Centro di massa e moto di sistemi di particelle

Come si può notare dal sistema generale di equazioni (4.4.1) i casi con
più di due particelle possono essere molto complicati dal punto di vista
matematico, anche conoscendo alcuni parametri della situazione finale.
In questo paragrafo studieremo il moto del centro di massa di un sistema
di particelle come sintesi dell’evoluzione del sistema stesso. In molti casi
complessi questo può essere utile per capire l’evoluzione del sistema o
parte di esso, soprattutto quando è impossibile descrivere la traiettoria
dopo l’urto di ogni singola particella che compone il sistema. Ricordiamo
che il centro di massa di un sistema di particelle è definito come il punto
in cui possiamo pensare sia applicata la risultante delle forze peso di tutte
le particelle, o dal punto di vista matematico la media, pesata sulle masse,
delle posizioni di ogni singolo punto materiale:

m1~r1 + m2~r2 + · · · + mn~rn ∑n m ~r ∑n m ~r


~rcm = = i=n 1 i i = i=1 i i (4.5.1)
m1 + m2 + · · · + m n ∑ i =1 m i M

dove il sistema ha n punti materiali ed M è la massa totale del siste-


ma. L’equazione (4.5.1) può anche essere scritta secondo le componenti
cartesiane:

∑in=1 mi xi

 xcm =

 M
∑in=1 mi yi
ycm = M
(4.5.2)
∑in=1 mi zi


 z =
cm M

Ora, immaginando che questi punti materiali si stiano muovendo, ognuno


con una certa velocità, andiamo a calcolare qual è la velocità media del

Francesco Saitta, Pordenone Settembre 2019 77


Quantità di moto ed urti

centro di massa tra due istanti di tempo t1 e t2 :

~rcm (t2 ) −~rcm (t1 )


~vcm =
t2 − t1
∑in=1 mi~ri (t2 ) n
− ∑i=1 Mi i 1
m ~r (t )
M
=
t2 − t1
n
∑i=1 i i (t2 ) − ∑in=1 mi~ri (t1 )
m ~
r
=
M ( t2 − t1 )
n
∑i=1 (mi~ri (t2 ) − mi~ri (t2 ))
=
M ( t2 − t1 )
n
∑ mi ∆r
i =1
~ i
=
M ( t2 − t1 )
∆r
~
∑in=1 mi (t2 −it1 )
=
M
∑in=1 mi vi
=
M
Ovvero per le velocità medie, e quindi anche per quelle istantanee va-
le il fatto che la velocità del centro di massa è la media, pesata con le
masse, delle velocità di tutte le singole particelle componenti il sistema.
Analogamente possiamo anche concludere che

∑in=1 mi~ai
~acm =
M
Riusciamo quindi a definire la cinematica del centro di massa a partire
dalla cinematica dei punti materiali componenti il sistema:

∑in=1 mi~ri
~rcm = (4.5.3)
M
∑in=1 mi~vi
~vcm = (4.5.4)
M
∑in=1 mi~ai
~acm = (4.5.5)
M
(4.5.6)

In particolare possiamo riscrivere la seconda equazione (4.5.4) nel seguen-


te modo:
n
~qcm = M~vcm = ∑ mi~vi = ~qi (4.5.7)
i =1

e la terza (4.5.5) come:

n
M~acm = ∑ mi~ai = ~R, (4.5.8)
i =1

con ~qi la quantità di moto iniziale del sistema e ~R la risultante di tutte


le forze agenti sul sistema. Possiamo quindi concludere che per il centro
di massa valgono le stesse leggi della cinematica e della dinamica che
abbiamo visto fin’ora per un sistema con un punto materiale soltanto:
il centro di massa è un punto che riassume in sé tutta la fisica di un
sistema di particelle.

Francesco Saitta, Pordenone Settembre 2019 78


Quantità di moto ed urti

Primo principio della dinamica e conservazione della quantità di moto In un


sistema di particelle isolato la quantità di moto del centro di massa si
conserva, esso è quindi in equilibrio ovvero persiste nel suo stato di quiete
o di moto rettilineo uniforme!

Secondo principio della dinamica e teorema dell’impulso La risultante delle


forze agenti sul sistema è uguale alla massa totale del sistema moltiplicata
per il vettore accelerazione del centro di massa ovvero la forza risultante
che agisce sul sistema è causa della variazione di quantità di moto del
centro di massa del sistema!

In altre parole, non importa quanti e quali urti facciano tra loro le parti-
celle componenti il sistema, il centro di massa continuerà fare il suo moto,
in equilibrio se il sistema è isolato, secondo le leggi del moto derivanti
dalle forze agenti se il sistema non è isolato. Gli esempi e le applicazio-
ni di questa analisi sono molte e molto utilizzate: possiamo pensare ad
esempio di voler descrivere il moto del centro di massa di un fuoco d’arti-
ficio (che continua il suo moto parabolico indipendentemente dal numero
di frammenti in cui si spezza - i frammenti anzi sono vincolati a spostarsi
facendo in modo che il centro di massa continui il moto parabolico) o la
descrizione degli urti tra particelle della fisica nucleare e la conseguente
cinematica delle particelle dopo l’urto all’interno dei grandi acceleratori
di particelle nel mondo. Dopo le analisi fatte in questo paragrafo, il siste-
ma di riferimento del centro di massa di un sistema di particelle diventa
un luogo privilegiato da cui descrivere un sistema di punti materiali.

Francesco Saitta, Pordenone Settembre 2019 79


Quantità di moto ed urti

Francesco Saitta, Pordenone Settembre 2019 80


Quantità di moto ed urti

4.6 Esercizi

1. Un giocatore di tennis imprime una forza media Fm = 50 N su di una pallina (m = 57 g) che arriva
alla racchetta con una velocità vi = 70 km/h. Supponendo che l’impatto tra racchetta e pallina duri
0,1 secondi, quale sarebbe la velocità della pallina dopo il colpo se non ci fosse deformazione della
pallina durante l’urto?

[v f = 245, 8 km/h]
2. Si calcoli la forza media che un pugile deve imprimere ad un sacco da 30 kg affinché esso si muova
con una velocità v = 2 m/s, supponendo che l’impatto tra guanto e sacco duri 0,3 secondi.

[Fm = 200 N]
3. Si calcoli la variazione di quantità di moto di un sistema su cui agisce una forza media esterna esterna
Fm = 500 N per un decimo di minuto.

[∆q = 3000 kg m/s]


4. Un calciatore in grado di imprimere una forza massima F = 500 N ad una palla di massa m = 200 g
vuole far raggiungere alla palla una velocità v = 120 km/h. Quanto deve durare l’impatto tra piede e
pallone?

[∆t = 0, 01 s]
5. Un battitore di baseball riesce a colpire la palla lanciata dall’avversario. Supponendo che la palla di
massa m = 100 g sia arrivata in orizzontale con una velocità vi = 30 m/s, che l’impatto con la mazza
duri ∆t = 5 ms e che dopo l’impatto la pallina abbia una velocità v f = 50 m/s e formi un angolo
α = 30 rispetto l’orizzontale, qual è l’intensità della forza applicata dal battitore alla pallina?

[F = 1549 N]
6. Un carrello di massa M = 50 kg si sta muovendo orizzontalmente alla velocità costante v = 30 km/h.
Quale sarà la velocità finale del carrello se da esso cade verticalmente un sacchetto di massa m = 5 kg?

[v f = 9, 3 m/s]
7. Se dallo stesso carrello del problema (6) viene lanciata una palla da 1 kg in direzione opposta alla
marcia del carrello con velocità v p = 20 km/h, quale sarà la velocità finale del carrello?

[v f = 8, 6 m/s]
8. Se dallo stesso carrello del problema (6) viene lanciata una palla da 1 kg nella stessa direzione della
marcia del carrello con velocità v p = 20 km/h, quale sarà la velocità finale del carrello?

[v f = 8, 4 m/s]
9. Antonio (A) lancia una palla, calciandola da terra con una velocità iniziale v0 = 10 m/s con un angolo
rispetto l’orizzontale α = π/3 rad, a Barbara (B) che si trova seduta su un carrello ad una altezza
20 cm da terra. Se la massa della palla è m = 200 g e la massa complessiva di carretto e Barbara è
M = 100 kg, si calcoli la velocità finale del carrello.

[v f = 10−2 m/s]
10. Un fuoco d’artificio di massa m viene lanciato in verticale. Nel punto più alto della sua traiettoria
il fuoco scoppia in 2 pezzi, il primo di massa m/3, il secondo di massa 2m/3. Si descriva il moto di

Francesco Saitta, Pordenone Settembre 2019 81


Quantità di moto ed urti

ogni singolo pezzo del fuoco d’artificio dopo lo scoppio, se il pezzo di massa minore appena dopo lo
scoppio ha una velocità v1 = 5 m/s.

[Ogni pezzo fa un moto parabolico con velocità iniziale orizzontale, il primo con v1 = 5 m/s, il
secondo con v2 = 2, 5 m/s]
11. Una cinquecento (m = 680 kg) tampona un camion (M = 5000 kg) alla velocità v = 60 km/h. Sup-
ponendo un urto completamente anelastico tra i due veicoli quale sarà la velocità finale del sistema
cinquecento/camion?

[v f = 7 km/h]
12. Si calcoli la variazione di energia cinetica del sistema nell’esercizio precedente.

[∆Ec = −83700 J]
13. Un placcaggio di rugby può essere considerato (se va a buon fine) come un urto completamente
anelastico. Si calcoli il modulo della velocità dei due giocatori appena dopo l’impatto se il primo
(m1 = 100 kg) si stava muovendo con una velocità v1 = 10 m/s formante un angolo α = π/3 rad con
la direzione della velocità secondo (m2 = 90 kg, v2 = 8 m/s).

[v f = 7, 9 m/s]
14. Un urto completamente anelastico è caratterizzato da una perdita di energia cinetica ∆Ec = 100 J. Se
l’urto è avvenuto tra due corpi di ugual massa, uno dei due inizialmente fermo e l’altro con quantità
di moto iniziale q1 = 50 kg m/s, quanto vale la velocità finale del sistema?

[v f = 4 m/s]
15. Due corpi di masse m1 = 5 kg e m2 = 10 kg si stanno muovendo con velocità di intensità v1 = 10
m/s e v2 = 8 m/s. Per quale angolo tra le due velocità si ha la massima dispersione di energia cinetica
durante l’urto? Ed in tal caso quanto vale questa dispersione?

[α = π rad, ∆Ec = −540 J]


16. Rispetto all’esercizio (7) si calcoli l’angolo per la dispersione minima di energia ed il valore di tale
dispersione.

[α = 0 rad, ∆Ec = −2, 3 J]


17. Due masse m1 = 10 kg ed m2 = 20 kg urtano in modo completamente anelastico avendo velocità
iniziali v1 = 5 m/s e v2 = 2, 5 m/s. Se la velocità finale del sistema ha modulo v f = 1 m/s quale era
l’angolo tra le velocità v~1 e v~2 ?

[α = 2, 5 rad]
18. Due palline di pongo fanno tra loro un urto completamente anelastico, la prima viene lanciata fron-
talmente contro la seconda con una velocità v1 = 5 m/s. Si misura come velocità finale delle due
palline v f = 2 m/s. Se le due palline hanno la medesima massa m = 250 g, qual è la velocità v2 della
seconda pallina?

[v2 = 1 m/s]
19. Due palline da biliardo, entrambe di massa m = 200 g si scontrano facendo un urto elastico. Suppo-
nendo che la prima si scontri con la seconda, inizialmente ferma, ad una velocità v1 = 3 m/s qual è la
velocità finale di ciascuna pallina?

Francesco Saitta, Pordenone Settembre 2019 82


Quantità di moto ed urti

[v1 f = 0 m/s; v2 f = 3 m/s]


20. Si risolva il problema (19) supponendo che m1 = 200 g ed m2 = 250 g.

[v1 f = −0, 3 m/s; v2 f = 2, 7 m/s, con segno positivo dato dal verso iniziale di v1 ]
21. Si risolva il problema (20) supponendo che la seconda pallina abbia una velocità iniziale v2 = 2 m/s
nella stessa direzione di ~v1 ma con verso opposto.

[v1 f = −2, 6 m/s; v2 f = 2, 4 m/s, con segno positivo dato dal verso iniziale di v1 ]
22. Tre palline (m1 = 2m2 = 4m3 = 250 g) sono disposte come in figura (4.4). Si trovino le velocità finali
delle tre palline supponendo che tutti gli urti siano elastici.

Figura 4.4: .

[v1 f = 3, 3 m/s; v2 f = 4, 4 m/s; v3 f = 17, 7 m/s]


23. Data la situazione in figura (4.5) si calcolino le velocità finali delle palline 1 e 2, assumendo tra loro
un urto elastico:

Figura 4.5: .

[v1 f = −6 m/s; v2 f = 4 m/s, con segno positivo dato dal verso iniziale di v1 ]
24. Data la situazione in figura (4.6) si trovi la distanza dal tavolino a cui cadono le due palline 1 e 2,
assumendo un urto elastico tra le due palline:

[d1 = 0, 25 m; d2 = 1, 15 m]
25. Una pallina viene lasciata cadere su un piano inclinato (α = π/3 rad rispetto l’orizzontale) senza
attrito da un’altezza h = 2 m. Supponendo che l’urto tra la pallina e il pavimento orizzontale una

Francesco Saitta, Pordenone Settembre 2019 83


Quantità di moto ed urti

Figura 4.6: .

volta arrivata in fondo al piano inclinato sia elastico, si calcoli a quale distanza dalla base del piano la
pallina toccherà terra di nuovo.

[d = 3, 5 m]
26. Si risolva il problema (25) ipotizzando un urto anelastico nel quale la pallina perde il 40% della sua
energia.

[d = 2, 1 m]
27. Due palle da biliardo m1 = 2 kg ed m2 = 5 kg di velocità iniziali v1 = 5 m/s e v2 = 8 m/s di verso
opposto si scontrano frontalmente. Quali sono le velocità finali se l’urto è anelastico e la perdita di
energia ammonta al 30% dell’energia iniziale?

[v1 f = 1, 6 m/s; v2 f = 11 m/s]


28. Una pallina viene sparata contro un bersaglio collegato ad una molla, come in figura (4.7). Si calcoli
la velocità iniziale del proiettile se la massima compressione della molla è di ∆ = 4 cm. Si consideri
l’urto tra pallina e molla come completamente anelastico.

Figura 4.7: .

[v p = 253 m/s]
29. Si risolva l’esercizio (28) nell’ipotesi in cui l’urto tra pallina e molla sia elastico.

[v p = 125 m/s]

Francesco Saitta, Pordenone Settembre 2019 84


Quantità di moto ed urti

30. Si risolva l’esercizio (28) nell’ipotesi in cui l’urto tra pallina e molla sia anelastico con una perdita di
energia del 20% rispetto l’energia iniziale.

[v p = 132 m/s]
31. Dato il doppio pendolo il figura (4.8) si calcoli l’altezza (distanza minima dal soffitto) raggiunta dalla
pallina m se l’urto tra le due palline è da considerarsi elastico.

Figura 4.8: .

[hm = 0, 3 m]
32. Si risponda alla domanda dell’esercizio (31) nel caso in cui l’urto tra le palline sia completamente
anelastico.

[hm = 1, 2 m]
33. Una massa M = 2 kg è appesa verticalmente ad una molla di costante elastica k = 1000 N/m. Una
freccetta (m = 50 g) viene lanciata verticalmente dal basso verso la massa. Supponendo che la freccetta
abbia una velocità v f = 3 m/s un istante prima di colpire la massa si calcoli la massima compressione
(rispetto alla sua posizione di riposo) della molla dopo l’urto, da considerarsi completamente anela-
stico.

[∆ = 18 mm]
34. Si consideri la situazione dell’esercizio (31) con il seguente cambiamento: la pallina m2 sia appog-
giata sul piano senza essere collegata al filo del pendolo. Si assuma il coefficiente d’attrito tra pallina
e pavimento k = 0, 2. Supponendo che l’urto tra le due palline sia un urto anelastico con energia
dispersa pari a ∆E = 10 J, si calcoli il lavoro fatto dalla forza d’attrito per fermare la pallina m2 . Si
calcoli inoltre in quanto tempo la pallina si ferma e quanto spazio percorre prima di fermarsi.

[t = 2, 2 s; s = 4, 7 m; L = 23 J]
35. Find the final speed of an electron, which moves at ve = 5000 m/s, after an elastic collision with a
standstill proton. Find also the final speed of the proton.

[ve = −4994 m/s; v p = 6 m/s]


36. Find the average strength that causes a change of momentum of a system ∆q = 50 kg m/s acting on
it for ∆t = 20 s.

Francesco Saitta, Pordenone Settembre 2019 85


Quantità di moto ed urti

[Fm = 2, 5 N]
37. A block of mass m1 is initially at rest on a frictionless horizontal surface. A bullet of mass m2 is fired
at the block with a speed v. The bullet sticks in the block, and the block ends up with a speed V.
What is the momentum of the bullet with speed v (in terms of m1 , m2 , and V) ?

[q = (m1 + m2 )V]
38. Consider two cars (m1 = 600 kg and m2 = 1200 kg) crashing at a intersection between the two streets
they were travelling. The two streets are perpendicular and the two speeds are v1 = 40 m/s and
v2 = 10 m/s. Find the final velocity of the cars if the crash can be approximated as a completely
inelastic collision.

[v f = 14, 9 m/s with and angle α = 0, 46 rad with respect to car 1]


39. Calculate the momentum of a rhino (m = 3500 kg) charging a hunter at a speed of 9 m/s. Compare
the rhino’s momentum with the momentum of a tranquilizer dart (m = 0, 2 g) fired at a speed v = 500
m/s. What is the loss of kinetic energy of the rhino just after the collision if the hunter hits him with
the tranquilizer dart?

[qr = 31500 kg m/s; qd = 0, 1 kg m/s; ∆Ec = 3 J]

Francesco Saitta, Pordenone Settembre 2019 86


5 Momento angolare e moto rotatorio

Nel corso del biennio abbiamo imparato che un sistema è un equilibrio


se la risultante delle forze agenti sul sistema è nulla (equilibrio di trasla-
zione) e se la risultante di tutti i momenti delle forze agenti sul sistema
è nulla (equilibrio rotazionale). Abbiamo poi studiato approfonditamen-
te la dinamica dei punti materiali fuori dall’equilibrio, con le tre leggi di
Newton, ma non abbiamo mai studiato cosa succede ad un corpo rigido
nel momento in cui la somma dei momenti delle forze sia diversa da zero.
L’argomento centrale di questo capitolo è proprio lo studio di corpi rigidi
fuori equilibrio, quando cioè ruotano di moto non uniforme.

I moti rotatori di un corpo rigido, che ricordiamo è un corpo non sog-


getto a deformazioni in cui cioè le mutue distanze tra i punti che lo
compongono restano invariate nel tempo, possono essere molto com-
plessi da descrivere sia in termini fisici che matematici. Per questo noi
ci limiteremo a descrivere in modo analitico solamente il caso in cui la
rotazione del corpo avvenga lungo un certo asse fissato1 : specificheremo 1
Possiamo immaginare di stu-
di volta in volta i concetti che valgono in generale e quelli che valgono diare un sistema con asse di ro-
tazione fissato oppure un siste-
solamente rispetto alla nostra situazione di studio. ma con asse di rotazione varia-
bile in un intervallo di tempo
talmente piccolo che la variazio-
Fu Eulero con uno dei suoi più importanti libri uno dei primi scien- ne dell’asse non è significativa
ziati a descrivere in modo analitico ed esauriente la dinamica dei corpi per la nostra analisi.
rigidi, in particolare con riferimento al momento d’inerzia. Nell’opera Basilea, 1707 - San Pietroburgo,
1783
troviamo tanto le definizioni di corpo rigido, momento d’inerzia e moto
(Euler, 1765)
rotatorio, quanto tutti i calcoli per descrivere al meglio il fenomeno. Eu-
lero certamente fu una delle menti matematiche e fisiche più importanti
ed influenti della storia della scienza, scrisse moltissimo e molte delle sue
intuizioni furono di fondamentale importanza per lo sviluppo successivo
di matematica e fisica.

5.1 Momento d’inerzia

Quando abbiamo studiato un sistema fuori dall’equilibrio di traslazione


abbiamo osservato che l’effetto dell’applicazione di una forza su un pun-
to materiale è l’accelerazione, ovvero il moto rettilineo non uniforme del
punto materiale stesso; analogamente ci aspettiamo che l’effetto dell’ap-
plicazione di un momento di una forza su un corpo rigido sia un moto
rotatorio non uniforme del sistema, ovvero il presentarsi di un’accelera-
zione angolare. Se questo è vero, ci aspettiamo di trovare un’equazione

Francesco Saitta, Pordenone Settembre 2019 87


Momento angolare e moto rotatorio

del tipo:
~ = I~α
M (5.1.1)

dove M ~ è la risultante dei momenti di forza applicati al sistema, α l’ac-


celerazione angolare ed I una certa costante di proporzionalità tra le due
grandezze. La costante di proporzionalità è chiamata momento d’iner-
zia, si misura in kg·m2 e fa le veci della massa inerziale nella dinamica
lineare: è l’inerzia rotazionale del corpo, ciò che si oppone alla variazione
di stato di moto rotatorio del corpo quando soggetto a momenti di forza.
Facciamo notare qui che la relazione (5.1.1) ha senso nel momento in cui
la rotazione avviene attorno ad un asse fissato, come vedremo in segui-
to il momento d’inerzia può essere calcolato e dipende dalla geometria
dell’oggetto ma anche dall’asse di rotazione dell’oggetto stesso: cambian-
do l’asse di rotazione I non è più costante e l’equazione data deve essere
modificata. Nei prossimi paragrafi cercheremo di darci ragione dell’equa-
zione (5.1.1) assumendo sempre rotazioni attorno ad un asse fissato in casi
via via più complessi.

5.1.1 Rotazione di un punto materiale


Immaginiamo un punto materiale in rotazione attorno ad un asse fisso
sotto l’effetto di una forza costante ~F diretta lungo la direzione della ve-
locità tangenziale ~v, come descritto in figura (5.1). Possiamo immaginare
che il punto materiale sia una persona seduta sul bordo di un tagadà 2 2
giostra meccanica presente in
nel momento in cui la giostra inizia a girare sotto l’effetto di una coppia molti parchi dei divertimen-
ti composta da un disco che
di forze costante, ancora senza inclinazione rispetto al suolo. può ruotare attorno al suo cen-
Evidentemente la forza ~F genera un momento M ~ = ~r × ~F perpendico- tro, anche attorno ad assi non
perpendicolari al terreno
lare al piano su cui ruota il punto materiale, ma anche un’accelerazione
tangenziale, che non modifica la distanza dal centro O del punto, ma au-
menta la velocità tangenziale e quindi quella angolare del punto di massa
m: v(t) = ω (t)r. Proviamo quindi a studiare la meccanica del sistema:

• Il modulo del momento della forza è ottenibile secondo la formula M =


rF, essendo la forza sempre perpendicolare al raggio,

• La forza è sempre tangente alla traiettoria, contribuisce quindi ad una


accelerazione tangenziale secondo il principio della dinamica F = mat ,

• Essendo la forza costante anche l’accelerazione è costante e dunque


l’accelerazione tangenziale media è sempre uguale all’accelerazione tan-
genziale istantanea, per cui possiamo scrivere: Figura 5.1: Rotazione accelerata
di un punto materiale
∆v ∆(ωr ) ω ( t2 )r − ω ( t1 )r ω ( t2 ) − ω ( t1 ) ∆ω
at = = = =r =r = rα
∆t ∆t t2 − t1 t2 − t1 ∆t

Mettendo assieme tutte queste considerazioni possiamo quindi scrivere:

M = rF
= rmat
= r2 mα

Francesco Saitta, Pordenone Settembre 2019 88


Momento angolare e moto rotatorio

Il che conferma il fatto che il momento della forza sia direttamente pro-
porzionale all’accelerazione angolare; ma non solo, questo ragionamen-
to ci fornisce anche l’espressione del momento d’inerzia per un punto
materiale che ruoti lungo una circonferenza:

I = mr2 (5.1.2)

Questa equazione, pur nella sua semplicità, dà molte informazioni sul-


la natura del momento d’inerzia: questa grandezza fisica è connessa alla
massa ed al modo in cui la massa è distribuita attorno al centro di rotazio-
ne del sistema, più la massa è concentrata attorno al centro e meno inerzia
rotazionale ha il sistema, viceversa più la massa è distante dal centro di
rotazione del sistema e più inerzia rotazionale ha il sistema. Pur studian-
do un fenomeno molto semplice ed idealizzato possiamo già cominciare
a capire ad esempio come mai per riuscire a completare le evoluzioni in
aria i pattinatori od i tuffatori si rannicchiano attorno all’asse della loro
rotazione.

5.1.2 Rotazione di un sistema di punti materiali


Se immaginiamo di complicare la situazione ed avere una serie di n pun-
ti materiali tutti ruotanti attorno allo stesso asse possiamo riproporre lo
stesso ragionamento del paragrafo precedente e giungere alla conclusione
che il momento d’inerzia del sistema così fatto è dato da:
n
I= ∑ mi ri2 (5.1.3)
i =1

Vediamo di nuovo quindi come nel determinare l’inerzia di un sistema


non conti solamente la massa di ogni singolo elemento del sistema ma
anche la sua distribuzione rispetto l’asse di rotazione. Questo passaggio
da punto materiale a sistema di punti materiali che ruotano tutti attorno
ad uno stesso asse fisso non è particolarmente significativo dal punto di
vista della sua concretezza, ma è importante per poter definire il momento
d’inerzia di un corpo rigido che ruota attorno ad un certo asse fissato.

5.1.3 Rotazione di un corpo rigido


Consideriamo ora un corpo rigido che ruota attorno ad un asse fisso, come
ad esempio una trottola che ruota in verticale o un pallone da pallacane-
stro sul dito di un cestista particolarmente abile, schematizzato come in
figura (5.2).
In questo caso possiamo pensare di suddividere il corpo rigido in molte
piccole parti di massa dm, ognuna delle quali contribuirà al momento
d’inerzia per dm r2 se r è la distanza di questo elemento di massa dall’asse
di rotazione. Per sommare poi ogni piccolo contributo dovremo fare una
somma particolare, che in matematica è data da un certo tipo di integrale
(materia di studio dell’ultimo anno di corso di matematica) e che si scrive:
Z
Figura 5.2: Corpo rigido che
I= dm r2 (5.1.4)
ruota attorno ad un asse fisso

Francesco Saitta, Pordenone Settembre 2019 89


Momento angolare e moto rotatorio

Evidentemente questo momento d’inerzia ancora dipende da come la


massa è distribuita attorno all’asse di rotazione. Noi al momento non
siamo in grado di calcolare questo tipo di momenti d’inerzia, per cui fare-
mo riferimento a tabelle, facilmente reperibili sul web, in cui sono riportati
i valori di alcuni momenti d’inerzia fissata la geometria del corpo rigido
e l’asse di rotazione passante per il centro di massa del corpo rigido. Per
esempio il momento d’inerzia di una sfera omogenea di massa m e rag-
gio r che ruota attorno ad un asse passante per il suo centro è 2/5 mr2 ;
se la sfera è invece cava con spessore del guscio trascurabile, il momento
d’inerzia diventa 2/3 mr2 ; il momento d’inerzia di un cilindro pieno a
base circolare, di massa m e raggio r qualsiasi sia la sua altezza, rispetto
ad un asse passante per il centro della circonferenza di base e parallelo
all’altezza è mr2 /2. In ogni caso si può sempre notare come il momento
d’inerzia aumenti man mano che la distribuzione di massa si concentra
lontano dall’asse di rotazione del corpo rigido, diminuisca man mano che
la massa si concentra intorno all’asse. Nei casi in cui l’asse di rotazione
non sia passante per il centro di massa del corpo rigido ma sia parallelo
ad un asse passante per il centro di massa si ricorre al Teorema di Steiner.

Teorema di Steiner Il momento d’inerzia di un corpo rigido ruotante at-


torno ad un asse fisso parallelo ad un asse passante per il centro di massa
cm del corpo rigido stesso è dato dalla relazione

I = Icm + md2 (5.1.5)

Dove Icm è il momento d’inerzia rispetto all’asse passante per il centro di


massa, m la massa del corpo e d la distanza tra i due assi. Ad esempio il
momento d’inerzia di una barra sottile di massa m e lunghezza L rispetto
ad un asse passante per il suo centro e perpendicolare alla barra stessa è
1/12 mL2 , se invece l’asse è perpendicolare alla barra ma passante per un
suo estremo esso diventa 1/3 mL2 = 1/12 mL2 + m( L/2)2 .

5.2 Momento angolare

I moti rotatori che riusciamo a descrivere in modo analitico al momento


sono dunque solamente quelli con asse di rotazione fisso. Quando l’asse
di rotazione non è fissato esiste comunque una relazione tra i momenti
delle forze e le accelerazioni angolari, la complicazione matematica che
noi non affronteremo è che la costante di proporzionalità non è un nume-
ro: il momento d’inerzia dipende dall’asse di rotazione e formalmente è
rappresentato non da un numero ma da un tensore, oggetto matematico
di cui non ci occupiamo.

Possiamo però descrivere in modo generale la rotazione di un corpo


rigido introducendo una nuova grandezza, il momento angolare. Questa
grandezza fisica si definisce, per un punto materiale in movimento, nel
seguente modo:
~L = ~r × ~q (5.2.1)

Francesco Saitta, Pordenone Settembre 2019 90


Momento angolare e moto rotatorio

dove ~L è appunto il momento angolare, ~q la quantità di moto ed ~r il


raggio vettore che collega il centro rispetto a cui vogliamo calcolare il
momento angolare e la posizione del punto materiale. Parlando di un
punto materiale che ruota rispetto ad un centro, come nell’esempio (5.1)
~r sarà lo stesso vettore rispetto a cui si calcola il momento della forza.
Sviluppando l’espressione (5.2.1) con i parametri dell’esempio si ottiene la
seguente relazione tra momento angolare e momento d’inerzia del punto
materiale:
~L = I ω
~ (5.2.2)

Anche in questo caso l’espressione, con gli opportuni ragionamenti ana-


loghi a quelli fatti nel paragrafo precedente, è valida solamente per un
punto materiale, un sistema di punti o un corpo rigido che ruotino attor-
no ad un asse fissato. Per arrivare ad una relazione tra la causa del non
~ e questa nuova grandezza dobbiamo conside-
equilibrio di rotazione ( M)
rare la definizione di momento di forza, di momento angolare e il teorema
dell’impulso (4.1.1)3 : 3
In particolare il passaggio ~r ×
∆~q = ∆(~r × ~q) è giustificato
~ dalla proprietà distributiva del
M = ~r × ~F
prodotto vettoriale rispetto alla
∆~q somma e divisione
= r×
∆t
r × ∆~q
=
∆t
∆(~r × ~q)
=
∆t
∆L~
=
∆t

Con ragionamenti analoghi a quelli fatti in precedenza possiamo conclu-


dere che la relazione
~ = ∆L
~
M (5.2.3)
∆t

è valida per qualsiasi corpo rigido su cui stia agendo un momento delle
~ Questa espressione è indipendente dal fatto che la ro-
forze risultante M.
tazione avvenga rispetto ad un asse fisso, vale in generale: l’applicazione
di un momento di forza su un corpo rigido è dunque causa di una varia-
zione di momento angolare. L’espressione (5.2.3) si riferisce al momen-
to medio applicato nell’intervallo di tempo ∆t, se volessimo considerare
invece il momento istantaneo troveremmo la relazione:

~
~ = dL
M (5.2.4)
dt

che giustificheremo matematicamente quando nel corso di matematica


verranno affrontate le derivate.

5.3 Teorema di conservazione del momento angolare

Abbiamo ormai iniziato a capire che un legge formalmente come la (5.2.4),


in cui l’effetto di una certa causa è la variazione di una grandezza fisica,

Francesco Saitta, Pordenone Settembre 2019 91


Momento angolare e moto rotatorio

prelude ad una legge di conservazione: se la causa è assente infatti non vi


è variazione dell’effetto e dunque la grandezza in questione si conserva.
Arriviamo così ad un nuovo teorema di conservazione.

In un sistema in cui la risultante dei momenti di forza è nulla il


momento angolare totale del sistema si conserva:

~
~ = ~0 ⇐⇒ d L = ~0 ⇐⇒ ~L = ~k
M (5.3.1)
dt

Aggiungiamo quindi la conservazione del momento angolare alle conside-


razioni ed all’elenco fatto precedentemente (4.2). L’importanza e l’effetto
della conservazione del momento angolare può essere testata da ognuno
di noi in casa, a patto di possedere una sedia girevole: sedendosi sulla
sedia girevole ed iniziando a ruotare sulla sedia possiamo notare come
allargare le braccia faccia frenare la rotazione, mentre stringerle a noi la
facciamo accelerare4 ; dopo aver provato questo esperimento fatto in casa 4
naturalmente trascuriamo gli
e verificato che in effetti accade proprio ciò che abbiamo descritto, si pro- effetti del momento della for-
za peso sulle braccia e ci scon-
vi a spiegare questo effetto sulla base della conservazione del momento triamo con l’attrito che fa co-
angolare e dalle definizioni date precedentemente. munque rallentare la rotazio-
ne: per effettuare al meglio
l’esperimento dovremmo esse-
5.4 Dinamica rotazionale re nel vuoto con una sedie
perfettamente oliata...

Studiare la dinamica rotazionale significa capire quanto lavoro fanno le


forze esterne quando vengono applicate ad un corpo rigido, e come que-
sto lavoro sia legato alla variazione di energia cinetica che chiameremo
rotazionale del sistema. Per far questo ci riferiremo sempre ad un punto
materiale che ruota lungo una circonferenza di raggio fissato sotto l’effet-
to di una forza di modulo costante F sempre tangente alla traiettoria del
punto; generalizzeremo poi i risultati ottenuti ad un qualsiasi corpo rigido
omettendo i ragionamenti ed i passaggi matematici che abbiamo accenna-
to nei paragrafi precedenti e che non siamo ancora in grado di effettuare
formalmente. Per i passaggi matematici ci riferiremo quindi sempre alla
figura (5.3).

5.4.1 Lavoro dei momenti delle forze


Consideriamo la situazione descritta in precedenza e cerchiamo di calco-
lare il lavoro fatto dalla forza ~F in un certo intervallo di tempo. Per far ciò
dobbiamo considerare un pezzo di traiettoria curvilinea s talmente piccolo Figura 5.3: Punto materiale in
da essere confondibile con la corda ~s = ~r (t2 ) −~r (t1 ); con questa approssi- rotazione accelerata
mazione possiamo affermare che la forza ~F è parallela al vettore ~s, vettore
spostamento che coincide con lo spazio percorso nell’intervallo di tempo
(t2 − t1 ) ed anche che s = rθ, θ l’angolo spazzato dal punto materiale.
Possiamo quindi giustificare i seguenti passaggi:

L = ~F ·~s
= Fs
= Frθ
= Mθ

Francesco Saitta, Pordenone Settembre 2019 92


Momento angolare e moto rotatorio

Pur avendo schematizzato la situazione possiamo dire che il lavoro fatto


da un momento torcente costante su un qualsiasi corpo rigido è dato dal
prodotto tra il modulo del momento per l’angolo spazzato dal corpo ri-
gido attorno all’asse di rotazione, che resta fisso nell’ipotesi di momento
torcente costante:
L = Mθ (5.4.1)

Naturalmente possiamo calcolare anche la potenza sviluppata dal mo-


mento torcente dividendo il lavoro fatto per l’intervallo di tempo ottenen-
do
P = Mω, (5.4.2)

dove ω è la velocità angolare media del corpo rigido nell’intervallo di


tempo (t2 − t1 ), o istantanea nell’approssimazione in cui l’intervallo di
tempo sia molto piccolo rispetto a tutto il moto considerato.

5.4.2 Energia cinetica rotazionale

Evidentemente se la risultante dei momenti torcenti agenti su un sistema


fa lavoro secondo la formula (5.4.1) il sistema, secondo la definizione di
energia introdotta nel primo capitolo, acquisterà energia cinetica in quan-
tità uguale al lavoro fatto dalla risultante dei momenti (trascurando tutti
gli attriti agenti sul sistema). Per capire come è fatta questa energia ci-
netica rotazionale, dovuta cioè al lavoro dei momenti torcenti, dobbiamo
ricordare la cinematica del moto circolare uniformemente accelerato:

1
θ (t) = α ( t − t i )2 + ωi ( t − t i ) + θ i
2
ω ( t ) = α ( t − t i ) + ωi

con α l’accelerazione angolare del sistema. Nel corso dei calcoli che segui-
ranno è utile la seguente relazione che esprime la velocità angolare media
in funzione della velocità angolare finale e quella iniziale nel caso di un
moto circolare uniformemente accelerato:

θ f − θi
ωm =
∆t
2 ( ∆t ) + ωi ∆t + θi − θi
1 2
α
=
∆t
1
= α∆t + ωi
2
α∆t + 2ωi
=
2
α∆t + ωi + ωi
=
2
ω f + ωi
=
2

A questo punto basta notare come un corpo rigido sotto l’effetto di un


momento torcente costante il sistema compie un moto circolare unifor-
memente accelerato caratterizzato dalle relazioni appena viste e seguire il

Francesco Saitta, Pordenone Settembre 2019 93


Momento angolare e moto rotatorio

seguente calcolo:

L = Mθ
∆L
= ∆θ
∆t
Iω f − Iωi
= ∆θ
∆t
∆θ
= ( Iω f − Iωi )
∆t
= ( Iω f − Iωi )ωm
ω f + ωi
= ( Iω f − Iωi )
2
(ω f − ωi )(ω f + ωi )
= I
2
2
ω f − ωi 2
= I
2
Iω 2f Iω 2
= − i
2 2

che evidenzia come il lavoro dei momenti torcenti dipenda solamente dal-
le velocità angolari iniziali e finali. Questa espressione dice dunque la
quantità di energia cinetica guadagnata dal sistema quando partendo da
una velocità angolare ωi raggiunge una velocità angolare ω f sotto l’effetto
del momento torcente costante di modulo M. L’espressione dell’energia
cinetica rotazionale è dunque:

1 2
Ec = Iω (5.4.3)
2

5.4.3 Moti di un corpo rigido


Cerchiamo ora di riassumere i possibili movimenti di un corpo rigido alla
luce di tutte le cose studiate fino ad ora. Considereremo come caso di
studio un disco omogeneo, concretizzabile per esempio con una ruota di
bicicletta o di un’automobile o di una motocicletta. Le considerazioni che
seguono possono poi essere applicate ad un generico corpo rigido.

5.4.3.1 Traslazione pura


Il moto più semplice che possiamo immaginare e che abbiamo già intro-
dotto nel corso del primo biennio di corso è il moto di traslazione pura,
ovvero il moto per cui ogni elemento del corpo rigido si muove con la stes-
sa velocità (sia essa costante o variabile). In questo caso, illustrato in figura
(5.4), il centro di massa del disco si muove con la stessa velocità di ogni Figura 5.4: Traslazione pura
punto del corpo: possiamo pensare ad esempio ad una ruota che poggia
su un piano completamente privo di attrito e viene spinta orizzontalmente
all’altezza del centro del disco stesso. Dal punto di vista energetico in
questo caso la ruota di massa m ha un’energia cinetica dovuta solo dalla
componente di traslazione e data dall’espressione:

1 2
Ec = mv
2

Francesco Saitta, Pordenone Settembre 2019 94


Momento angolare e moto rotatorio

5.4.3.2 Rotazione pura


In questo capitolo abbiamo introdotto un modo per descrivere la rotazio-
ne di un corpo rigido dal punto di vista della sua dinamica. Il moto di
rotazione più semplice per un corpo rigido è chiamato rotazione pura ed
è la rotazione attorno ad un fulcro, senza considerare l’attrito sul fulcro
stesso. Immaginiamo per esempio di far ruotare la ruota di una bicicletta
sollevandola da terra. In figura (5.5) è descritto il moto del disco quando
la rotazione avviene attorno al centro di massa: il centro di massa resta
fermo, mentre tutti i punti del disco hanno la stessa velocità angolare (co-
stante o variabile che sia). La velocità tangenziale di ogni punto materiale
del disco è quindi sempre tangente alla circonferenza di centro il centro
di massa e raggio la distanza tra centro e posizione del punto materiale
stesso: a parità di distanza dal centro il modulo della velocità tangenziale
è sempre la stessa.
Dal punto di vista energetico in questo caso la ruota ha un’energia ci-
netica dovuta solo dalla componente di rotazione e data dall’espressione:

1 2
Ec = Iω
2
Figura 5.5: Rotazione pura
con I il momento d’inerzia del disco ed ω la velocità angolare di ogni
punto del disco stesso.

5.4.3.3 Rotolamento puro


Il moto di rotolamento puro è la combinazione dei due moti visti in pre-
cedenza, possiamo immaginarlo come il moto della ruota di una bicicletta
quando è completamente assente lo scivolamento della ruota sul terre-
no: il punto di contatto tra ruota e terreno è sempre in quiete. La figu-
ra (5.6) illustra la situazione della velocità di alcuni punti della ruota: è
semplicemente la somma vettoriale delle velocità dei due casi precedenti.
Dal punto di vista energetico in questo caso la ruota ha un’energia
cinetica dovuta sia alla componente di traslazione che a quella di rotazione
e data quindi dall’espressione:

1 2 1
Ec = mv + Iω 2 Figura 5.6: Puro rotolamento
2 cm 2

5.5 Parallelismo tra dinamica traslatoria e rotatoria

In questo capitolo abbiamo studiato la dinamica rotatoria dei corpi rigi-


di ed abbiamo potuto notare come ci siano moltissime analogia formali
e concettuali tra la dinamica rotatoria e quella traslatoria già studiata in
precedenza. La tabella (5.1) le riassume ed evidenzia, i diversi simboli
sono gli stessi utilizzati in precedenza pertanto non necessitano ulteriori
spiegazioni. È invece importate riflettere sulla portata di tale osservazio-
ne: sembra proprio vero che l’Universo sia scritto in termini matematici !!! (Galilei, 1623)
Ed è interessante e sconvolgente da certi punti di vista come leggi fisiche
descrivono concetti analoghi siano descritte allo stesso modo in termini
matematici: ad esempio la relazione causa effetto e la definizione di iner-
zia trovano in ~F = m~a e M ~ = I~α, due equazioni che differiscono solo per

Francesco Saitta, Pordenone Settembre 2019 95


Momento angolare e moto rotatorio

il nome dato alle variabili, la loro espressione matematica o l’espressione


del teorema di conservazione di grandezze fisiche diverse tra loro ha la
stessa forma matematica. Questo fa davvero pensare al mistero della ca-
pacità della matematica nel descrivere la natura ed all’estrema armonia e
simmetria presente nelle leggi che regolano il nostro universo.

Tabella 5.1: Parallelismo tra le


Moti traslatori Moti rotatori quantità della dinamica trasla-
toria e quelle della dinamica
~s θ rotatoria
~v ~
ω
~a ~α = ~

dt
~F ~
M
m I
~F = m~a ~ = Iα
M
~q = m~v ~L = I ω
~
~F = d~q ~ = d~L
M
dt dt
~F = ~0 ⇒ ~q = ~k ~ = ~0 ⇒ ~L = ~k
M
L = ~F ·~s L = Mθ
P = ~F · ~v P = Mω
1
Ec = 2 mv
2 Ec = 12 Iω 2

5.6 Equazioni cardinali della dinamica

Sappiamo che ai fisici piace molto la sintesi e l’idea di descrivere il mag-


gior numero di fenomeni naturali con il minor numero di equazioni è un
punto fisso per le comunità scientifiche in tutta la storia della fisica; con-
cludiamo quindi il capitolo mettendo in evidenza le due equazioni più
importanti tra quelle viste, quelle che mettono in relazione le cause della
variazione delle quantità che in assenza di queste cause resterebbero co-
stanti, che chiamiamo equazioni cardinali della dinamica poichè a partire
da queste possiamo definire i teoremi di conservazione e ricostruire tutta
la dinamica vista fin’ora:
~F = d~q (5.6.1)
dt
~
M~ = dL (5.6.2)
dt

Francesco Saitta, Pordenone Settembre 2019 96


Momento angolare e moto rotatorio

Francesco Saitta, Pordenone Settembre 2019 97


Momento angolare e moto rotatorio

5.7 Esercizi

1. Si calcoli il momento angolare della terra nel suo moto di rivoluzione attorno al sole.

[L = 2, 6 × 1040 kg m2 /s]
2. Si calcoli il momento angolare della terra nel suo moto di rotazione; si approssimi la terra ad una
sfera rotante attorno ad un suo diametro.

[L = 7 × 1027 kg m2 /s]
3. Si calcoli il momento angolare di un disco 33 giri, nell’ipotesi che la sua massa sia m = 180 g.

[L = 7 × 10−3 kg m2 /s]
4. Un motore è in grado di erogare una coppia di intensità 200 Nm per un massimo di 3,5 secondi. Di
quanto varia il momento angolare dell’albero cui il motore è collegato?

[∆L = 700 kg m2 /s]


5. Un disco di massa m = 5 kg e raggio R = 30 cm inizia a ruotare attorno al suo asse. In 10 secondi
passa da una velocità angolare ωi = 3 rad/s ad una velocità angolare ω f = 6 rad/s. Qual è l’intensità
del momento torcente che ha causato questa accelerazione?

[M = 0, 07 Nm]
6. Un disco di massa m = 10 kg e raggio R = 50 cm sta ruotando alla frequenza ν = 2 Hz. Si ferma per
effetto delle forze d’attrito in 4 secondi. Qual è l’intensità del momento torcente applicato dalle forze
d’attrito sul disco?

[M = 3, 9 Nm]
7. Un giocatore di pallacanestro fa ruotare la palla su un dito finché la palla gira ad una frequenza su-
periore a ν = 30 Hz. Supponendo che inizi a far girare la palla con una velocità angolare ωi = 250
rad/s, e che l’attrito agisca sulla palla con un momento torcente di modulo M = 5 Nm, per quanto
tempo riuscirà il giocatore a far ruotare la palla sul suo dito? Si supponga la palla una sfera (piena)
perfetta di massa M = 600 g e raggio R = 24 cm.

[∆t = 0, 17 s]
8. Che momento torcente deve applicare un ragazzo per far ruotare le sue due bolas (m = 200 g l’una)
facendo raggiungere loro una velocità ω = 4 rad/s in 15 secondi? Si supponga che le bolas siano sfere
perfette che si trovano alle estremità di un cordino lungo 1,2 m, fatto ruotare tenendone il centro in
mano.

[M = 0, 04 Nm]
9. Uno scienziato vuole conoscere il momento d’inerzia di uno strano oggetto, rispetto almeno uno dei
suoi assi di simmetria. Ha l’idea di iniziare a farlo ruotare attorno all’asse prescelto, per riuscire nel
suo intento. Rispetto la rotazione osservata riesce a misurare: il momento torcente applicato, M = 40
Nm, la durata dell’esperimento ∆t = 10 s, la frequenza finale ν = 3 Hz (l’oggetto partiva da fermo)
Quale sarà la sua conclusione sul momento d’inerzia dell’oggetto?

[I = 21 kg m2 /s2 ]

Francesco Saitta, Pordenone Settembre 2019 98


Momento angolare e moto rotatorio

10. Una sbarra omogenea di lunghezza l = 2 m e massa m = 800 g si trova appesa in posizione verticale.
Un ragazzo inizia a farla ruotare attorno al punto in cui è appesa applicando un momento torcente
costante M = 40 Nm per un tempo t = 2 s. Quale sarà la velocità angolare raggiunta dalla sbarra?

[ω = 75 rad/s]
11. Un disco di ghiaccio omogeneo (r = 20 cm, m = 200 g) sta ruotando ad una velocità angolare
ωi = 10 rad/s. Per effetto degli attriti il disco inizia a sciogliersi e perdere massa. Supponendo che
sciogliendosi sia sempre approssimabile ad un disco omogeneo dello stesso raggio, si calcoli la massa
del disco quando il disco raddoppia la sua velocità angolare.

[m = 100 g]
12. Un tuffatore per aumentare la sua velocità angolare e riuscire a fare il tuffo previsto dal suo numero
si piega su se stesso dimezzando la distanza massima del suo corpo da centro di rotazione. Se la
sua massima velocità angolare raggiunta prima di piegarsi è ω1 = 2 rad/s, quale sarà la sua velocità
angolare dopo il piegamento?

[ω f = 8 rad/s]
13. Un giocoliere sta facendo ruotare sopra la sua testa una pallina legata all’estremità di una corda
lunga l = 80 cm, di massa m = 500 g e raggio trascurabile, alla frequenza ν = 20 Hz. Per aumentare la
velocità angolare della pallina fino a ω f = 200 rad/s senza applicare momento torcente cosa deve fare?

[ridurre la lunghezza della corda fino ad un valore l f = 63 cm]


14. Una freccetta di massa m = 25 g che si muove con la velocità v f = 50 m/s si conficca tangenzialmen-
te sul bordo di un disco fermo di massa M = 500 g e raggio R = 20 cm. Si trovi la velocità angolare
finale del sistema disco/freccetta.

[ω f = 22, 7 rad/s]
15. Un cilindro vuoto di massa m = 500 g sta ruotando attorno al suo asse con una velocità angolare
ωi = 10 rad/s. Il cilindro viene riempito di un certo materiale ad un ritmo di 20 grammi al secondo.
Dopo quanto tempo la velocità angolare raggiungerà il valore ω f = 3 rad/s? Si supponga che l’ope-
razione di riempimento del cilindro avvenga senza imprimere un momento torcente al sistema.

[t = 58 s]
16. Si risolva l’esercizio (14) nel caso in cui il disco non sia inizialmente fermo ma stia ruotando, con
verso concorde alla velocità della freccetta, con una velocità angolare ωi = 0, 5 rad/s.

[ω f = 0, 52 rad/s]
17. Si risolva l’esercizio (14) nel caso in cui il disco non sia inizialmente fermo ma stia ruotando, con
verso opposto alla velocità della freccetta, con una velocità angolare ωi = 0, 5 rad/s.

[ω f = 0, 47 rad/s]
18. La figura (5.7) mostra dall’alto una palla di pongo (m = 200 g) che sta per colpire una porta ini-
zialmente ferma (di lunghezza totale L = 50 cm e massa M = 5 kg). Nell’ipotesi che la palla resti
attaccata alla porta dopo l’urto, si calcoli la velocità angolare finale del sistema palla/porta.

[ω f = 4, 6 rad/s]

Francesco Saitta, Pordenone Settembre 2019 99


Momento angolare e moto rotatorio

Figura 5.7: .

19. Come cambia in percentuale la velocità angolare di un tagadà (M = 2000 kg) se le 10 persone (m = 60
kg) a bordo si spostano dal bordo (R = 3 m) al centro della giostra?

[aumenta del 60%]


20. Un cilindro di massa M = 1 kg e e volume V = 5 l sta ruotando attorno al suo asse pieno di una
massa m = 2 kg d’acqua ad una velocità angolare ω = 3 rad/s. Ad un certo punto si apre un foro sul
fondo del cilindro e l’acqua inizia ad uscire al ritmo di r = 0, 2 kg/s. Quale sarà la velocità angolare
ω1 del cilindro dopo un tempo t = 5 s? Quale sarà la velocità angolare ω2 del cilindro quando tutta
l’acqua è fuoriuscita dal foro?

[ω1 = 4, 5 rad/s; ω2 = 9 rad/s]


21. Quanto lavoro fa il motore di un’auto di momento d’inerzia I = 1500 kgm2 che fornisce una coppia
M = 200 Nm per 12 secondi?

[L = 1920 J]
22. Quanto lavoro fa il motore di un’auto di momento d’inerzia I = 1500 kgm2 che fornisce una coppia
M = 200 Nm per raggiungere la velocità angolare finale ω f = 5 rad/s?

[L = 18750 J]
23. Su un volano di momento d’inerzia I = 200 kgm2 agisce una coppia di forze costante che fa un
lavoro L = 300000 J. Qual è la frequenza iniziale νi del volano se la frequenza raggiunta è ν f = 50 Hz?

[νi = 49, 2 Hz]


24. Quanto lavoro devono fare gli attriti per fermare la rotazione di un cilindro di massa M = 30 kg e
raggio R = 50 cm che ruota attorno al suo asse con velocità angolare ω = 1, 5 rad/s?

Francesco Saitta, Pordenone Settembre 2019 100


Momento angolare e moto rotatorio

[L = 8, 4 J]
25. Una pallina sferica di massa m = 200 g e raggio r = 20 cm viene lanciata in orizzontale compiendo
un lavoro L = 1900 J. Supponendo che non ci siano attriti qual è la sua velocità angolare, prima che
l’effetto della gravità diventi non trascurabile, se la sua velocità è v = 100 m/s?

[ω = 750 rad/s]
26. Si calcoli l’energia totale di un asta sottile (M = 200 kg, l = 30 cm) che sta ruotando alla frequenza
ν = 50 Hz attorno ad uno dei suoi estremi senza traslare.

[E = 296 kJ]
27. Si calcoli l’energia totale di un cilindro (M = 2 kg, R = 20 cm) che, facendo un moto di puro rotola-
mento, percorre 3 metri in 2 secondi.

[E = 3, 4 J]
28. Si risolva il problema (30) nel caso in cui a muoversi sia una sfera della stessa massa e dello stesso
raggio del cilindro. Si commenti il risultato.

[E = 3, 2 J]
29. Si calcoli la variazione di energia nel processo fisico descritto dal problema (18).

[∆E = −59, 3 J]
30. Un cilindro (M = 5 kg, R = 30 cm) viene lasciato scivolare lungo un piano inclinato (α = π/6 rad)
da un’altezza h = 3 m. Quale sarà la sua velocità angolare nel momento in cui raggiunge il fondo del
piano inclinato? Si supponga un moto di rotolamento puro per il cilindro.

[ω = 21 rad/s]
31. Si risolva il problema (30) nel caso in cui a cadere sia una sfera della stessa massa e dello stesso
raggio del cilindro. Si commenti il risultato.

[ω = 22 rad/s]
32. Un’asta sottile omogenea di massa m = 2 kg e lunga l = 1, 2 m si trova in posizione verticale quando
viene leggermente spostata dalla posizione di equilibrio instabile in cui si trovava. Si calcoli la velocità
del suo punto estremo (il più alto nel momento in cui si trova in posizione di equilibrio) un attimo
prima di toccare terra. Come cambia il risultato se la massa dell’asta dovesse raddoppiare?

[v = 3 m/s; il risultato non dipende dalla massa dell’asta]


33. Quanto lavoro devono fare le forze d’attrito per fermare un disco di massa m = 10 kg che sta roto-
lando con un moto di puro rotolamento di velocità vcm = 15 m/s?

[L = 1687, 5 J]
34. Qual è la massima altezza raggiunta da un’asta sottile di massa m = 200 g e lunghezza l = 30 cm
lanciata verso l’alto con velocità vcm = 5 m/s e frequenza di rotazione attorno al suo centro di massa
ν = 3 Hz, nell’ipotesi che non agiscano attriti sul sistema e che nel punto più alto l’asta non ruoti.

[h = 1, 4 m]

Francesco Saitta, Pordenone Settembre 2019 101


Momento angolare e moto rotatorio

35. Un giocatore di golf colpisce la pallina con una forza media Fm = 200 N per un intervallo di tempo
∆t = 20 ms. Calcola la massima altezza raggiunta dalla pallina se la pallina può essere approssimata
con una sfera di raggio r = 5 cm e massa m = 100 g, la velocità iniziale della pallina forma un angolo
α = 60 con l’orizzontale e la velocità angolare sia data nell’ipotesi che la pallina qualche istante dopo il
lancio inizi a ruotare con una velocità angolare come se la pallina facesse un moto di puro rotolamento
in aria senza variare la velocità del centro di massa e lo mantenga poi costante.

[h = 28, 6 m]
36. Find the angular momentum of the moon in its revolution around the earth.

[L = 2, 9 × 1034 kg m2 /s]
37. Find the angular momentum of the moon in its rotation. Approximates the moon to a sphere rotating
around its diameter.

[L = 2, 9 × 1024 kg m2 /s]
38. Find the mass of a wheel (R = 40 cm) that is accelerating from ω1 = 0 rad/s to ω2 = 5 rad/s in 25
seconds under the action of a torque M = 20 Nm.

[m = 1250 kg]
39. Find the total mechanical energy of the earth in its revolution around the sun and in its rotation
around its diameter.

[ET = 1, 5 × 1053 J]
40. A rope is wrapped around a cylinder (M = 125 kg, R = 50 cm), with a fixed frictionless axis. The
other end of the rope is tied to a block (M = 10 kg). What is the angular acceleration ac of the cylin-
der? What is the linear acceleration ab of the block? Assume that the rope does not slip on the cylinder.

[ac = 2, 7 rad/s2 , ab = 1, 4 m/s2 ]

Francesco Saitta, Pordenone Settembre 2019 102


6 Gravitazione universale

Nel 1687, nel suo libro Philosophiae Naturalis Principia Mathematica, lo stes-
so in cui enunciò le tre leggi della dinamica, Newton enunciò la legge di
gravitazione universale. Parlando di forza gravitazionale e legge di gravi-
tazione universale non possiamo non accennare al ruolo che ebbe questa
legge nella storia della fisica e nell’evoluzione dalla cosmologia aristote-
lica alla cosmologia moderna.

La legge di Newton si pone come la formalizzazione matematica della


rivoluzione Copernicana, i cui protagonisti furono tra gli altri Coperni-
co1 , Ticho Brahe2 , Keplero3 , Galileo e Newton, che sostituì alla 1
Toruń, 1473 - Frombork, 1543
visione di Aristotele4 e Tolomeo5 di un universo geocentrico la visio- Castello di Knutstorp, 1546 -
2

Praga, 1601
ne moderna di un universo in cui la terra non è altro che un pianeta di 3
Weil der Stadt, 1571 - Ratisbo-
una stella come molte altre, il sole. La teoria Newtoniana fu considerata la na, 1630
corretta descrizione dell’universo per più di 200 anni, quando Einstein 4
Stagira, 384-83 a.C. - Calcide,
322 a.C
formulò la relatività ristretta prima, nel 1905, e la relatività generale poi, 5
Pelusio, 100 - 175
nel 1916, teorie che misero le basi per una nuova concezione dell’universo
e la cosmologia come studiata al giorno d’oggi.

Anche per il nostro corso questa teoria riveste un’importanza parti-


colare: rappresenta infatti la prima teoria fisica, come intesa da metodo
sperimentale studiato al primo anno, che affrontiamo; per la prima volta
metteremo assieme le nozioni di cinematica e dinamica studiate fin qui per
descrivere un fenomeno naturale di primissima importanza nell’universo
come la gravitazione.

6.1 Le leggi di Keplero

Keplero analizzo ed interpretò i dati raccolti da Tycho Brahe in due libri Figura 6.1: Prima legge di Ke-
plero: S è il sole, P uno qualsiasi
nei quali sono contenute le famose tre leggi di Keplero 6 .
dei suoi pianeti.
Queste leggi nate da un’attentissima analisi di una mole enorme di dati
6
(Kepler, 1609, 1619)
e rilette con la convinzione che l’universo debba rispondere ad un’esigen-
za di armonia dei corpi celesti sono state tra le premesse fondamentali per
la teoria Newtoniana della gravitazione, come vedremo in seguito.

6.1.1 La prima legge di Keplero


La prima legge di Keplero asserisce che, come descritto dalla figura (6.1)
ogni pianeta del sistema solare compie un’orbita ellittica in cui il sole è uno dei
due fuochi.

Francesco Saitta, Pordenone Settembre 2019 103


Gravitazione universale

Tabella 6.1: Eccentricità delle


Pianeta Eccentricità dell’orbita orbite ellittiche dei pianeti
del sistema solare; fonte
Mercurio 0,205 dei dati NASA/NSSDC
http://nssdc.gsfc.nasa.gov/
Venere 0,007 planetary/factsheet/
Terra 0,017
Marte 0,094
Giove 0,049
Saturno 0,057
Urano 0,046
Nettuno 0,011

6.1.2 La seconda legge di Keplero


Le seconda legge di Keplero asserisce che ogni pianeta del sistema solare
nella sua orbita spazza aree uguali in tempi uguali. Come illustrato anche in
figura (6.2) questa legge ha diretta conseguenza sulle velocità dei pianeti
nelle diverse parti dell’orbita: essi avranno velocità minore nelle aree più
lontane dal sole, velocità maggiori nelle aree più vicine ad esso; il punto di
velocità massima, quello dell’orbita più vicino al sole si chiama perielio,
quello di velocità minima e massima distanza dal sole afelio

6.1.3 La terza legge di Keplero


La terza legge di Keplero asserisce che per ogni pianeta del sistema solare
Figura 6.2: Seconda legge di Ke-
resta costante il rapporto tra il quadrato del suo periodo di rotazione attorno al plero: S è il sole, P uno qualsiasi
sole e il cubo del semiasse maggiore dell’orbita: dei suoi pianeti.

T2
=k (6.1.1)
a3

6.2 La legge di gravitazione universale

Riportiamo qui una giustificazione matematica della legge che Newton


propose come sintesi di tutte le osservazioni ed i ragionamenti fatti dagli
scienziati vissuti dalla metà del 1400 in poi: cercheremo di comprendere
il motivo della forma che la forza gravitazionale assume. Il grande merito
di Newton fu quello di riuscire a dimostrare matematicamente quello che
alcuni astronomi avevano intuito, assieme al coraggio di ipotizzare che il
tipo di forza che fa cadere un oggetto da un tavolo sulla terra sia la stessa
che fa ruotare la luna attorno al nostro pianeta o un pianeta attorno alla
sua stella: la legge di gravitazione universale che segue è l’esempio di una
delle generalizzazioni fisico-matematiche più importanti della storia della
scienza.

Dalla tabella (6.1) possiamo notare come le eccentricità dei pianeti del
sistema solare siano tutte molto piccole: possiamo quindi pensare che
l’approssimazione delle orbite ellittiche dei pianeti con orbite circolari
possa essere una buona approssimazione. L’ipotesi fatta ci aiuta dal pun-

Francesco Saitta, Pordenone Settembre 2019 104


Gravitazione universale

to di vista matematico nel trovare l’espressione della forza di gravitazione


universale, ma non ne modifica la forma; ragioneremo in seguito sulla
forma delle traiettorie di corpi sotto l’effetto della forza gravitazionale.
Faremo quindi riferimento alla figura (6.3) per i calcoli che seguono.
La forza che il sole applica sul pianeta, per la seconda legge di New-
ton deve essere uguale alla massa del pianeta m P moltiplicata per la sua
accelerazione:
~FSP = m P~a P (6.2.1)

Ipotizzando il moto del pianeta circolare uniforme di raggio R e periodo


T 7 si ha a P = v2P /R = 4π 2 R/T 2 , che porta ad una forza di attrazione
Figura 6.3: S è il sole men-
centripeta tre P uno qualunque dei piane-
ti del suo sistema, nell’approssi-
4π 2 R
FSP = m p . (6.2.2) mazione di orbita circolare. ~FSP
T2 è la forza che il sole applica sul
pianeta, mentre ~FPS è la forza
Fino a questo punto la dinamica descritta non ha nulla a che vedere con la che il pianeta applica sul sole.
gravitazione od il sistema solare, è semplicemente la dinamica di un corpo 7
Nella nostra esperienza possia-
che si muove di moto circolare uniforme; inserendo però nell’equazione mo dire che la terra compie un
(6.2.2) la terza legge di Keplero (6.1.1) si ottiene giro attorno al sole sempre nel-
lo stesso tempo, che definiamo
essere un anno solare.
4π 2 m P
FSP = , (6.2.3)
k S R2

dove k S è la costante che caratterizza tutti i pianeti che ruotano attorno al


sole. Notiamo che la forza con cui il sole attrae un pianeta è direttamente
proporzionale alla massa del pianeta stesso ed inversamente proporzio-
nale al quadrato della distanza tra pianeta e sole. L’equazione (6.2.3) è
dunque valida come legge di gravitazione per tutti i pianeti che compio-
no un moto di rivoluzione attorno alla stella che chiamiamo sole; l’idea
che evidenziò tutta la genialità di Newton fu quella di ipotizzare che la
natura di questa forza fosse la stessa per tutte le masse, ovvero ipotizzare
che prese comunque due masse m1 ed m2 esse si attraggano con una forza
diretta lungo la congiungente le due masse con un’intensità direttamente
proporzionale alle masse ed inversamente proporzionale al quadrato del-
la loro distanza.

Per capire come può essere fatta questa forza torniamo alla descrizione
del sistema sole-pianeta: se il sole applica una forza ~FSP sul pianeta, si-
gnifica che per il terzo principio della dinamica il pianeta deve applicare
sul sole una forza uguale e contraria ~FSP = −~FPS . Se immaginiamo di
descrivere il sistema da un riferimento solidale con il pianeta possiamo
immaginare che la forza di cui risente il sole sia del tipo

4π 2 mS
FPS = , (6.2.4)
k P R2

dove k P ora è la costante della terza legge di Keplero per un sistema


che ruota attorno al pianeta P. Già in questo ragionamento vediamo la
generalizzazione che necessaria per poter costruire una teoria universale
della gravitazione, le leggi di Keplero sono leggi osservative valide in li-
nea di principio solamente per il sistema in cui i pianeti ruotano attorno

Francesco Saitta, Pordenone Settembre 2019 105


Gravitazione universale

al sole, noi stiamo immaginando che siano valide per qualsiasi sistema
gravitazionale. Vale dunque:

~FSP = −~FPS
4π 2 mS 4π 2 m P
=
k P R2 k S R2
mS mP
=
kP kS
mS k S = mP k P

possiamo quindi definire una costante più generale k = mS k S = m P k P da


poter inserire nelle formule (6.2.3) e (6.2.4), ottenendo:

4π 2 mS m P
FSP =
k R2
4π 2 mS m P
FPS = .
k R2

L’intuizione di Newton fu quella di assumere 4π 2 /k come costante uni-


versale, la costante di gravitazione universale G e dire che due masse
qualsiasi m1 ed m2 si attraggono con una forza del tipo:

~F = G m1 m2 r̂ (6.2.5)
r2

La costante di gravitazione universale fu calcolata per la prima volta


da Henry Cavendish (Nizza, 1731 - Londra, 1810) con l’utilizzo di una
bilancia a torsione (Cavendish, 1798), ed al giorno d’oggi si assume essere

Nm2
G = 6, 67 · 10−11 (6.2.6)
kg2

6.3 Il concetto di campo

Parlando di forza gravitazionale ci troviamo per la prima volta di fronte


al concetto di campo in fisica. Questo concetto è di fondamentale impor-
tanza per il ruolo che ha nell’impostazione di tutta la fisica moderna, e
per questo cercheremo di darne una semplice spiegazione in queste righe.
Il concetto di campo nasce in fisica per dare una spiegazione plausibi-
le ad un fatto provato ma di difficile spiegazione: esistono delle forze a
distanza, ovvero che agiscono senza bisogno di contatto, di un mediato-
re che faccia imprimere la forza da un corpo ad un altro. Una forza di
contatto è semplice da comprendere: studiando il fenomeno del calcio di
punizione di un calciatore ad esempio, è facile capire come il giocatore
riesca ad imprimere una certa forza sul pallone: colpendola tramite il pie-
de e lo scarpino! Ma se lasciamo cadere una penna dalla mano, come fa
la terra a ”comunicare” alla penna di dirigersi verso il centro della terra
con una ben definita accelerazione ~g? Nella storia della scienza, fino agli
studi di Newton sulla gravitazione, tutti gli scienziati ed i filosofi erano
fermamente convinti che una forza dovesse necessariamente essere forza
di contatto; con la formalizzazione della legge di gravitazione universale

Francesco Saitta, Pordenone Settembre 2019 106


Gravitazione universale

invece si ebbe la prima teorizzazione di una forza a distanza: questo fatto


fu difficilmente digerito dalla comunità scientifica e trovò completa solu-
zione solo con la formalizzazione del concetto di campo di forza dopo gli
studi sull’elettromagnetismo alla fine del XIX secolo. Per capire di che
cosa si tratta torniamo all’esempio precedente: come fa la penna a ”sa-
pere” come muoversi verso la terra? L’idea fondamentale è che la terra,
per il solo fatto di esistere, modifica lo spazio nel suo intorno, in modo
da far percepire a tutte le masse circostanti la sua presenza; in che modo?
Tramite il campo gravitazionale. La modifica dello spazio da parte di una
massa può essere descritta dalla figura (6.4).
Figura 6.4: Nella figura è raffigu-
Modificando lo spazio intorno a se di fatto la massa sta lasciando delle rato in basso lo spazio senza mas-
informazioni sul tipo di forza (per questo campo di forze) di cui risentirà se: un piano, un’infinita distesa
piatta; immaginando di posizionar-
un’altra massa nel caso in cui dovesse posizionarsi in una certa posizione: si in questa distesa tutti i punti so-
~F = m~g, dove ~F è la forza, m la massa “di prova” (l’ipotetica massa che no equivalenti: non vi è differen-
si trova nelle vicinanze della massa “sorgente”, che genera il campo) e za alcuna nel posizionarsi in uno
qualsiasi dei punti del piano. Nel-
~g il campo. In questo modo è facile capire come si risolve, per lo meno la figura in alto invece è raffigura-
da un punto di vista concettuale, il problema della forza a distanza: il to lo spazio modificato da una mas-
sa; non c’è più un piano, lo spa-
“mediatore” della forza, ciò che permette alla forza di essere impressa da zio non è più uguale a se stesso
un corpo ad un altro, è il campo stesso. Il campo è dunque un vettore in ogni punto: posizionarsi in un
che in ogni punto dello spazio circostante la sorgente dà l’informazione punto non equivale a posizionarsi
in un qualsiasi altro, la massa ha
sul tipo di forza (modulo, direzione e verso) di cui risentirà una massa modificato lo spazio intorno a sé!
collocandosi in quel punto dello spazio. Ogni qual volta ci si trova in
presenza di una forza a distanza si definisce il relativo campo che funziona
sempre nello stesso modo, dando l’informazione di forza ad ogni punto
dello spazio, come descritto in figura(6.5); ciò che cambia da caso a caso
sono le sorgenti della forza: nel caso gravitazionale sono le masse, nel
caso elettrico (che non abbiamo ancora studiato) sono le cariche, e così
via. Dal punto di vista matematico un campo di forze è una funzione che

Figura 6.5: Con C ~ è indicato un


campo generico, generato da una
certa proprietà della materia, iden-
tificata con p (massa, carica,...). La
figura mostra l’effetto generale di
un campo di forza: esso dà l’in-
formazione di forza per ogni cor-
po dotato della proprietà p. Il vet-
tore campo dipende dal punto del-
lo spazio in cui si trova, per cui
l’informazione è relativa alla forza
di cui risentirebbe un corpo in quel
determinato punto dello spazio.

associa ad un vettore tridimensionale (la posizione nello spazio (x,y,z)) un


vettore tridimensionale (il campo stesso). Abbiamo definito un campo di
forze, in quanto si tratta in fisica di una quantità di importanza molto
rilevante, ma in modo analogo si possono definire campi di velocità, di
accelerazione, o di qualsiasi quantità vettoriale di interesse. Nella fisica
moderna il concetto di campo assume un ruolo fondamentale, in quanto
dal punto di vista concettuale un campo di forza porta con sé tutte le

Francesco Saitta, Pordenone Settembre 2019 107


Gravitazione universale

informazioni legate alla forza a prescindere dal soggetto che “subisce” la


forza stessa: a differenza della fisica classica e del percorso che abbiamo
seguito, per cui prima si studia la forza e poi si ricava il campo relativo,
in fisica moderna ogni qual volta si voglia studiare un interazione se ne
studia prima il campo ed in seguito si focalizza sulla forza che esso genera.

6.3.1 Linee di campo


Spesso rappresentare un campo vettoriale può essere complesso e poco
chiaro: i vettori si accavallano e non si capisce l’andamento reale del cam-
po. Per questo motivo si utilizza una convenzione assodata nel tempo per
rappresentare i campi di forza in fisica: le linee di campo. Le linee di
campo sono delle linee che vengono tracciate nel piano o nello spazio con
lo scopo di rappresentare graficamente l’andamento del campo. Esistono
delle semplici regole per disegnarle:
1. Le linee di campo sono fatte in modo tale che il campo sia sempre
tangente ad esse;

2. La densità di linee di campo è direttamente proporzionale all’intensità


del campo;

3. Le linee di campo sono orientate secondo l’orientazione del campo


stesso.
In figura(6.6) vediamo un chiaro esempio di linee di campo: le linee di
campo gravitazionale generate da una massa M.

6.4 Il campo gravitazionale

Nel caso gravitazionale dunque, unendo la legge di gravitazione uni-


versale con la (6.5) otteniamo la seguente espressione per il campo, che
Figura 6.6: Linee di campo gra-
chiameremo ~g: vitazionale generate da una massa
m m
~g = G 2 r̂ (6.4.1)
r
Notiamo come le unità di misura del campo gravitazionale siano N/kg
ovvero m/s2 , il campo gravitazione è un’accelerazione, come possiamo
dedurre anche dal confronto tra la relazione generale (6.5) e la seconda
legge della dinamica. Una delle applicazioni per noi più interessanti del-
l’equazione (6.4.1) è la sua applicazione alla terra: proviamo a calcolare
quanto vale il campo gravitazionale terrestre e quanto cambia ad esempio
passando dal livello del mare alla cima del monte Everest (h=8848m):
MT 5, 97 · 1024
~g(r = R T ) = G 2
r̂ = 6, 67 · 10−11 r̂ = 9, 81r̂ (6.4.2)
RT (6, 37 · 106 )2
MT 5, 97 · 1024
~g(r = R E ) = G 2
r̂ = 6, 67 · 10−11 r̂ = 9, 78r̂
( RT + h) (6, 37 · 106 + 8, 848 · 103 )2
(6.4.3)
Nel calcolo non abbiamo riportato le unità di misura per non appesantire
la notazione, resta vera l’osservazione fatta precedentemente per cui l’uni-
tà di misura del campo gravitazionale è quella di un’accelerazione. Come

Francesco Saitta, Pordenone Settembre 2019 108


Gravitazione universale

possiamo notare l’affermazione che abbiamo sempre utilizzato fin’ora l’ac-


celerazione di gravità vale 9,81 m/s2 su tutta la superficie terrestre è senz’altro
una buona approssimazione anche considerando la legge di gravitazione
universale di Newton, il valore del campo varia dal livello del mare alla
cima del monte più alto della terra solamente dello 0,3%: per la maggior
parte dei fenomeni naturali che osserviamo quotidianamente od esperi-
menti che possiamo immaginare di compiere in un laboratorio terrestre g
è considerabile costante!

6.5 L’energia potenziale gravitazionale

Quando un fisico si trova di fronte ad una nuova forza una delle prime
domande che sorgono nella sua mente è: la forza che sto considerando è con-
servativa? Posso trovare un’espressione per la sua energia potenziale? Vedremo
in queste righe come la forza gravitazionale è una forza conservativa e
daremo l’espressione dell’energia potenziale gravitazionale. A differen-
za dei casi di forze studiate durante lo scorso anno e riprese anche nel
capitolo (2) la forza gravitazionale ha un’espressione matematica tale per
cui non siamo ancora i grado di affrontare la dimostrazione completa del-
l’espressione che scriveremo: daremo qui una giustificazione, speriamo
ragionevole, del fatto che la forza gravitazione è una forza conservativa,
rimandando al momento in cui nel corso di matematica si affronterà il
calcolo integrale una più soddisfacente dimostrazione.
Immaginiamo di avere una massa m1 che genera un campo gravitazio-
nale in cui la massa m2 si trova immersa. La massa m2 si trova inizialmente
nel punto A ed il nostro proposito è quello di darci ragione del fatto che,
volendo spostare la massa in un qualsiasi altro punto B dello spazio, il
lavoro che la forza gravitazionale compie non dipende dal percorso effet-
tuato dalla massa m2 ; la situazione è descritta dalla figura (6.7). Cerche- Figura 6.7: Tre possibili percor-
si per portare la massa m2 da
remo ora di dare ragione del fatto che il percorso 1 ed il percorso 2 della A e B. Il lavoro fatto dalla for-
figura sono equivalenti in termini di lavoro fatto dalla forza gravitazio- za gravitazionale per spostare la
massa m2 dipende dal percorso?
nale, a quel punto dovremo credere che ogni altro percorso è equivalente
ad essi, anche il percorso 3, che pur sembra molto diverso... Per quanto
riguarda il percorso 1 possiamo dire che la forza è sempre parallela alla
traiettoria e quindi il lavoro, come detto nel capitolo (2) parlando di forze
variabili, è dato dall’area della curva definita dalla forza dal raggio r A al
raggio r B , come illustrato in figura (6.8)
Ora basta notare che in un qualsiasi pezzo di traiettoria circolare con
centro m1 , come la prima parte del percorso 2, la forza è sempre perpen-
dicolare alla traiettoria (ricordiamo che per intervalli di tempo abbastan-
za piccoli possiamo confondere la corda che esprime lo spostamento con
l’arco di circonferenza che esprime la traiettoria e la tangente alla cur-
va in quel punto): il lavoro in quel tratto di traiettoria è quindi nullo, e
nel secondo tratto è sempre dato dall’area di figura (6.8). Abbiamo co-
sì dimostrato che il lavoro della forza gravitazionale lungo il percorso 1
è uguale al lavoro lungo il percorso 2, ed in generale uguale a qualsiasi
percorso che preveda una combinazione di tratti circolari con centro m1 e
Figura 6.8: Lavoro come area
tratti radiali. L’atto di fiducia che richiesto ora è quello di credere che sia della funzione F (r )
dimostrabile l’indipendenza del lavoro per ogni percorso che colleghi A e

Francesco Saitta, Pordenone Settembre 2019 109


Gravitazione universale

B e che l’espressione che si ottiene per l’energia potenziale gravitazionale


è:
m m2
U = −G 1 (6.5.1)
r

Il lavoro dipende quindi solo dalle distanze iniziale e finale della massa
m2 rispetto alla massa m1 :

1 1
L = −∆U = Ui − U f = Gm1 m2 ( − ) (6.5.2)
rf ri

6.6 Pianeti e satelliti

In questo paragrafo studieremo come i concetti studiati per la gravitazio-


ne universale possono essere utilizzati per analizzare la dinamica di un
sistema gravitante a due masse, la prima di massa molto maggiore della
seconda, come può essere un pianeta con uno dei suoi satelliti (naturali
od artificiali) o una stella con un suo pianeta. Affinché un satellite possa
gravitare stabilmente attorno alla massa che lo attrae la forza centripeta
gravitazionale ~Fg deve essere bilanciata dalla forza centrifuga ~Fc , come
descritto dalla figura (6.9).
Dal punto di vista algebrico possiamo immaginare di calcolare la rela-
zione tra la velocità e la distanza che il satellite deve avere per rimanere
in equilibrio, supponendo di conoscere la massa del corpo che genera il
campo gravitazionale sul satellite stesso:
Figura 6.9: Bilanciamento delle
Fc = Fg forze in un sistema gravitazio-
nale a due corpi nel sistema di
mM riferimento del corpo di massa
mac = G 2
r maggiore
v2 mM
m = G 2
r r
v2 mM
m = G 2
r rr
GM
v =
r
Dal punto di vista energetico possiamo quindi sfruttare la relazione tro-
vata per calcolare la relazione tra energia totale del sistema e posizione di
equilibrio del satellite:

1 2 mM 1 GM mM 1 mM
Etot = mv − G = m −G =− G
2 r 2 r r 2 r

6.6.1 I satelliti artificiali della terra


Grazie anche e sopratutto agli studi sulla gravitazione universale l’uomo
ha potuto mettere in orbita dei satelliti che gravitano attorno al nostro
pianeta ed hanno diverse funzioni, tra le quali sopratutto l’astronomia,
la meteorologia, le telecomunicazioni e la navigazione. Il primo satellite
artificiale messo in orbita dagli uomini fu il sovietico Sputnik I nel 1957,
il più famoso probabilmente è stato l’Hubble Space Telescope messo in or-
bita nel 1990 da una collaborazione tra NASA8 ed ESA9 . Le equazioni 8
l’agenzia spaziale statunitense
9
l’agenzia spaziale europea
Francesco Saitta, Pordenone Settembre 2019 110
Gravitazione universale

viste precedentemente mettono in relazione la velocità che devono avere


i satelliti con l’altezza rispetto alla superficie terrestre (h = r − R T ) in cui
devono essere posizionati per essere in equilibrio. Particolare importanza
al giorno d’oggi rivestono due tipi di satelliti:

• Geostazionari:
si definiscono in questo modo satelliti che hanno un’orbita circolare po-
sta sul piano definito dall’equatore, con periodo di rivoluzione uguale
al periodo di rotazione della terra in modo tale da apparire in posizio-
ne fissa da un osservatore solidale con il sistema terrestre. Per avere
tale periodo essi devono essere ad un distanza dalla superficie terrestre
pari ad h = 35863km:
r s s
GM 2πr GMTrot2 GMTrot2
3 3
v= ∧v = =⇒ r = 2
=⇒ h = 2
− RT
r Trot 4π 4π

• Polari:
si definiscono in questo modo satelliti che hanno un’orbita che attra-
versa i poli; sono studiati in modo da sorvolare sempre aree differenti
sotto di sé, acquisendo immagini o video della parte di pianeta sotto di
sé. Essi si trovano tipicamente ad altezze non molto elevate, attorno ai
1000 km di altitudine rispetto al livello del mare.

6.7 Traiettorie ed energia

Abbiamo già notato come l’energia totale di un sistema gravitante a due


corpi pianeta-satellite ha energia totale negativa: in questo paragrafo stu-
dieremo la relazione esistente tra energia di un sistema gravitante a due
corpi e traiettoria dei corpi del sistema. Per semplicità considereremo
sempre uno dei due corpi molti più massiccio del secondo, in modo da
poter considerare inerziale un sistema solidale con esso. Immaginiamo
per esempio di lanciare un oggetto di massa m dalla superficie terrestre.
Il sistema considerato è conservativo, ci è conveniente quindi avere la
descrizione dell’energetica del sistema stesso: esso ha energia potenziale

mM
Ug = − G
RT

(con M la massa ed R T il raggio del pianeta terra) ed un energia cinetica

1 2
Ec = mv
2

, con v la velocità iniziale della massa lanciata. La quantità totale di


energia del sistema è dunque

1 2 mM
Etot = mv − G
2 RT

, composta evidentemente dalla differenza di due quantità positive. Siamo


dunque di fronte a tre possibilità:

Francesco Saitta, Pordenone Settembre 2019 111


Gravitazione universale

• Etot > 0:
in questo caso man mano che la massa si muove allontanandosi dalla
terra sia la sua energia cinetica che l’energia potenziale del sistema si
avvicinano a zero; per la conservazione dell’energia esisterà un momen-
to in cui l’energia potenziale si annullerà e al corpo resterà solamente
energia cinetica: esso dunque sarà libero dall’influenza gravitazionale
terrestre e si potrà muovere nello spazio. Chiameremo un sistema di
questo genere aperto.

• Etot < 0:
in questo caso invece esisterà un momento in cui l’energia cinetica si
esaurirà con una certa quota di energia potenziale: il corpo ricadrà
dunque verso la superficie terrestre. Chiameremo un sistema di questo
genere legato.

• Etot = 0:
il caso limite definisce la cosiddetta velocità di fuga, la minima velocità
che deve avere il corpo per liberarsi dall’influenza gravitazionale terre-
stre (o del pianeta in cui si trova). Imponendo l’equazione Etot = 0 si

trova v f uga = 2GM/R T = 11200m/s.

In generale diremo che un sistema gravitazionale con energia totale nega-


tiva è un sistema legato, i cui elementi cioè continueranno a gravitare gli
uni attorno agli altri ed in generale sono attratti gli uni dagli altri, mentre
un sistema con energia totale positiva è un sistema i cui elementi tende-
ranno ad allontanarsi gli uni rispetto agli altri. Questa regola che non
abbiamo giustificato matematicamente in generale può essere considera-
ta valida per tutte le forze la cui energia potenziale si annulla quando la
distanza tra i corpi che risentono della forza tende ad essere molto gran-
de: sistemi con energia totale negativa sono in generale sistemi attrattivi
e sistemi con energia totale positiva sono in generale sistemi repulsivi.
Proseguendo con il corso di fisica vedremo l’applicazione di questa regola
anche studiando l’elettrostatica.

É interessante a questo punto far notare come da un punto di vista


matematico le diverse possibili orbite di un sistema gravitazionale siano
tutte curve coniche, che come sappiamo furono introdotte dal matematico
greco Apollonio tra il terzo ed il secondo secolo avanti Cristo. Lo stu-
dio delle coniche dopo le analisi degli studiosi greci fu sostanzialmente
abbandonato per più di un millennio quando Keplero ebbe l’intuizione
e fece lo sforzo di analizzarle come possibili curve con cui interpretare i
dati ereditati da Brahe. Keplero compì moltissimi studi matematici sul-
la natura delle coniche e le loro eccentricità e riuscì ad affermare la sua
prima legge, in seguito la formalizzazione della dinamica di sistemi gravi-
tazionali dimostrò come il campo gravitazionale generi in modo naturale
traiettorie coniche a seconda delle caratteristiche di partenza del sistema.

Francesco Saitta, Pordenone Settembre 2019 112


Gravitazione universale

Francesco Saitta, Pordenone Settembre 2019 113


Gravitazione universale

6.8 Esercizi

1. Si calcoli la forza con cui la terra è attratta dal sole.

[F = 3, 3 × 1022 N]
2. Si calcoli la forza con cui la luna è attratta dalla terra.

[F = 1, 97 × 1020 N]
3. Qual è la forza di attrazione gravitazionale tra due persone di massa m = 80 kg alla distanza di un
metro l’una dall’altra?

[F = 4, 3 × 10−7 N]
4. Due masse m1 = 10 kg ed m= 25 kg sono poste ad una distanza d = 5 m l’una dall’altra. Dove deve
posizionarsi una massa m3 = 10 g per essere in equilibrio?

[sulla congiungente le masse m1 ed m2 ad una distanza d = 193 cm da m1 ]


5. Tre masse m A = 20 kg, m B = 20 kg ed mC = 5 g sono poste sui vertici di un triangolo equilatero ABC
di lato l = 2 m. Si calcoli la forza che agisce sulla massa mC posta sul vertice C del triangolo.

[F = 2, 8 × 10−12 N, diretta perpendicolarmente al lato AB del triangolo con verso tale che la massa
sia attratta verso AB]
6. Tre masse m A = 20 kg, m B = 10 kg ed mC = 5 g sono poste sui vertici di un triangolo equilatero ABC
di lato l = 2 m. Si calcoli la forza che agisce sulla massa mC posta sul vertice C del triangolo.

[F = 2, 2 × 10−12 N, formante un angolo α = 259 rispetto la base AB]


7. Tre masse m A = 25 kg, m B = 25 kg ed mC = 25 g sono poste sui vertici di un triangolo isoscele
rettangolo ABC di cateti AC = BC = 2 m. Si calcoli la forza che agisce sulla massa mC posta sul
vertice C del triangolo.

[F = 1, 5 × 10−11 N, diretta lungo la bisettrice dell’angolo AĈB]


8. Tre masse m A = 5 kg, m B = 15 kg ed mC = 25 kg sono allineate con AC = 20 cm, CB = 45 cm e
AB = 25 cm. Si calcoli la forza che agisce sulla massa mC .

[F = 3, 3 × 10−7 N, diretta verso A]


9. Quattro masse m A = 20 kg, m B = 25 kg, mC = 20 kg ed m D = 20 g sono poste sui vertici di un
quadrato ABCD di lato l = 30 cm. Si calcoli la forza che agisce sulla massa m D .

[F = 6 × 10−10 N, diretta lungo la diagonale del quadrato verso B]


10. Si calcoli il peso misurato con un dinamometro di un uomo di massa m = 70 kg che si trovi su una
mongolfiera ad un’altezza h = 11 km sul livello del mare. Se si pesasse con una bilancia, di quanto
crederebbe di essere dimagrito?

[P = 676 N, crederebbe di avere perso m = 1 kg]


11. A che altezza si trova un uomo di massa m = 90 kg che pesandosi ha come risposta dalla bilancia
m = 85 kg?

[h = 150 km]

Francesco Saitta, Pordenone Settembre 2019 114


Gravitazione universale

12. Si trovi il campo gravitazione generato da una massa m = 1000 kg ad una distanza r = 50 m. Di
quale forza gravitazionale risentirebbe un uomo di massa m P = 80 kg se si trovasse ad una distanza r
dalla massa m?

[g = 2, 7 × 10−11 m/s2 , F = 2, 1 × 10−9 N]


13. Si trovi il campo gravitazionale generato dalla terra ad una distanza h = 12000 km da terra.

[g = 2, 7 m/s2 ]
14. A che distanza dalla superficie terrestre il campo gravitazionale generato dal nostro pianeta si di-
mezza?

[h = 2600 km]
15. Qual è il valore del campo gravitazionale lunare sulla superficie terrestre?

[g = 3, 3 × 10−5 m/s2 ]
16. Qual è il valore del campo gravitazionale solare sulla superficie terrestre?

[g = 5, 6 × 10−3 m/s2 ]
17. Si trovi il valore del campo gravitazionale generato da un anello sottile uniforme di massa m = 40
kg e raggio r = 30cm nel suo centro.

[g = 0 m/s2 ]
18. Quale massa genera un campo gravitazione di intensità g = 9, 8 m/s2 ad una distanza di 5 metri?

[m = 3, 7 × 1012 kg]
19. Tre masse m A = 20 kg, m B = 20 kg ed mC = 20 kg sono poste sui vertici di un triangolo equilatero
ABC di lato l = 2 m. Si calcoli il campo gravitazionale generato dalle tre masse sul punto medio del
lato AB.

[g = 4, 4 × 10−10 m/s2 diretto in verticale verso il vertice C]


20. Quattro masse m A = 40 kg, m B = 40 kg, mC = 20 kg ed m D = 20 kg sono poste sui vertici di un
quadrato ABCD di lato l = 30 cm. Si calcoli il campo gravitazionale al centro del quadrato.

[g = 4, 2 × 10−8 m/s2 diretto perpendicolarmente e verso il lato AB]


21. Si calcoli il lavoro necessario per costruire un triangolo equilatero di lato 50 cm con tre masse uguali
m = 10 kg.

[L = 4 × 10−8 J, fatto dalla forza gravitazionale]


22. Si calcoli il lavoro necessario per costruire un quadrato di lato 20 cm con quattro masse uguali m = 20
kg.

[L = 7, 2 × 10−7 J, fatto dalla forza gravitazionale]


23. Tre masse m A = 20 kg, m B = 25 kg ed mC = 10 kg sono poste sui vertici di un triangolo equilatero
ABC di lato l = 1, 5 m. Qual è il lavoro che il campo gravitazionale fa o subisce per portare le masse
su un triangolo equilatero come il primo, ma di lato l = 20 cm?

[L = 2, 8 × 10−7 J, fatto dalla forza gravitazionale]

Francesco Saitta, Pordenone Settembre 2019 115


Gravitazione universale

24. Quattro masse m A = 10 kg, m B = 40 kg, mC = 20 kg ed m D = 15 kg sono poste sui vertici di un


quadrato ABCD di lato l = 50 cm. Qual è il lavoro che il campo gravitazionale fa o subisce per portare
le masse in linea nell’ordine ABCD con AB = BC = CD = 20 cm?

[L = 3, 5 × 10−7 J, fatto dalla forza gravitazionale]


25. Quanto lavoro hanno fatto i motori di un astronave (m = 50000 kg) partita dalla superficie terrestre
quando questa raggiunge quota h = 20000 km?

[L = 23, 7 × 1011 J]
26. A quale altezza dalla superficie terrestre si trova un satellite che ruota attorno al nostro pianeta con
un periodo T = 10 giorni?

[h = 1, 9 × 108 m]
27. A quale velocità si muove un satellite che orbita attorno alla terra ad una quota h = 50000 km sul
livello del mare?

[v = 2642 m/s]
28. Un satellite in orbita attorno alla terra alla velocità v = 200 m/s misura un peso P = 50 N per una
massa m = 50 kg. Il satellite è in equilibrio su un’orbita stabile?

[No, perché...]
29. Dalla cima di un grattacelo alto h = 200 m un tizio vuole lanciare una pallina in modo che riesca
ad entrare in orbita all’altezza da cui è stata lanciata. A quale velocità dovrebbe lanciare la pallina,
supponendo un lancio a velocità iniziale orizzontale, per riuscire nel suo intento?

[v = 7860 m/s]
30. A che altezza arriverebbe un corpo di massa m = 100 kg lanciato dalla superficie terrestre con velo-
cità pari ad un terzo della velocità di fuga? E se il corpo avesse massa m = 20 kg?

[h = 796 km, indipendentemente dalla massa del corpo]


31. A che altezza si trova un satellite di massa m = 500 kg in orbita attorno alla terra, se il sistema
terra-satellite ha energia totale E = −0, 1 J?

[h = 9, 8 × 1017 m]
32. Con che velocità si deve lanciare verso l’alto un oggetto di massa m = 250 g dalla superficie terrestre
affinché raggiunga l’altezza massima h = 10 km?

[v = 440 m/s]
33. Qual è l’energia del sistema terra-satellite geostazionario se il satellite ha massa m = 4500 kg?

[E = −2 × 1010 J]
34. Qual è la velocità di fuga di un oggetto di massa m = 100 kg che viene lanciato da una piattaforma
che si trova ad una quota h = 200 km dalla superficie terrestre?

[v = 10900 m/s]
35. Con quale velocità si deve lanciare un oggetto di massa m = 100 kg verso la luna affinché il corpo
arrivi sulla superficie lunare con velocità nulla? Si supponga che il moto dell’oggetto sia rettilineo e

Francesco Saitta, Pordenone Settembre 2019 116


Gravitazione universale

senza attrito.

[v = 11119 m/s]
36. Find the gravitational field of Saturn on the planet’s surface.

[g = 10, 3 m/s2 ]
37. Find the potential gravitational energy of the system earth-moon.

[U = −7, 8 × 1020 J]
38. A satellite is gravitating around the earth at a velocity v = 50 km/h. What is its heigh with respect
to the earth surface?

[h = 2 × 1012 m]
39. Two masses m1 = 40 kg and m2 = 20 kg are fixed at a distance d = 4 m. What is the gravitational
strength acting on a third mass m3 = 1 kg positioned in the middle of the line defined by m1 and m2 ?

[F = 3, 3 × 10−10 N towards m1 ]
40. Find the gravitational strenght due to the sun acting on the farest planet (from the sun) of the solar
system.

[F = 6, 4 × 1020 N]

Francesco Saitta, Pordenone Settembre 2019 117


7 Termodinamica

Come vedremo introducendo il primo principio della termodinamica in


uno dei prossimi paragrafi questa parte di fisica si è sviluppata nel dician-
novesimo secolo per opera di diversi scienziati dell’epoca in relazione allo
sviluppo tecnologico della rivoluzione industriale di quegli anni. Per par-
lare di termodinamica richiamiamo alcuni concetti visti durante il primo
biennio nello studio di temperatura e calore:
• Temperatura: grandezza fisica legata alle sensazioni di caldo e freddo,
che possiamo misurare con un termometro sulla base degli effetti di
una sua variazione su corpi liquidi e solidi. La presente definizione è
una definizione operativa che necessita di un ampliamento concettua-
le per comprendere davvero quale sia la natura di questa grandezza
fisica con cui abbiamo quotidianamente a che fare e che troviamo co-
sì complicato definire in modo soddisfacente. La sua unità di misura
nel sistema internazionale sono i Kelvin, la cui conversione con i gradi
centigradi da noi più utilizzati è data da T (K ) = t(C ) + 273.15. In-
dicheremo sempre la temperatura misurata in gradi centigradi con la
lettera minuscola t, misurata in Kelvin con la lettera maiuscola T.

• Gas perfetto: insieme di particelle o molecole con le seguenti caratteri-


stiche:

1. le molecole hanno dimensioni trascurabili rispetto alla media della


loro distanza reciproca e sono indistinguibili le une dalle altre: sono
approssimabili a punti materiali identici;
2. le molecole sono moltissime si muovono in modo disordinato e
casuale;
3. le molecole non risentono a distanza le une dalle altre (non sono sog-
gette ad attrazioni gravitazionali, elettriche,...): si parla di molecole
non interagenti;
4. le molecole urtano solamente in modo elastico, tra loro e con le
pareti del recipiente in cui eventualmente si trovano;

• Numero di moli e numero di Avogadro: la mole è una delle sette unità


di misura fondamentali ed esprime la quantità di sostanza; questa uni-
tà di misura è pratica dal punto di vista chimico e fisico delle molecole
perchè evita di lavorare con numero molto grandi (il numero di mo-
lecole/atomi) o numeri molto piccoli (le masse molecolari/atomiche).
Una mole rappresenta la quantità di materia pari ad un numero di Avo-
gadro di elementi (NA = 6, 022 × 1023 ), altrimenti definita anche come

Francesco Saitta, Pordenone Settembre 2019 119


Termodinamica

la quantità di sostanza equivalente (che contiene tante molecole/atomi)


a 12g di 12 C.

• Parametri di stato: i parametri fisici che definiscono in modo completo


la situazione macroscopica di un gas; essi sono la pressione P, il volu-
me V, la temperatura T ed il numero di moli n.

• Legge di Boyle: un gas perfetto che subisce una trasformazione a tem-


peratura costante segue la seguente legge PV = P0 V0

• Prima legge di Gay-Lussac: un gas perfetto che subisce una trasforma-


zione a pressione costante segue la seguente legge V = V0 (1 + αt) =
V0 αT, da cui si può ricavare α = 1/273, 15.

• Seconda legge di Gay-Lussac: un gas perfetto che subisce una trasfor-


mazione a volume costante segue la seguente legge P = P0 (1 + βt) =
V0 βT, da cui si può ricavare β = α = 1/273, 15.

• Equazione di stato dei gas perfetti: un gas perfetto che subisce una
qualsiasi trasformazione segue la seguente legge: PV = nRT, con n
il numero di moli e R ∼ 8, 31J/(mol K) la costante universale dei gas
perfetti

• Calore: grandezza fisica che determina la variazione di temperatura o


il cambiamento di stato fisico (solido, liquido, aeriforme) di un corpo.
La sua unità di misura è la caloria, la quantità di calore necessaria per
far innalzare di un grado centigrado, da 14,5 a 15,5, la temperatura di
un grammo d’acqua. Ogni sostanza ha un particolare calore specifico,
necessario per far innalzare di un grado la temperatura un kilogram-
mo di sostanza cs = Q/(m∆T ), un calore latente di fusione, necessario
per far passare dallo stato solido a quello liquido un kilogrammo di so-
stanza Q f = Q/m ed un calore latente di evaporazione, necessario
per far passare dallo stato liquido a quello aeriforme un kilogrammo
di sostanza Qv = Q/m

• Equivalenza tra calore e lavoro: nel 1850 lo scienziato inglese James


Prescott Joule fece il suo famoso esperimento che abbiamo descritto
nei dettagli nel corso del primo biennio dimostrando come facendo
del lavoro meccanico su una sostanza esso può essere completamente
trasformato in variazione di temperatura della sostanza stessa trasfe-
rendole quindi calore. Joule calcolò l’equivalente meccanico del calore,
ovvero la conversione tra lavoro esterno fatto sul sistema e calore tra-
sferito alla sostanza: per ogni caloria trasferita al sistema si fanno 4,186J
di lavoro meccanico. La portata concettuale di questa equivalenza è il
fatto di poter accostare il calore al lavoro e all’energia, affermando che

Francesco Saitta, Pordenone Settembre 2019 120


Termodinamica

sono possibili trasformazioni dell’uno negli altri e viceversa, secondo


l’equivalenza 1cal = 4, 186J.

• Principio zero della termodinamica: Se un corpo A ha la medesima


temperatura di un corpo B ed il corpo B ha la medesima temperatura di
un corpo C, allora A e C hanno la stessa temperatura. Questo principio
che sembra banale permette però di definire l’equilibrio termico tra
corpi che non possono essere messi a contatto tra loro, con l’uso di un
terzo corpo che può essere ad esempio un termometro.

• Convenzioni di segno per calore e lavoro: evidenziamo qui inizial-


mente le convenzioni per il segno di calore e lavoro rispettivamente
assorbito/ceduto e fatto/subito:

– Q > 0 - calore assorbito dal gas;

– Q < 0 - calore ceduto dal gas;

– L > 0 - lavoro fatto dal gas;

– L < 0 - lavoro subito dal gas.

Prima di proseguire con i due principi della termodinamica che ci permet-


teranno di capire come trattare la dinamica di sistemi in cui consideriamo
anche lo scambio di calore nel bilancio energetico studieremo un model-
lo microscopico per i gas perfetti, che ci aiuterà nella comprensione della
natura di grandezze come la temperatura e l’energia cinetica associata ad
un gas.

7.1 Teoria cinetica dei gas perfetti

Il modello che costruiremo in questo paragrafo per i gas perfetti ci permet-


terà di capire la natura microscopica di alcune grandezze termodinamiche
che in genere consideriamo come riassuntive del sistema (pressione, tem-
Figura 7.1: Gas perfetto in una
peratura); questo modello è un primo esempio di quella branca della fisica scatola: sono qui rappresentate
chiamata fisica statistica che cerca di studiare un sistema complesso come solo 11 particelle, evidentemen-
te nella nostra schematizzazione
somma o unione di moltissimi sistemi elementari: un intero libro di una
il numero di particelle di gas è
delle collane più famose di fisica teorica 1 molto più elevato.
fu dedicato a questa parte di scienza che noi studieremo solamente nel- 1
(Landau e Lifshitz, 1999b)
la sua parte più semplice ed iniziale.

Consideriamo quindi un gas perfetto in una scatola cubica di lato l:


ogni particella i si muoverà quindi in una direzione casuale con una certa
velocità ~vi , come rappresentato in figura (7.1). Fissando un sistema car-
tesiano in un angolo della scatola possiamo scomporre le velocità di ogni
particella nelle tre direzioni ~vi = (vix ; viy ; viz ) ed analizzare la dinamica
del problema una direzione alla volta. Secondo le ipotesi di gas perfetto
le particelle urtano sempre in modo elastico con le pareti della scatola: co-
Figura 7.2: Urto di una particel-
me descritto dalla figura (7.2), ogni volta che una particella urta con una
la di gas contro una parete della
parete la componente perpendicolare alla superficie della sua quantità di scatola.

Francesco Saitta, Pordenone Settembre 2019 121


Termodinamica

moto si inverte e dunque la particella imprime sulla parete una forza ~F⊥

~F⊥ = − ∆~q⊥ = 2m~v⊥


∆t ∆t
2 Le particelle si muovono senza interazioni interne, le componenti della 2
Da notare come la forza im-
loro velocità restano sempre costanti tranne quando si invertono per l’urto pressa sulla particella è data
dalla variazione della quantità
con una parete, l’urto con la parete avviene quindi ogni ∆t = 2l/v⊥ e la di moto della particella rispet-
forza diventa to al tempo, e dunque la forza
mv2⊥ che la particella imprime sulla
F⊥ = parete è data, per il terzo prin-
l cipio della dinamica, dalla sua
Immaginando quindi che nella scatola ci siano N particelle la forza media opposta
che viene applicata su una parete dal gas è data da
N
m
Fm =
l ∑ v2⊥i
i =1

e dunque la pressione esercitata dal gas su una parete della scatola è data
dal rapporto tra la forza media applicata e la superficie della parete:
N N
m m
P=
l3 ∑ v2⊥i = V ∑ v2⊥i
i =1 i =1

con V = l 3 il volume della scatola. A questo punto notiamo come nelle


ipotesi di gas perfetto e moto casuale le componenti di velocità del gas
nelle tre direzioni cartesiane x, y, z devono provocare la stessa pressione
in ogni parete; si dovrà avere
N N N N
∑ v2xi = ∑ v2yi = ∑ v2zi (= ∑ v2⊥i )
i =1 i =1 i =1 i =1

e quindi
N N N N N N
∑ v2i = ∑ (v2xi + v2yi + v2zi ) = ∑ v2xi + ∑ v2yi + ∑ v2zi = 3 ∑ v2⊥i
i =1 i =1 i =1 i =1 i =1 i =1

da cui
N
m
P=
3V ∑ v2i
i =1
N
m
PV =
3 ∑ v2i
i =1

A questo punto possiamo utilizzare la legge di stato dei gas perfetti per
dare un’interpretazione microscopica della temperatura, in funzione delle
velocità delle particelle di gas:
N N
m m
nRT =
3 ∑ v2i =⇒ T = 3nR ∑ v2i ,
i =1 i =1

che sostanzialmente lega la temperatura di un gas con il grado di agita-


zione meccanica delle particelle che lo compongono, per questo spesso si
parla di temperatura come misura dell’agitazione termica del gas; la co-
stante di proporzionalità può essere riscritta ricordando che il numero di
moli è uguale al numero di molecole fratto il numero di Avogadro:
N
N N m
n=
NA
=⇒ T = A
3NR ∑ v2i ,
i =1

Francesco Saitta, Pordenone Settembre 2019 122


Termodinamica

Questo passaggio sembra complicare la formula, ma ci permette di in-


travedere la velocità quadratica media all’interno della formula; si definisce
velocità quadratica media di un gas:
s
∑iN=1 v2i
vqm =
N

La velocità quadratica media di un gas, vqm , è un numero che ci fornisce


informazioni sulla media dei moduli delle velocità delle particelle: non
tiene infatti conto della direzione delle velocità ma solamente dei suoi
moduli. È inoltre interessante perché legata all’energia cinetica media del
gas, Ec :
∑ N 1 mv2 1
Ec = i=1 2 i = mv2qm
N 2
Ecco che sostituendo quest’espressione nella relazione ottenuta preceden-
temente per la temperatura otteniamo:

2NA
T= Ec (7.1.1)
3R
la temperatura è dunque una grandezza fisica direttamente proporzionale
all’energia cinetica media del gas! Questa relazione conferma e quantifica
in modo preciso l’intuizione di temperatura legata all’agitazione termica
e quindi al movimento delle particelle componenti il gas. L’equazione
(7.1.1) è solitamente espressa come:

3
Ec = kB T (7.1.2)
2

con k B = R/NA ∼ 1, 38 × 10−23 J/K, la costante di Boltzmann. Notiamo


anche come la legge di stato dei gas perfetti possa essere scritta in termini
formalmente identici con la costante universale dei gas perfetti

PV = nRT

oppure con la costante di Boltzmann

PV = Nk B T

Questi due modi apparentemente uguali per scrivere la legge indicano pe-
rò due visioni opposte: la prima una visione macroscopica che coinvolge
il numero di moli del gas, mentre la seconda una visione microscopica che
coinvolge invece il numero esatto di particelle che compongono il gas.

7.1.1 Principio di equipartizione dell’energia


L’equazione (7.1.2), oltre a dare significato fisico alla temperatura, esprime
il punto di partenza per i ragionamenti che hanno portato alla formula-
zione di un importante principio della fisica statistica, il quale asserisce
che l’energia totale di un gas si distribuisce equamente sui gradi di libertà delle
molecole che lo costituiscono; ad ogni grado di libertà corrisponde una quantità
di energia cinetica pari a 1/2k B T. Senza entrare nei dettagli diciamo qui

Francesco Saitta, Pordenone Settembre 2019 123


Termodinamica

che per grado di libertà di un corpo rigido in fisica si intende il numero


minimo di variabili indipendenti tra loro con le quali è possibile definire
la posizione e l’orientamento del corpo stesso. Nel caso di punti materia-
li non vincolati ogni punto materiale porta con sé 3 gradi di libertà, ma
già una molecola biatomica ha 5 gradi di libertà: i due punti materiali
avrebbero in tutto 6 gradi di libertà, ma sono vincolati a stare ad una certa
distanza media, il che riduce i gradi a 5. Per quanto riguarda il nostro
interesse è importante sapere che un gas composto da molecole monoa-
tomiche ha 3 gradi di libertà, biatomiche 5 e triatomiche 7. È importante
notare invece la portata concettuale di questo principio: ancora una volta
la natura si comporta in modo simmetrico e matematicamente elegante
distribuendo in modo che potremmo definire democratico l’energia nelle
diverse coordinate che definiscono il gas e di come la sua energia cineti-
ca media dipenda da quanto si muovono le molecole al suo interno e da
come sono fatte (e quindi dai modi in cui si possono muovere). Eviden-
temente quindi la (7.1.2) può essere generalizzata, per un gas con f gradi
di libertà:
f
Ec = k B T (7.1.3)
2

7.2 First principle of thermodynamics

7.2.1 Introduction to Thermodynamics

Thermodynamics was born in the early nineteenth century, during the


industrial revolution, to improve the efficiency of steam engines. Then,
thanks to scientists as James Joule (Saldford, 1818 - Sale, 1889), Rudolf
Clausius (Koslin, 1822 - Bonn, 1888), William Thomson ”Lord Kelvin”
(Belfast, 1824 - Largs, 1907), Sadi Carnot (Parigi, 1796 - Parigi, 1832), Lud-
wig Boltzmann (Vienna, 1844 - Duino, 1906), thermodynamics has become
the branch of physics that studies the exchange of energy between a sy-
stem and its environment. The starting point for the work of these scien-
tists of thermodynamics has been their discovery that the heat is not a
fluid-like substance, but is a measure of the energy exchanged in thermo-
dynamical processes. Joule’s experiment, we studied last year, pointed out
how mechanical work done on a system can be tranformed into heat fin-
ding the equivalence between heat and mechanical work: 1Kcal = 4186J.
Once accepted this fact has been natural to study all physical phenomena,
in particular those involving gases, taking into account heat in the energy
exchanges: this led to the formulation of the first law of thermodynamics,
which we will analyze in detail in these notes, stating that the total energy
is conserved regardless of the nature of forces involved in the considered
system. Using the first principle of thermodynamics it was possible to
study many kinds of steam engines and thermal machines, from an ener-
getic point of view, trying to maximize their efficency. We will analyze in
detail the thermal machines in the next section, but we have to say here
that a thermal engine is a tool that converts heat into mechanical work,
and was discovered by physicists such as Carnot, Clausius and Kelvin,
that it is impossible to take a certain amount of heat and transform all of

Francesco Saitta, Pordenone Settembre 2019 124


Termodinamica

it into mechanical work: in other words, it is impossible to have a ther-


mal machine with a 100 % efficency. This is only one formulation of the
second law of thermodynamics: in the 1860s Clausius introduced entropy
as a ratio of heat to temperature, and stated the Second Law in terms of
the increase of this quantity. Boltzmann then gave a statistical intepre-
tation of entropy, as the ”disorder degree” of a system, and the second
law became the law of the increasing disorder in the Universe. In this
way, the second law has become important not only from the ”thermo-
dynamic” point of view, but also from the philosophical one; according
to thermodynamics, the universe slowly will die because all the reactions
within it will lead to a uniform heat distribution, all warm regions (stars)
of the universe will disappear and so life (human life in particular): this
is called the ”heat death” of the universe. As we shall see in a couple
of years, during the twentieth century many new physical theories were
developed in disagreement or in completion of all the classical theories:
in particular, quantum mechanics, relativity and modern cosmology can
explain different scenarios for the evolution of the universe.

7.2.2 Thermal Machines


We define as thermal machine any tool that, by means of thermodynamic
processes, transforms heat into mechanical work. This general definition
needs to be schematized to be described in a scientific way, anche the
scheme is the following:

• the system is a gas inside a cylinder, closed by a piston;

• moving the piston the gas can change its volume;

• the gas can exchange heat with external sources;

• an ideal source, heat reservoir, is a source with heat capacity C = ∞: it


does not change temperature even exchanging heat with the system;

• the overall thermodynamical process as to be cyclic, the gas has to


became again to its initial condition to restart the process.

The simplest thermal machine we can imagine is described in fig.(7.3),


a black box where all the thermodynamical processes take place, absor-
bing heat from a hot source, giving heat to a cold source, and produ-
cing mechanical work. Then, since heat is a kind of energy, it has to be
L = Q2 − Q1 . To define and study precisely a thermal machine we ha-
ve to understand which is the energy involved and how this energy can
turn into mechanical work, that is what we are going to study in the next
paragraphs.

7.2.3 Internal Energy


As already seen in the study of the kinetic model for an ideal gas, tempe-
rature is the macroscopic effect of the average kinetic energy of the gas.
Since for an ideal gas there is no potential energy (there are no interac-
tions, the gas is ”free”), there is no ”structure” in the gas molecules and Figura 7.3: A generic thermal
machine

Francesco Saitta, Pordenone Settembre 2019 125


Termodinamica

they interact just by means of elastic scattering, the kinetic energy is sim-
ply mv2 /2 for each molecule and can be considered as the total energy of
the gas. This total average energy of internal molecular motion is called
internal energy and should always be taken into account when considering
the conservation of energy.
The internal energy of an ideal gas of N molecules can be expressed in
terms of the average kinetic energy of the molecules

3
U = N h Ek i = nRT;
2

so at any change on the gas temperature it will correspond a change on


the internal energy ∆U = 32 nR∆T.
This energy is also interpreted in terms of a heat exchange: U = Cv T,
where Cv is called the “heat capacity at constant volume”, the heat neces-
sary to change the temperature of the gas by 1 degree through a process
with constant volume: it equals 23 nR for monoatomic gases, 52 nR for bia-
tomic gases, it holds the same rule of the equipartion theorem. We define
also the “molar heat capacity at constant volume”: cv = Cv /n the heat
necessary to change the temperature of one mole of the gas by 1 degree
through a process with constant volume; we will see that this definition is
useful studying the conservation of energy in thermodynamics.
We also notice here that the internal energy is a function of state, i.e. it
depends only on the temperature of the gas, it is defined for each point in
the Clapeyron space of the gas.

7.2.4 Heat capacities of gases


We studied that the heat exchange between two bodies or fluids is regu-
lated by
Q = C∆T,

where C is a constant depending on the bodies or fluids called heat capa-


city. We are going to study in these pages that the heat exchange between
gases (or between gases and a body) depend on the particular process
acting on the gas. In this perspective is simple to understand that the
heat capacity of a gas is not a constant but depends on the thermodyna-
mical process the gas is involved into; we define two heat capacities in
particular:

1. heat capacity at constant volume Cv - if the gas is involved into an


isochoric process the heat exchange is given by

Q = Cv ∆T

2. heat capacity at constant pressure C p - if the gas is involved into an


isobaric process the heat exchange is given by

Q = C p ∆T

Francesco Saitta, Pordenone Settembre 2019 126


Termodinamica

7.2.5 Mechanical work of gases


The internal energy of a system can be increased at the expense of me-
chanical energy as was demonstrated in Joule’s experiment: there exist
machines that produce mechanical work depending on what processes
are involved in their functioning. We will always refer to a gas ( with
state parameters ( p, V, T ) representing pressure, volume and temperature
) inserted in a cilinder with a base surface S; the piston will move into
the cylinder under the effect of external forces or the pressure of the gas
itself. Therefore the mechanical work done by the gas will be connected
with the motion of the piston: dL = ~F · d~s = F ∗ ds = p ∗ S ∗ ds = pdV
(general expression even if the force is not constant). As in mechanics the
work is represented by the area under the curve F(s) in the s-F plane or,
as in our case, the area under the curve p(V) in the V-P plane (Clapeyron
plane).

Figura 7.4: A generic cylinder


filled with gas and the represen-
tation of the work made by the
gas itself on the piston

Francesco Saitta, Pordenone Settembre 2019 127


Termodinamica

7.2.5.1 Isobaric processes


An isobaric process (fig.(7.5)) is a thermodynamical process in which the
pressure stays constant. Since the pressure is constant the force acting on
the system is constant, therefore the work will be: L = ~F ·~s = p ∗ S ∗ s =
p ∗ ∆V.

Figura 7.5: An isobaric process


in the Claperyron Plane

In our practical example we would have a movable piston in a cylinder,


so that the pressure inside the cylinder is always at the pressure given by
the piston and the atmosphere. An increase in the temperature of the gas
would imply an increase in the volume of the piston, in such a way to
mantain the same pressure given by the piston.

Francesco Saitta, Pordenone Settembre 2019 128


Termodinamica

7.2.5.2 Isothermal processes


An isothermal process is a process where the gas is maintained at the same
temperature, being in contact with a huge body called “heat reservoir”, a
body with C = +∞. The temperature of the gas remains constant because
the heat is free to flow from the gas to the heat reservoir. In an isothermal
expansion (fig.(7.6)), if the system is closed, the pressure will be inversely
proportional to the volume: PV = nRT = P0 V0 . In this case the work
as a more complicated expression than the isobaric one: dL = p ∗ dV =
nRT Vf 3
V dV ⇒ L = nRT ln Vi .
3
We have to define here the ma-
thematical function called loga-
rithm y = ln( x ). This function
Figura
will be formally
7.6: An isothermal
defined during
process
next yearin in
thethe
Claperyron Plane
math class; for
our purposes we can say that
y = ln( xx2 ), x1 ∈ R+ , x2 ∈ R+ ,
1
is the area between the x-axis
of the cartesian plane and the
function y = 1/x between x1
and x2 . At the moment you can
easily calculate the logarithm of
a number using your electronic
calculators.

In our practical example we would have the system immersed in a


large constant-temperature bath (imagine a huge mixture of water and ice
at 273K). Any work performed by the system will be lost to the bath, but
its temperature will remain constant: an increase in volume of the gas
would imply a decrease in pressure, and viceversa.

Francesco Saitta, Pordenone Settembre 2019 129


Termodinamica

7.2.5.3 Isochoric processes


An isochoric process (fig.(7.7)), also called a constant-volume process, is
a thermodynamic process during which the volume of the closed system
undergoing such a process remains constant. The work done by the gas
in this case is zero: dL = p ∗ dV = 0.

Figura 7.7: An isochoric process


in the Claperyron Plane

In our practical example we would have the piston locked in a parti-


cular position (for example screwed to close the cylinder): an increase in
temperature would imply in this case an increase in pressure, and vice-
versa. An isochoric process is also known as an isometric process or an
isovolumetric process.

Francesco Saitta, Pordenone Settembre 2019 130


Termodinamica

7.2.5.4 Adiabatic processes


An adiabatic process (fig.(7.8)) is a thermodynamic process in which no
heat is transferred to or from the working gas. If there is no heat transfer-
red all the work done by the gas is enterly connected with a change in the
internal energy of the gas itself, therefore L = −∆U = −ncv ∆T.

Figura 7.8: An adiabatic process


in the Claperyron Plane

In our practical example we would have both the cylinder and the pi-
ston made by perfect thermal insulator, in such a way that there cannot be
any exchange of heat or energy between the gas and the external atmo-
sphere. It can be prooved that in an adiabatic process holds the relation
C
PA VA = PB VB , where γ = Cvp ; C p is the so called "heat capacity at con-
γ γ

stant pressure", the heat necessary to change the temperature of the gas
by 1 degree through a process with constant pressure. We will define also
c p as the “molar heat capacity at constant pressure”.

Francesco Saitta, Pordenone Settembre 2019 131


Termodinamica

7.2.6 The first law of thermodynamics


We are now ready to introduce the ”first law of thermodynamics”, which
is nothing more than the conservation law for internal and mechanical
energy into a thermodynamical system. The starting point of this princi-
ple is the experimental observation that in any thermodynamical process
the difference between the heat exchanged and the work done by the sy-
stem is a particular function depending only on the initial and final state
of the system, representing the total energy of the system. This function
turns out to be (when taking into account ideal gases) the internal energy
previously defined.

1. Energy conservation in an adiabatic process. If there are no exchanges


of heat with the environment (as in an adiabatic process) than conser-
vation of energy requires that the work L done by the gas be equal to
the loss of its internal energy U:

L = −∆U

where the minus sign means that the internal energy is lost when the
gas expands and does work.

2. Energy Conservation in an Isothermal Process. If we consider an


isothermal expansion of the gas we notice that the internal energy has
to be constant (U = U ( T )). If the internal energy has to be constant all
the work done by the gas has to turn into heat, and all the heat given
to the gas has to be spent by means of mechanical work. The work L
done by the gas must be equal to the heat Q absorbed by it:

L=Q

3. Energy conservation in an isochoric process. If the volume of the gas


is not changing all the heat taken or given to the gas cannot transform
in mechanical work, since the piston cannot move, but it transforms
into an increase or loss of internal energy; if the gas acquires heat it
will increase the internal energy, otherwise the internal energy will
decrease:
Q = ∆U

4. Energy conservation in a general process: First law of thermodyna-


mics. In a general process usually both heat and internal energy are
converted into work. Since heat absorbed by the gas contributes to ma-
king work, and a loss of internal energy can be turned into work, the
total balance of work, heat and internal energy is:

L = Q − ∆U,

or
∆U = Q − L

and is known as the first law of thermodynamics. It says that the


internal energy gained by a system must be equal to the heat absorbed
by the system minus work done by the system.

Francesco Saitta, Pordenone Settembre 2019 132


Termodinamica

5. First law and the Mayer relationship . Let’s consider a general process
in the Clapeyron plane: it can be decomposed into many tiny isobaric
processes. For each of these processes holds
∆U = Q − L,
where4 4
The first relation holds for any
∆U = ncv ∆T, process, the second one holds
since each process is an isoba-
Q = nc p ∆T ric process, and the last one can
be obtained using the universal
L = p∆V = nR∆T. law of ideal gases.
We can conclude that for any thermal process the first principle of ther-
modynamics, the conservation of energy in thermal processes, can be
written as the Mayer relationship:
c p − cv = R.

This law connects a universal constant, R, with two particular constants


depending on the gas we are considering: no matter which gas we are
dealing with, the difference between c p and cv will equal the constant
R!

7.3 Rendimento delle macchine termiche

Dal punto di vista tecnologico di chi progetta una macchina termica è


interessato a capire quanta percentuale del calore che viene ceduto alla
macchina termica dall’esterno verrà convertito in lavoro. Questo indica-
tore, solitamente rappresentato con la lettera greca η, è detto rendimento
della macchina termica:
L
η= (7.3.1)
Qa

Dove L è il lavoro prodotto dalla macchina termica e Q a il calore assorbito


dalla macchina stessa (ceduto alla macchina dall’ambiente esterno). Dalle
considerazioni del paragrafo precedente, visto che una macchina termica
deve essere ciclica per poter essere utilizzabile in modo industriale e non
essere “usa e getta”, possiamo definire il rendimento di una macchina
termica su un singolo ciclo:
L Q a − Qc Qa
η= = = 1−
Qa Qa Qc
con Qc il valore assoluto del calore ceduto dalla macchina termica all’am-
biente esterno. La costruzione di macchine termiche iniziò già tra il di-
ciassettesimo ed il diciottesimo secolo in Inghilterra per opera di scienziati
come Denis Papin (Blois, 1647 - Londra 1714), Thomas Savery (Shilstone,
1650 - Londra 1715), Thomas Newcomen (Dartmouth, 1663 - Londra 1729)
e James Watt (Greenock, 1736 - Handsworth, 1819); fu però nella Francia
del secolo successivo che la tecnica si coniugò con lo studio scientifico ana-
litico ed approfondito della termodinamica, per opera sopratutto di Sadi
Carnot (Parigi, 1796 - Parigi, 1832). Vediamo di seguito alcune delle più
famose macchine termiche, alcune delle quali hanno contribuito in mo-
do decisivo alla rivoluzione industriale del diciannovesimo secolo e sono
state protagoniste della tecnica fino ai giorni nostri.

Francesco Saitta, Pordenone Settembre 2019 133


Termodinamica

7.3.1 Macchina di Stirling


La macchina di Stirling fu realizzata e brevettata nel 1816 dal pastore pro-
testante scozzese Robert Stirling (Methven, 1790 - Methven 1878), ed è
ancora oggi in commercio. Si tratta di un motore ad aria calda di cui non
studieremo i dettagli ingegneristici ma che risponde al ciclo termodinami-
co in figura (7.9): il motore lavora tra due temperature Tc la temperatura
più calda e T f ed i due volumi V1 e V2 con due trasformazioni isocore e
due isoterme.
Studiamo l’energetica del ciclo di Stirling:

• A −→ B: La trasformazione è isocora, per cui il lavoro fatto dal gas


è nullo e il calore assorbito è dato dalla differenza di energia interna
Q = ∆U = ncv ∆T = ncv ( Tc − T f ) > 0
Figura 7.9: Ciclo termodinami-
co di Stirling A → B → C → D
• B −→ C: La trasformazione è isoterma, per cui il lavoro fatto dal
tra le due temperature Tc e T f e
gas è uguale al calore assorbito nell’espansione da V1 a V2 Q = L = i due volumi V1 e V2 .
nRTc ln(V2 /V1 ) > 0

• C −→ D: La trasformazione è nuovamente isocora, con calore ceduto


dal gas Q = ∆U = ncv ∆T = ncv ( T f − Tc ) < 0

• D −→ A: La trasformazione è isoterma, con lavoro subito dal gas


uguale al calore ceduto per la compressione da V2 a V1 Q = L =
nRT f ln(V1 /V2 ) < 0

I due calori scambiati nelle trasformazioni isocore sono chiamati di rigene-


razione e non vengono scambiati con l’esterno: la macchina è studiata in
modo che il calore di rigenerazione ceduto nella trasformazione C −→ D
viene restituito al gas nella trasformazione A −→ B; il rendimento della
macchina di Stirling (ideale) è dunque
nRT f ln(V1 /V2 )
η = 1−
nRTc ln(V2 /V1 )
Tf
= 1−
Tc

7.3.2 Macchina di Carnot


La macchina di Carnot fu concepita dal fisico francese in una sua ope-
ra fondamentale (Carnot, 1824), nella quale pose le basi per moltissi-
mi concetti fondanti per la termodinamica che stiamo studiando, tra cui
l’introduzione della macchina di Carnot, descritta in figura (7.10).
Questa macchina è una macchina teorica, pensata dal fisico francese
con l’intento di costruire la macchina termica lavorante tra due sorgen-
ti termiche con il massimo rendimento possibile: per questo pensò di
connettere le due isoterme dove avviene lo scambio di calore con le sor-
genti con due trasformazioni adiabatiche, in cui non c’è scambio di calore. Figura 7.10: Ciclo termodinami-
Studiando l’energetica della macchina: co di Carnot A → B → C → D
tra le due temperature Ti e T f .
• A −→ B: La trasformazione è adiabatica, per cui il calore scambiato dal
gas è nullo ed il lavoro fatto è dato dalla differenza di energia interna
L = ∆U = ncv ∆T = ncv ( Tc − T f ) > 0

Francesco Saitta, Pordenone Settembre 2019 134


Termodinamica

• B −→ C: La trasformazione è isoterma, per cui il lavoro fatto dal


gas è uguale al calore assorbito nell’espansione da VB a VC Q = L =
nRTc ln(VC /VB ) > 0

• C −→ D: La trasformazione è nuovamente adiabatica, con lavoro


subito dal gas L = ∆U = ncv ∆T = ncv ( T f − Tc ) < 0

• D −→ A: La trasformazione è isoterma, con lavoro subito dal gas


uguale al calore ceduto per la compressione da VD a VA Q = L =
nRT f ln(VA /VD ) < 0
Anche in questo caso possiamo calcolare il rendimento con la (??):
nRT f ln(VC /VB ) T f ln(VC /VB )
η = 1− = (7.3.2)
nRTc ln(VA /VD ) Tc ln(VA /VD )
Possiamo ottenere una relazione migliore andando a valutare il rappor-
to VC /VB ed il rapporto VA /VD sfruttando le leggi delle trasformazioni
adiabatiche ed isoterme. Valgono sicuramente:
γ γ
p A VA = p B VB

p B VB = pC VC
γ γ
pC VC = p D VD
p D VD = p A VA
che possono essere riscritte come:
γ γ
p A VA = p B VB
VC
p B = pC
VB
γ
VD
pC = p D γ
VC
VA
pD = p A
VD
che ci suggerisce di sostituire tutte le pressioni delle ultime tre equazioni
per ottenere:
γ −1 γ −1
γ VA VD VB
p A VA = p A γ −1
VC
che si può scrivere come
  γ −1   γ −1
VA VB
=
VD VC
ovvero
VA V
= B
VD VC
che sostituito nel rendimento della macchina di Carnot (7.3.2) restituisce
un’espressione molto più semplice:

Tf
ηc = 1 − (7.3.3)
Tc

È facile notare come questo rendimento sia uguale a quello di una macchi-
na di Stirling che lavora tra le stesse due temperature; ci daremo ragione
di questo fatto nei prossimi paragrafi.

Francesco Saitta, Pordenone Settembre 2019 135


Termodinamica

7.3.3 Macchina di Otto (motore 4 tempi)


La macchina di Otto, conosciuta anche come motore a 4 tempi, ancora
oggi alla base della costruzione della maggior parte dei motori a benzina,
fu proposta dall’ingegnere tedesco Nikolaus August Otto 5 nel 1876. La 5
(Holzhausen, 1832 - Colonia,
1891)
macchina dal punto di vista termodinamico ideale è descritta in figura
(7.11).
Quando questa serie di trasformazioni termodinamiche è implementata
per il funzionamento di un motore a benzina dobbiamo immaginare un
sistema come rappresentato in figura (7.12). Descriviamo qui i 4 tempi

Figura 7.11: Trasformazioni ter-


modinamiche per la macchina
di Otto a 4 tempi A → B → C →
D → E → B → A.

Figura 7.12: Il cuore ter-


modinamico di un motore a
scoppio

della macchina:

• Aspirazione A −→ B: il pistone si trova nella sua posizione più alta,


si apre la valvola di aspirazione, viene inserita dal carburatore (o dal-
l’impianto di iniezione) il gas (una miscela di aria e benzina nel caso
del motore a benzina) a pressione costante in quanto l’apertura della
valvola mette a pressione atmosferica il gas all’interno del cilindro. Il
gas si espande quindi dal volume V1 al volume V2 .

• Compressione B −→ C: si chiude la valvola di aspirazione, il gas al-


l’interno del cilindro subisce una compressione dovuta all’inerzia del
pistone che nel ciclo ideale possiamo considerare come adiabatica. In
questa fase non vi è scambio di calore con l’esterno ed il lavoro subito
dal gas è dato da L = −∆U = ncv ∆T = ncv ( TC − TB ) < 0

• Scoppio ed espansione C −→ D −→ E: quando il pistone si trova nel


punto più alto una candela posta tra le due valvole scocca una scintilla;
la temperatura del gas aumenta molto e molto velocemente (a volu-
me costante) provocando la combustione del gas che di conseguenza
si dilata spingendo nuovamente il pistone verso il basso. Nel ciclo

Francesco Saitta, Pordenone Settembre 2019 136


Termodinamica

ideale questa trasformazione può essere di nuovo considerata adia-


batica. Nella fase isocora non vi è lavoro ed il gas acquista il calore
Q = ∆U = ncv ∆T = ncv ( TD − TC ) > 0. Nella fase adiabatica il gas si
espande senza scambiare calore con l’esterno, dunque compie il lavoro
L = −∆U = ncv ∆T = ncv ( TE − TD ) > 0

• Scarico E −→ B −→ A: quando il pistone si trova nella sua posizione


più bassa, si apre la valvola di scarico permettendo al gas di uscire
dal cilindro: a volume costante la pressione torna al valore iniziale di
pressione atmosferica, di seguito il pistone per inerzia torna verso l’alto
spingendo il gas fuori dal cilindro. Nella fase isocora il gas non compie
lavoro, cede il calore Q = ∆U = ncv ∆T = ncv ( TB − TE ) < 0. Nella fase
isobara il gas esce dal cilindro e fa tornare il pistone nella posizione
iniziale ed il ciclo può ricominciare dall’iniezione.

Il rendimento termodinamico della macchina si può calcolare sul ciclo


B −→ C −→ D −→ B:
ncv ( TE − TB ) T − TB
η = 1− = 1− E (7.3.4)
ncv ( TD − TC ) TD − TC

Possiamo ottenere una relazione migliore andando a valutare il rappor-


to TE /TB ed il rapporto TC /TD sfruttando le leggi delle trasformazioni
adiabatiche:
γ −1
TB VB = TC VCγ−1
γ −1
TE VE = TD VDγ−1
Da cui, sapendo che VC = VD VE = VB e dividendo membro a membro le
equazioni possiamo ottenere

TB T
= C
TE TD
Vale dunque
TB
TE 1 − TE T
η = 1− = 1− E
TD 1 − TC TD
TD

Ecco allora che il rendimento della macchina di Otto è più semplice e in


dipendenza unicamente delle due temperature TB e TD . Usando ancora
le equazioni delle trasformazioni adiabatiche possiamo esprimere questo
rendimento come
γ −1
T V
η = 1 − E = 1 − 1γ−1
TD V 2
in unica dipendenza dei due volumi V2 e V1 , la geometria del cilindro e del
tipo di gas γ = c p /cv . Il rapporto r = V2 /V1 viene solitamente chiamato
rapporto di compressione e di conseguenza il rendimento si esprime come

η = 1 − r 1− γ (7.3.5)

7.3.4 Macchina Diesel


La macchina Diesel, brevettata nel 1892 da Rudolf Diesel 6 , si differen- 6
(Parigi, 1858 - canale della
zia dal punto di vista termodinamico dal motore Otto per la modalità di manica, 1913)

Francesco Saitta, Pordenone Settembre 2019 137


Termodinamica

scoppio del gas, come descritto dal grafico (7.13). La combustione del gas
in un ciclo Diesel non viene innescata dalla scintilla di una candela ma
dalla compressione del gas stesso. La fase di scoppio (l’isocora del ciclo
precedente) in questo caso viene sostituita con un’espansione isobara.
Con un’analisi analoga a quella fatta per il ciclo Otto possiamo trova-
re il rendimento termodinamico della macchina Diesel in funzione delle
temperature TB ,TC ,TD ,TE :

cv TE − TB
η = 1−
c p TD − TC

Per riscrivere in modo più semplice questo rendimento è conveniente


esplicitare il rapporto di compressione r = V2 /V1 ed il rapporto di com- Figura 7.13: Ciclo termodinami-
co Diesel 4 tempi A → B → C →
bustione c = V3 /V2 per poi ottenere il rendimento D → E → B → A.

cγ − 1
η = 1 − r 1− γ (7.3.6)
γ ( c − 1)

7.3.5 Macchina Frigorifero


Definiamo macchina frigorifero una macchina termica il cui effetto è quel-
lo di assorbire calore da una sorgente ad una temperatura T f e cederlo ad
una sorgente Tc , con Tc < T f subendo una certa quantità di lavoro L, che
per il primo principio della termodinamica è dato da L = Q a − Qc . Pen-
sandoci bene questo ciclo termodinamico descrive proprio ciò che accade
nel frigorifero di casa nostra: è un elettrodomestico che prende calore da
un ambiente freddo (l’interno del frigorifero) e lo cede all’esterno (il retro
dei nostri frigoriferi ha solitamente una serpentina che si scalda con l’uso
del frigorifero) subendo il lavoro fornito dalla rete elettrica di casa.
Nel caso delle macchine frigorifere non si parla di rendimento termo-
dinamico ma di coefficiente di prestazione k, che esprime la quantità di
calore ceduto alla sorgente calda rispetto al lavoro subito:

Qc
k= (7.3.7)
L
Figura 7.14: Ciclo frigorifero, il
Applicazione comune del modello frigorifero, sono le pompe di calore: passaggio di calore dalla tempe-
strumenti che vediamo abitualmente nelle case e che chiamiamo solita- ratura più fredda T f e la tempe-
ratura più calda Tc , subendo il
mente condizionatori elettrici. Essi spostano calore da una sorgente a lavoro L
temperatura minore ad una a temperatura maggiore sfruttando il lavoro
della rete elettrica: possono essere usati per scaldare l’interno di un edi-
ficio d’inverno, prendendo il calore dall’esterno e cedendolo all’interno,
o per raffreddarlo d’estate, prendendo il calore dall’interno e cedendolo
all’esterno.

7.4 Secondo principio della termodinamica

Il secondo principio della termodinamica può essere espresso in diver-


se forme, come vedremo nei prossimi paragrafi, ognuna della quali ri-
porta sempre allo stesso concetto: qualsiasi macchina termica possiamo
inventarci, il budget energetico dell’universo sarà sempre a favore della

Francesco Saitta, Pordenone Settembre 2019 138


Termodinamica

dispersione di energia in calore. Applicato alla nostra esperienza quo-


tidiana7 questo principio può sembrare ovvio e per nulla fondamenta- 7
le automobili spostandosi im-
le, ma di fronte ad un’attenta analisi che cercheremo di impostare qui mettono calore nell’atmosfera
sotto forma di gas, così come
ci accorgeremo dell’importanza concettuale che si cela dietro a questa tutti gli impianti industriali; i
considerazione. computer ed i telefoni che usia-
mo ogni giorno emettono ca-
lore nell’ambiente, riflettendoci
7.4.1 Enunciati di Clausius, Kelvin-Planck e loro equivalenza bene qualsiasi azione facciamo
provoca un surriscaldamento in
Enunciato di Clausius Non si può costruire una macchina termica il cui qualche parte dell’ambiente in
cui siamo.
unico risultato sia quello di trasferire calore da una sorgente fredda ad
una sorgente calda.

Enunciato di Kelvin-Planck Non si può costruire una macchina termica


il cui unico risultato sia quello di produrre lavoro assorbendo calore da
un’unica sorgente.

Equivalenza dei due enunciati Per dimostrare l’equivalenza dei due enun-
ciati procederemo con una dimostrazione per assurdo nel seguente modo:
invece di dimostrare che l’enunciato di Clausius (Cl) implica l’enuncia-
to di Kelvin-Planck (KP) Cl =⇒ KP e viceversa KP =⇒ Cl dimostre-
remo che la negazione dell’enunciato di Clausius (nCl) implica la ne-
gazione dell’enunciato di Kelvin-Planck (nKP) nCl =⇒ nKP e viceversa
nKP =⇒ nCl.
nCl =⇒ nKP
Immaginiamo di avere a disposizione una macchina termica che neghi
l’enunciato di Clausius e di collegarla ad una macchina termica che in-
vece rispetta l’enunciato di Kelvin-Planck come in figura (7.15). L’effetto
netto della macchina in figura è quello di assorbire una quantità di calore
Q2 − Q1 dalla sorgente calda, producendo un lavoro L senza cedere calore Figura 7.15: Collegamen-
to di una macchina termica
alla sorgente fredda: negando l’enunciato di Clausius siamo riusciti a ne-
di Kelvin-Planck con una
gare l’enunciato di kelvin-Planck costruendo una macchina termica il cui macchina non-Clausius
unico risultato è quello di produrre lavoro assorbendo calore da un’unica
sorgente!
nKP =⇒ nCl
Immaginiamo ora di avere a disposizione una macchina termica che neghi
l’enunciato di Kelvin-Planck e di collegarla ad una macchina termica che
invece rispetta l’enunciato di Clausius come in figura (7.16). L’effetto netto
della macchina in figura è quello di trasferire il calore Q2 da una sorgente
fredda ad una calda senza subire lavoro dall’esterno: negando l’enunciato
di Kelvin-Planck siamo riusciti a negare l’enunciato di Clausius costruen-
do una macchina termica il cui unico risultato è quello di trasferire calore
da una sorgente fredda ad una sorgente calda!
Figura 7.16: Collegamento di
una macchina termica di Clau-
7.4.2 Enunciato di Carnot ed entropia sius con una macchina non-
Kelvin-Planck
7.4.2.1 Enunciato di Carnot
Non si può costruire una macchina termica che lavora tra due sorgenti con
rendimento superiore al rendimento della macchina di Carnot che lavora

Francesco Saitta, Pordenone Settembre 2019 139


Termodinamica

tra le medesime sorgenti:


η ≤ ηc

Ogni macchina termica reversibile8 che lavori tra due sorgenti termiche 8
definiamo macchina termica
inoltre ha rendimento pari al rendimento della corrispondente macchina reversibile una macchina ter-
mica in cui ogni stato del
di Carnot: gas è perfettamente conosciu-
ηirrev < ηrev = ηc < 1 (7.4.1) to, ciclo che quindi può esse-
re percorso in entrambi i sen-
si (orario ed antiorario nel pia-
7.4.2.2 Entropia termodinamica no di Clapeyron per esempio).
In generale una macchina re-
Nel caso della macchina di Carnot (e quindi di una qualsiasi macchina versibile è un’idealizzazione di
una macchina termica, che può
termica reversibile) è interessante studiare l’uguaglianza tra il rendimento essere approssimata con tra-
generale in funzione del calore scambiato ed il rendimento particolare in sformazioni quasi-statiche, ma
mai realizzata perfettamente in
funzione delle due temperature delle sorgenti:
natura.

Qc Tf Qa Qc
1− = 1− =⇒ = (7.4.2)
Qa Tc Tc Tf

Questa semplice relazione matematica ci dice come se è vero che la quan-


tità di calore assorbita dalla sorgente calda è maggiore della quantità di
calore ceduta alla sorgente calda (ed è bene che sia così da un punto di
vista tecnologico, perché genera lavoro meccanico) è altrettanto vero che
la quantità definita dal rapporto tra il calore scambiato e la temperatura
della sorgente con cui lo ha scambiato è una costante. Ecco allora come
ad un fisico che osserva questi calcoli viene il dubbio che questa quantità,
che si conserva in un ciclo di Carnot, possa essere una quantità interes-
sante dal punto di vista termodinamico. Definiamo allora come entropia
termodinamica9 la seguente quantità S: 9
Introdotta per la prima vol-
ta da Clausius nel 1864 (Clau-
sius, 1864), che prese spunto da
Q
∆S = (7.4.3) una parola greca che significa
T trasformazione per definire una
grandezza fisica che egli inter-
In questo modo possiamo riscrivere l’enunciato di Carnot nel seguente pretò come la traccia di ogni tra-
sformazione termodinamica in
modo: natura.
∆S = 0 (7.4.4)

Ovvero che la variazione di entropia di un gas che subisce un ciclo ter-


modinamico reversibile chiuso è uguale a zero. L’entropia viene definita
per differenze: ogni qual volta si scambia calore con una sorgente ad una
certa temperatura avviene una variazione di entropia del gas: aumenta se
il calore è assorbito, diminuisce se il calore è ceduto. L’unità di misura
dell’entropia è il rapporto tra Joule e Kelvin (J/K). È importante ed inte-
ressante notare come l’entropia sia una funzione di stato termodinamica,
una funzione che dipende solo dai parametri di stato: in un percorso chiu-
so nel piano di Clapeyron infatti la variazione di entropia è nulla, come
si può notare dall’equazione (7.4.2). Questa osservazione è interessante
per il calcolo dell’entropia in una data trasformazione: la sua variazio-
ne non dipende dalla serie di trasformazioni che possiamo fare, ma solo
dallo stato iniziale e dallo stato finale del gas. Supponiamo quindi che il
gas passi da un generico stato A ad un generico stato B con una trasfor-
mazione reversibile: per calcolare la variazione di entropia del gas non è
evidentemente utilizzabile la formula (7.4.3) in quanto gli scambi di calore

Francesco Saitta, Pordenone Settembre 2019 140


Termodinamica

non avvengono a temperatura costante. Gli unici due casi semplici sono il
caso isotermo appunto in cui si può applicare la formula ed il caso adia-
batico in cui non essendoci scambio di calore l’entropia rimane costante.
Nel caso più generale dal punto di vista concettuale occorre dividere il
percorso in moltissimi tratti quasi-statici per cui sia possibile pensare che
il trasferimento di calore avvenga a temperatura costante, e sommare poi
tutti questi contributi; dal punto di vista matematico questo corrisponde
a risolvere un integrale di cui diamo qui solo il risultato:
   
Qi TB VB
∆S = S( B) − S( A) = ∑ Ti
= ncv ln
TA
+ nR ln
VA
(7.4.5)
i

7.4.2.3 Disuguaglianza di Clausius


Nel caso in cui un ciclo termodinamico non sia reversibile non possia-
mo calcolare la variazione di entropia delle trasformazioni del ciclo come
sommatoria dei diversi rapporti tra i calori scambiati e le temperature a
cui sono stati scambiati, ma possiamo sempre ottenere una relazione per
questa sommatoria che chiamiamo disuguaglianza di Clausius:

Qi
∑ Ti
<0 (7.4.6)
i

7.4.2.4 Entropia di sistemi isolati, universo e morte termica dell’universo


Consideriamo un sistema termodinamico isolato, che non scambia calore
con l’esterno, ed immaginiamolo cambiare il suo stato da A a B attraverso
una trasformazione reale irreversibile: non possiamo dunque applicare
l’equazione (7.4.5) per calcolare l’eventuale variazione di entropia nella
trasformazione considerata. La situazione è schematizzata in figura (7.17).
Sicuramente per quanto detto in precedenza vale (7.4.6), che possiamo
dividere in due parti:
   
Qi Qi
∑ Ti + ∑
Ti B→ A
<0 Figura 7.17: La zona ombreg-
i A→ B i giata indica una trasformazio-
ne irreversibile, in cui non co-
E, siccome la trasformazione B → A è reversibile il secondo addendo è nosciamo di preciso lo stato
proprio S( A) − S( B) e si ha del gas in ogni istante della
trasformazione
 
Qi
∑ Ti + S( A) − S( B) < 0
i A→ B

ovvero  
Qi
S( B) − S( A) > ∑ Ti
i A→ B

Ma se il sistema è isolato come da ipotesi significa che il gas non scambia


calore con l’esterno e dunque si ha sempre

S( B) > S( A) (7.4.7)

Questo significa che l’entropia di un sistema isolato aumenta sempre, ogni


volta che si compie una trasformazione termodinamica irreversibile al suo

Francesco Saitta, Pordenone Settembre 2019 141


Termodinamica

interno, e siccome come abbiamo già visto in precedenza se compiamo tra-


sformazioni reversibili l’entropia non varia possiamo dire che In un siste-
ma termodinamico isolato l’entropia non può mai diminuire, qualsiasi
trasformazione accada, ed ogni qual volta si compie una trasformazione
irreversibile essa aumenta sempre. Il sistema isolato che più caratterizza
lo studio della fisica e stimola l’intelletto umano è sicuramente l’universo
intero. Con il passare del tempo e lo svolgersi delle trasformazioni ir-
reversibili reali all’interno dell’universo l’entropia tenderà sempre ad au-
mentare, tutta l’energia tenderà a dissiparsi in calore e la temperatura ad
uniformarsi in tutto lo spazio disponibile: i fisici del tempo previdero così
la morte termica dell’universo. In realtà la cosmologia moderna prevede
diversi fenomeni ed il futuro dell’universo non è così certo come potrebbe
sembrare dal secondo principio della termodinamica.

7.4.2.5 Espansione libera di un gas


L’esempio più comune che viene fatto di un sistema chiuso in cui l’en-
tropia aumenta è l’espansione libera di un gas. Immaginiamo di avere
un gas all’interno di una stanza di volume fissato isolata termicamente
dal resto dell’universo, all’interno della quale vi è una piccola boccetta di
gas che ad un certo punto viene aperta. Quello che possiamo dire spe-
rimentalmente e dalla nostra esperienza quotidiana, è che il gas per sua
natura si distribuisce nella stanza in modo uniforme, occupando tutto il
volume a sua disposizione. Questa trasformazione si chiama espansione
libera in quanto il gas non scambia calore con l’esterno né compie lavoro
sulle pareti della scatola: per il primo principio della termodinamica non
cambia nemmeno la sua energia interna, la sua temperatura finale è quin-
di uguale alla sua temperatura finale. Nel piano di Clapeyron dunque,
pur non conoscendo i dettagli della trasformazione realmente avvenuta,
possiamo dire che i due punti iniziale A e finale B si trovano sulla stessa
curva isoterma alla temperatura TA , come mostrato in figura (7.18).
Da quanto detto in precedenza quindi possiamo calcolare la variazione
di entropia del sistema usando la legge (7.4.5) sapendo che la temperatura
finale ed iniziale sono uguali tra loro. A seconda della grandezza della
boccetta dunque ci sarà una certa variazione di entropia del sistema, che
Figura 7.18: Espansione libera
sicuramente però sarà positiva10 : di un gas perfetto
Il logaritmo di una quantità
10
     
TB VB VB
∆S = ncv ln + nR ln = nR ln >0 maggiore di 1 è sempre positivo
TA VA VA

7.4.2.6 Entropia e freccia del tempo


L’entropia spesso viene considerata come freccia del tempo, indicatore
della successione degli eventi che avvengono in natura. Molti fenomeni
meccanici sono simmetrici rispetto al tempo: immaginando di guardare
l’urto tra due palline non possiamo pensare di scoprire se stiamo guar-
dando un fenomeno dal passato verso il futuro o viceversa; guardando un
processo termodinamico invece abbiamo il modo per scoprire la direzio-
ne del tempo, dire se stiamo guardando il fenomeno dal passato verso il
futuro o viceversa: se l’entropia sta aumentando il sistema sta evolvendo
sicuramente dal passato verso il futuro! In fisica si parla spesso di questa

Francesco Saitta, Pordenone Settembre 2019 142


Termodinamica

simmetria (o non simmetria) rispetto al tempo, si parla di simmetria T:


esistono importantissimi teoremi e ricerche di fisica delle particelle che
mirano a studiare la simmetria rispetto al tempo o la sua violazione.

7.4.2.7 Entropia statistica e disordine


La definizione statistica di entropia fu introdotta da Ludwig Boltzmann11 11
La cui definizione si trova
alla fine del diciannovesimo secolo (Boltzmann, 1877). L’idea che Boltz- anche incisa sulla tomba dello
scienziato austrico a Duino, vi-
mann propose è la seguente: l’entropia di un sistema è legata alla pro- cino a Trieste dove si tolse la vita
babilità che ha un determinato stato di esistere. Questa definizione si è durante una vacanza nel 1906
sviluppata nel contesto della meccanica statistica , trova senso nella de-
scrizione microscopica di un gas e dà origine al concetto di entropia come
disordine. Il significato di disordine che si lega a quello di entropia non è
di senso comune ed è strettamente legato alla meccanica statistica: tanto
più la configurazione è probabile quanto più è disordinata e con entropia
elevata. La famosa equazione di Boltzmann è la seguente

S = k ln(W ( A)) (7.4.8)

dove k è la costante di Boltzmann e W ( A) è il numero di microstati del gas


che generano lo stato termodinamico A. Possiamo fare un esempio molto
semplice e schematico, ma significativo per la comprensione di questa
definizione di entropia. Immaginiamo di avere una scatola con al suo
interno 4 particelle di gas ed una parete di separazione nel mezzo della
scatola. In figura (7.19) mostriamo le possibili configurazioni in cui il gas

Figura 7.19: Microstati possibi-


li per 4 particelle in una scatola
con due spazi disponibili

può disporsi nella scatola: lo stato macroscopico con più microstati che lo
definisce è quello in cui ci sono due particelle da una parte e dall’altra;
per la statistica è lo stato più probabile, per la definizione di Boltzmann
è lo stato con maggiore entropia, per esperienza è lo stato in cui un gas
tende ad evolvere liberamente, qualsiasi fosse la sua condizione iniziale!

Francesco Saitta, Pordenone Settembre 2019 143


Termodinamica

Francesco Saitta, Pordenone Settembre 2019 144


Termodinamica

7.5 Esercizi

1. Quanto calore è necessario affinché un kilogrammo di ghiaccio alla temperatura T = 250 K evapori
completamente?

[Q = 3, 1 × 106 J]
2. Un litro di gas perfetto raddoppia la sua pressione a temperatura costante, quale sarà il suo volume
alla fine della trasformazione?

[V = 0, 0005 m3 ]
3. Che volume occupa una mole di gas perfetto alla temperatura T = 300 K che si trova alla pressione
atmosferica?

[V = 0, 025 m3 ]
4. Di quanto varia in percentuale la temperatura di un gas perfetto durante un’espansione isobara dal
volume iniziale V1 = 5 l al volume finale V2 = 8 l?

[varia del 60%]


5. Un gas perfetto in equilibrio alla pressione atmosferica in un contenitore di volume V = 2 m3 si trova
alla temperatura T = 200 K. Quante moli di gas sono contenute nel contenitore?

[n = 122 mol]
6. Il tappo di una bottiglia da un litro sopporta al massimo una pressione di 50 atmosfere. All’interno
della bottiglia si trovano 3 moli di gas perfetto alla temperatura T = 300 K. Il tappo manterrà la botti-
glia chiusa o verrà scoperchiato dalla pressione del gas?

[No, la pressione del gas è P = 74 atm]


7. Ad un cubetto di ghiaccio di massa m = 300 g alla temperatura t = −20 vengono fornite 300kcal.
Quale sarà la situazione finale del cubetto di ghiaccio?

[Evapora tutto]
8. Quanta massa di ghiaccio alla temperatura di passaggio di stato verrà fusa con la quantità di calore
necessaria per far evaporare 1 g d’acqua?

[m = 7 g]
9. A che temperatura si troveranno 5 moli di gas perfetto che subiscono una trasformazione isocora dalla
situazione iniziale PA = 3 atm, TA = 200 K fino alla situazione finale in cui PB = 10 atm? E se le moli
di gas fossero state 2, cosa sarebbe cambiato?

[T = 667 K, indipendentemente dalle moli di gas]


10. Si calcoli la quantità di aria (in moli) presente in un’aula scolastica. Si supponga che la stanza sia
larga 5 metri, lunga 6 metri, altra 3 metri, che la pressione sia quella atmosferica e che la temperatura
sia primaverile (t = 23C). Come cambia questo numero d’inverno senza riscaldamento nella stanza
(t = 3C)? Si supponga che l’aria che respiriamo sia un gas perfetto.

[n = 3706 mol in primavera; n = 3975 mol in inverno]

Francesco Saitta, Pordenone Settembre 2019 145


Termodinamica

11. Si calcoli la velocità quadratica media di una certa quantità di idrogeno molecolare (H2 ) allo stato
gassoso ed alla temperatura T = 300 K.

[vqm = 2500 m/s]


12. Si calcoli l’energia cinetica media di una certa quantità di elio gassoso (He) alla temperatura T = 0C

[Ēc = 5, 6 × 10−21 J]
13. Che tipo di gas ha, alla temperatura T = 300 K, un’energia cinetica media Ēc = 1, 449 × 10−20 J?

[Triatomico]
14. Che pressione imprimono sulle pareti del contenitore in cui si trovano 5 moli di un gas monoatomico
di densità d = 5 × 10−25 kg/m3 e velocità quadratica media vqm = 1000 m/s?

[P = 5 atm]
15. Un mole di gas monoatomico si trova alla temperatura Ti = 20C. Se un certo macchinario compie
un lavoro L = 100 J sul gas, a quale temperatura si troverà il gas, potendo trascurare qualsiasi tipo di
dispersione?

[T = 301 K]
16. Qual è la temperatura di una mole di gas descritto dal punto (1; 2 × 104 ) nel Piano di Clapeyron? Si
assumano le unità di misura del SI.

[T = 2407 K]
17. Si studi l’energetica del ciclo termodinamico descritto in figura (7.20). Si assuma n = 3 mol. In
particolare si trovi il lavoro fatto dal gas nel ciclo.

Figura 7.20: Il ciclo segue la


sequenza A → B → C → D

[L = 966 J]
18. Si studi l’energetica del ciclo termodinamico descritto in figura (7.21). Si assuma n = 1 mol. In
particolare si trovi il lavoro fatto dal gas nel ciclo ed il calore assorbito.

[L = 570 J; Q = 2570 J]

Francesco Saitta, Pordenone Settembre 2019 146


Termodinamica

Figura 7.21: Il ciclo segue la


sequenza A → B → C → D

19. Quanto calore serve affinché, in una trasformazione isocora, la temperatura di una mole di gas per-
fetto monoatomico aumenti la sua temperatura da T1 = 200 K a T2 = 600 K?

[Q = 4986 J]
20. Qual è l’energetica del ciclo termodinamico descritto in un piano di Clapeyron (V (m3 ); P( Pa)) da
una circonferenza di centro C (3; 30000) e raggio r = 10? In particolare si trovi il lavoro fatto dal gas
nel ciclo ed il calore assorbito.

[L = 314 J; Q = 314 J]
21. Si calcoli il rendimento del ciclo termodinamico dell’esercizio (17)

[η = 30%]
22. Si calcoli il rendimento del ciclo termodinamico dell’esercizio (18)

[η = 25%]
23. Si calcoli il lavoro fatto da 5 cicli di una macchina ideale di Carnot dal rendimento ν = 60%, che
assorbe Q = 500 J dalla sorgente calda.

[L = 1500 J]
24. Si calcoli la variazione di energia interna di 10 moli di gas perfetto biatomico che subiscono una
trasformazione isobara dallo stato (5; 2) nel piano di Clapeyron (V (l ); P( atm)) fino allo stato (9; 2).

[∆U = 20 atm l ]
25. Si risolva l’esercizio (24) nel caso in cui il gas sia monoatomico.

[∆U = 1200 J]
26. Si calcoli il lavoro fatto da una mole di gas monoatomico perfetto in un’espansione adiabatica che
triplichi il volume del gas a partire dallo stato (2; 5) nel piano di Clapeyron (V (l ); P( atm)).

[L = 791 J]

Francesco Saitta, Pordenone Settembre 2019 147


Termodinamica

27. Si calcoli la variazione di energia interna di due moli di gas biatomico che passi dalla temperatura
T1 = 200 K alla temperatura T2 = 350 K.

[∆U = 6233 J]
28. Si calcoli il calore ceduto alla sorgente fredda di una macchina di Carnot di rendimento η = 0, 7, se
il lavoro della macchina è pari a L = 500 J.

[Q = 214 J]
29. Si calcoli il calore assorbito da una mole di gas perfetto che percorra un ciclo definito dal triangolo
ABC nel piano di Clapeyron (V (m3 ); P( Pa)) definito da A(2; 20000), B(4; 20000) e C (3; 40000).

[Q = 20000 J]
30. Un cilindro di volume V = 1, 5 m3 chiuso ermeticamente contiene due moli di gas biatomico alla
temperatura T = 300 K. Quale temperatura raggiungerà il gas se dall’esterno si fornisce Q = 4000 J di
calore?

[T f = 396 K]
31. Si calcoli la variazione di entropia del gas nella trasformazione descritta nel problema (30)

[∆S = 11, 5 J/K]


32. Si calcoli la variazione di entropia di una mole di un gas perfetto monoatomico che si espande libe-
ramente fino a triplicare il suo volume.

[∆S = 9, 1 J/K]
33. Si calcoli la variazione di entropia dell’universo nello scioglimento di un cubetto di ghiaccio di massa
m = 200 g alla temperatura di fusione messo a contatto con una sorgente a temperatura Ts = 300 K.

[∆S = 22 J/K]
34. Si calcoli la variazione di entropia di una mole di gas perfetto monoatomico che subisce una trasfor-
mazione irreversibile dal punto (2; 6) al punto (3; 4) nel piano di Clapeyron (V (l ); P( atm)).

[∆S = 3, 4 J/K]
35. Una macchina di Carnot lavora tra due sorgenti costituite l’una da un enorme blocco di ghiaccio a
T f = 0C, l’altra da un’enorme quantità di acqua in ebollizione. Si calcoli il rendimento della macchina
di Carnot ed il lavoro svolto quando si è fusa una quantità di ghiaccio m = 10 Kg.

[η = 27%; L = 1, 2 × 106 J]
36. Una certa quantità di gas perfetto, inizialmente nello stato con pressione pari a 101 kPa, volume pari
a 25 l e temperatura pari a 300 K, subisce due trasformazioni successive. Dapprima la temperatura au-
menta a pressione constante, raggiungendo il valore di 400 K. Successivamente la temperatura rimane
costante, mentre il volume viene dimezzato. Dopo aver rappresentato le trasformazioni sul piano di
Clapeyron, si determinino i valori finali delle variabili che descrivono lo stato del gas.

[P3 = 202 kPa; V3 = 17 l; T3 = 400 K, n = 1 mol]


37. Un motore termico di rendimento η = 25% produce 18,3 kJ di lavoro. Quanto calore deve assorbire
per svolgere tale lavoro?

Francesco Saitta, Pordenone Settembre 2019 148


Termodinamica

[Q = 73200 J]
38. Si calcoli l’aumento di entropia dell’universo quando 10 g di ghiaccio a 273 K sciolgono in 100 kg
d’acqua a 293 K. Si supponga che la temperatura dell’acqua rimanga costante.

[∆S = 0, 83 J/K]
39. Un ragazzino lancia una palla di massa m = 200 g con una velocità iniziale v0 = 50 m/s. Di quanto
è aumentata l’entropia nell’universo quando la pallina si ferma dopo il lancio? Si supponga che la
temperatura dell’ambiente in cui il ragazzino lancia la palla sia T = 290 K

[∆S = 0, 86 J/K]
40. A un tizio scivola di mano un oggetto di massa m = 2 kg da un’altezza h = 1, 3 m in una giornata
di sole (t = 30 C). Di quanto aumenta l’entropia dell’universo quando l’oggetto è caduto a terra?

[∆S = 0, 08 J/K]
41. Describe the energetic balance in a ciclic process ABCD:
• AB adiabatic process: TA = 300K, TB = 400K, VA = 2lt, VB = 1lt
• BC isothermal process: VC = 3lt
• CD adiabatic process
• DA isothermal process
Assume 10 moles of a monoatomic gas. Show the process in the Clapeyron plane.

[L = 34 kJ; Q = 34 kJ]
42. Find the work done by one mole of monoatomic ideal gas in the cycle described in fig.7.22. In which
of these processes the gas gains/yields heat?

Figura 7.22: The cycle follows


the sequence A → B → C → A

Francesco Saitta, Pordenone Settembre 2019 149


Termodinamica

[L = 1200 J ]
43. Six moles of a biatomic ideal gas changes its state gaining 40J of internal energy and 100J of heat.
What is the amount of work done by the gas? What happens if the gas would be monoatomic instead
of biatomic?

[L = 60 J]
44. One mole of a monoatomic ideal gas expands isobarically gaining ∆U = 10J of internal energy. As-
suming that it expansion is ∆V = 10l, what is its pressure?

[P = 667 Pa]
45. Find the work done by one mole of monoatomic ideal gas in the cycle described in fig.7.23. In which
of these processes the gas gains/yields heat?

Figura 7.23: The cycle follows


the sequence A → B → C →
D→A

[L = 1600 J]
46. One mole of a monoatomic ideal gas at temperature T1 = 230K compresses adiabatically to a tem-
perature T2 = 300K. Find the work done by the gas. If the gas was initially occupying a volume
V1 = 10m3 , find V2 .

[L = −873 J, V2 = 6.7 m3 ]
47. Ten moles of a monoatomic ideal gas at temperature T1 = 260K, and volume V1 = 20l expands
isobarically to a volume V2 = 40l. Then it is compressed (adiabatically) back to V1 . Finally it returns
to the first state. Draw the cycle in the Clapeyron plane, find the work done by the gas in the cycle,
verifying the first law of thermodynamics.

[L = −16668 J]

Francesco Saitta, Pordenone Settembre 2019 150


Termodinamica

48. Find the work done by one mole of monoatomic ideal gas in the cycle described in fig.7.24. In which
of these processes the gas gains/yields heat? Find ∆U A→ D .

Figura 7.24: The cycle follows


isoth.
the sequence A −−→ B →
isoth.
C −−→ D

[L = 520 J,∆U A→ D = 300 J]


49. Find the entropy change of three moles of monatomic gas that performs a transformation isobaric
starting from (4 l; 2 atm) and doubling its volume.

[∆S = 43 J/K]
50. Find the entropy change of two moles of diatomic gas that performs a reversible transformation from
A to B: TA = 200 K, VA = 80 m3 , TB = 400 K and VB = 10 m3 .

[∆S = −5, 8 J/K]

Francesco Saitta, Pordenone Settembre 2019 151


Bibliografia

Berkeley G. (1721). De Motu.

Boltzmann L. (1877). Über die Beziehung zwischen dem zweiten Hauptsatz der mechanischen Wärme-
theorie und der Wahrscheinlichkeitsrechnung respektive den Sätzen über das Wärmegleichgewicht.
Wiener Berichte, 2(76), 373–435.

Carnot S. (1824). Réflexions sur la puissance motrice du feu et sur les machines propres à développer cette
puissance.

Cavendish H. (1798). Experiments to Determine the Density of the Earth. Philosophical Transactions of the
Royal Society of London, 88, 496–526.

Clausius R. (1864). Abhandlungen über die mechanische Wärmetheorie.

Descartes R. (1644). Principia Phiolosphie.

Euler L. (1765). Theoria motus corporum solidorum seu rigidorum.

Feynman R. P. (2001). La fisica di Feynman.

Galilei G. (1588-1592). De Motu Antiquiora.

Galilei G. (1623). Il saggiatore.

Galilei G. (1632). Dialogo sopra ai due massimi sistemi del mondo.

Galilei G. (1638). Discorsi e dimostrazioni matematiche intorno a due nuove scienze.

Kepler J. (1609). Astronomia nova.

Kepler J. (1619). Harmonice mundi.

Landau L. D.; Lifshitz E. M. (1999a). Fisica Teorica, volume 1.

Landau L. D.; Lifshitz E. M. (1999b). Fisica Teorica, volume 5.

Leibniz G. W. (1849-1863). Mathematische Schriften. herausg. von C.I.Gerardt, Berlino Halle.

Mach E. (1893). The Science of Mechanics - A Critical and Historical Account of its Development.

Newton I. (1687). Philosophiae Naturalis Principia Mathematica.

Francesco Saitta, Pordenone Settembre 2019 153

Potrebbero piacerti anche